Doug’s Post

Doug Cotton has been dropping comments here for a couple of years now.   Like many skeptic blogs, the threads become polluted with his unusual brand of CO2 based climate change denial.  His argument has changed dramatically over the years but in fairness (or perhaps unfairness) to him, he says it has not.  Now I think I have been more patient here than any other blog with his unusual proclamations but have been forced to snip many of his recent comment.  Doug has even surpassed me as the most snipped commenter here and while I was well ahead of other readers he leaves me running a distant second.

I have spent a few days this week asking and Doug has spent time answering questions about his theory.   Currently, he has talked himself into a very tight corner with respect to his CO2 doesn’t cause warming theory.    In fact, it seems pretty clear to me that he has disproven his own theory but we are awaiting an answer.   I decided to put it here because the old thread was way too long.

We are considering two planets with equal albedo.  Both planets are dry and one has a non-absorbing Nitrogen (N2) atmosphere, the other has an absorbing Carbon Dioxide (CO2) atmosphere.  They experience the same solar input and all other features are the same.  Doug has proclaimed the truth of Loschmidt’s temperature gradient which matches the dry adiabatic lapse rate so we agree in the existence of the gradient, he has agreed that the planets each emit at the same temperature when viewed from space, and he admits that the average emission altitude of the CO2 planet is higher than the ground level emission of our Nitrogen planet.

If the atmosphere is cooler as we go higher in altitude, and the emission temperatures of both are the same yet the average emission altitude of the CO2 planet is non-zero, then the ground temperature of the CO2 planet must be warmer than the N2 planet – global warming theory is proven.   Doug doesn’t seem to yet realize just how much of a pickle he has gotten himself into and at the time of writing this post he has gone offline for the night (in his area) but when he comes back, I anticipate he will explain how his theory can correct these statements such that his theory is not proven wrong.

I did not expect his admission of the above points as they completely disprove his conclusion that CO2 does not cause warming.  He must rectify the conflict in his reasoning or concede the point.

We shall see.

 

281 thoughts on “Doug’s Post

  1. Just for completeness, Doug has repeatedly challenged that anyone who could prove his book wrong would receive $5000 as a payment. I have already freed him from that obligation here as we don’t want money influencing the integrity of the discussion.

      1. In the context of there being substantial evidence of natural climate cycles (predominantly those with periodicity of 1000 years and 60 years superimposed thereon) which are very, very strongly correlated with the inverted plot of the scalar sum of the angular momentum of the Sun and all the planets, and there being no evidence of temperature data varying from such cycles, you have absolutely no empirical evidence to support this contention, nor any valid physics, because radiation to a planet’s surface is not the primary determinant of its temperature, and you cannot prove it to be the case on Earth or any other planet.

      2. Unfortunately the problem isn’t that simple. You have more than just the atmospheric temp to consider. You have to look at ocean, aerosols and other factors as well.

        1.3 is not something to worry about though is it. It probably is dramatically beneficial.

  2. Jeff Id,
    Quite agree with you, but why introduce an adiabatic lapse rate? In no case it will be adiabatic. A gradient is always related to heat flux and in an atmosphere of N2, there is no general upward flux.

    1. I agreed with it for two reasons. First, I’m not convinced Loschmidt was wrong, perhaps you can shed some light on that for me. I didn’t participate in the blog excitement on the matter a couple of years ago and don’t see any big issues like second law violations. Second, any planet will have conductive transfer to the surface layer which should create convection so you will get some level of gradient. If we simply admit the gradient exists and then admit the emission altitude increased the game is over and Doug appears to do both of those.

      1. Jeff Id,

        On the substance of the mechanism of greenhouse effect, I agree with you, that is not the question.

        The Loschmidt hypothesis is an intelectual curiosity. In fact, gradient and heat flow are inseparable. On the other hand, a supposedly adiabatic gradient can only exist with convection and there is no convection without a thermal profil imbalance, no thermal profil imbalance without heat flux that is to say without a heat source on one side and dissipation on the other side.

        But precisely because GHG troposphere is not an adiabatic medium and the critical gradient is not the adiabatic gradient.

        1. Phi

          I don’t agree. The gradient arises solely due to vertical air movements, up and own. In fact it requires both. Convection is one source of movement but actually any process that produces advective air movement will generate vertical mixing. Any advection that moves air horizontally from A at the surface to B somewhere else at the surface for example requires that the space at A be refilled with more incoming air. And the air that was at B has to move aside to allow the incoming air from A to fill that space.

          So the inflow at A and the outflow at B is forced. And nothing constrains these flows to just be horizontal. Air can flow downwards from above to fill A as well for example. And air can move upwards at B to get out of the way. With a 3 dimensional atmosphere flows will be 3 dimensional as well, not just constrained to 2 dimensions. Direct convection at the site of the movement isn’t needed if there is some driver elsewhere that can initiate air movements. Daytime vs Nighttime insolation changes for example. Throw in coriolis force and you have a recipe for complex air movements. Which then will be in 3 dimensions.

          And all the Adiabatic Lapse rate requires is vertical mixing. It doesn’t require localized heating leading to convection, so long as there is a non–local driver. Local convection will act as well but it isn’t required, it is just additional. And the magnitude of the Lapse Rate is determined solely by the pressure differential. Essentially by gravity producing a vertical pressure profile in the atmosphere.

          And the troposphere as a whole doesn’t eed to be adiabatic – obviously it isn’t. But the Adiabatic Lapse rate is driven by the expansion of rising and the compression of falling parcels of air. Each parcel can be adiabatic even though the tropospere isn’t. Localised thermal equilibrium can exist even when it doesn’t occur globally.

          Obviously this is ignoring the saturated lapse rate associated with evaporation and condensation of H2O which has to be added in to get the real world lapse rate but the adiabatic part doesn’t need convection locally t odrive it.

          1. I agree with Glenn. With GHG there is some interaction of the temperature and heat flux, because absorption.re-emission is a form of enhanced conduction.. But without GHG, it’s purely gas movement. The radiative flux passes through without interaction.

          2. Imagine an isothermal atmosphere without GHG and add a single heat point source on the ground. The state of the column is hyper-stable, a hot air bubble will appear near surface, the pressure will increase, the TOA rise locally and hot air expelled laterally without causing convection in highly stable upper layers. There will be vertical movements only in the thickness of the hot air bubble elsewhere offsets will be by lateral transfers.

            A second point, our troposphere is not an adiabatic medium because it contains GHG and is the target and the source of radiative fluxes.

          3. Glen and Nick,

            I am having difficulty with the Loschmit statement in that I can see no reason why there shouldn’t be a temperature differential in an atmosphere with no net energy transfer between layers. It all seems perfectly consistent with thermodynamics to me except for the statement that heat always flows from hot to cold. It seems that this statement is too simplistic and that energy transfer in a gravitational field can full well follow a gradient. We all accept the relativistic gradient expected in gravity with no net energy flow yet the linear velocity gradient drives some of people wild because it causes unmached temperatures in two equal height columns of gas (or solids) that have the same base temperature. That doesn’t bother me though because it is not a perpetual motion machine or free energy as any energy transfer simply turns into heating or cooling of the entire column. I can think of no simplistic examples which disprove the theory.

            I’ve read comments from DeWitt, Carrick, Steve F and others on the subject written a long time ago but didn’t get involved, now I’m still having problems figuring out what is wrong with the concept.

            Oh and Glen is right in that any vertical movement of the gas, thermal or otherwise should result in the lapse rate. It is well written. I wonder why that is any different than what Loschmidt has written except that it came from a different derivation.

          4. I realized an example where Lochmidts theory would fail in that a perpetual motion situation occurs. It is from DeWitts two perfectly insulated vertical cylinders having different Cp gasses. If the gasses stabilized at a different gradient and each end of the insulated cylinder was joined with a thin thermally conductive membrane such that heat could travel between the cylinders at the end, you have a perpetual motion machine!

            It doesn’t work. I think the same is true if gas diffusion created the adiabatic lapse rate. It couldn’t work by diffusion alone. My conclusion is that the gas would have to be transported in packets from an outside source to maintain the gradient. It isn’t very satisfying but it must work that way to avoid getting free energy.

          5. Phi

            The atmosphere as a whole doesn’t have to be adiabatic. What matters is that individual parcels of air can rise and fall adiabatically or very nearly so. With a large air mass the air near the center is certainly in a near adiabatic state.

            “Imagine an isothermal atmosphere without GHG and add a single heat point source on the ground. The state of the column is hyper-stable, a hot air bubble will appear near surface, the pressure will increase, the TOA rise locally and hot air expelled laterally without causing convection in highly stable upper layers. There will be vertical movements only in the thickness of the hot air bubble elsewhere offsets will be by lateral transfers.”

            This isothermal atmospheric column isn’t stable. In the event that you could artificially create one then let it progress naturally it would breakdown. Take your example of this isothermal column and your heat source at the base. Your heat source heats air near the base. Its pressure increases without any increase in density since the pressure is increased motion of the same number of molecules. So this parcel of air is now at a higher pressure than the surrounding air. Being at a higher pressure the parcel will expand, pushing the surrounding air aside and its pressure will drop. And its density. Now the heated parcel of air has a lower density than the surrounding air. So it will rise. As it rises it keeps expanding further and rising further. Each iteration of expansion involves our parcel of air doing work on the surrounding air. So it is losing heat because it is doing work. Eventually it reaches a height where it has lost so much energy due to work done that it is in energy balance with the surrounding air parcels and stops rising. And it has cooled.

            Conservation of Mass then requires that the air that has moved up must be replaced. And since its movement is vertical, there must be a corresponding downwards movement somewhere to counter balance this. And this descending parcel of air experiences the reverse. As it descends surrounding air at higher pressure does work on it, adding energy to it and raising it temperature.

            The trap when thinking about adiabatic processes is that adiabatic refers to energy transfers in/out of the process as heat. Energy can also be transferred in/out as work. So a parcel of air can change/have changed its total energy content without needing a heat flow.

          6. Glenn,
            As I said, I agree with your contention that air movements sustain the lapse rate. It’s something I’ve blogged about over the years. I think now that there is quite a lot that can be quantified, and I’ve written a new post here.

          7. Nick Stokes,
            You are in complete fog. Vertical motions in atmosphere are the result and not the cause of temperature differential !

            To what strange physics climatology can lead !

          8. Nick Stokes,
            I have to add something in your favor: your approach is entirely logical.

            Having accepted the premise of the neophlogistics promoted by IPCC, you could not come to any other conclusion.

            It is stupid, but so is neophlogistics.

        2. “I don’t agree. The gradient arises solely due to vertical air movements”

          No it doesn’t. There’s no such thing in the Uranus troposphere and no surface at its base, nor solar radiation.

          The gradient evolves as molecules move between collisions, even in tall sealed insulated cylinders of gas.

          And if I’m wrong, then so is the Second Law of Thermodynamics because the gradient is the state of thermodynamic equilibrium with no unbalanced energy potentials. If you think there are, then explain why. Otherwise stop promulgating false “fissics.”

          .

          1. Doug

            Where ever you have a 3 dimensional fluid there are movements in all 3 dimensions. Nothing is ever perfectly 2 dimensional and as soon as any sort of turbulence arises then 3 D movement. And that is all you need at produce a gradient.

          2. Doug

            The ‘energy potentials’ ARE balanced. Rising air is transferring energy upwards and falling air is transferring air downwards. Don’t just focus on the 2nd Law. You have to remember what the effects of Conservation of Mass implies as well.

      2. Phi, I think we agree on all of this. I suppose the flux in these planets is created by solar incoming radiation. This strikes the ground of the planets and warms the air. If we assume the planets are revolving then warm air masses will collect and rise cooling in generally an adiabatic fashion. having no water on the planets, I wonder just how far off of a dry adiabatic lapse rate the planetary atmospheres would be. If Loschmidt is right, they would be exactly on. Interesting to me is that Loschmidt’s hypothesis contradicts that gradient and heat flow are inseparable. The idea is no heatflow if the gradient is in the same G/Cp magnitude predicted. I find the concept interesting in that while I get the velocity drop with altitude, much of atmospheric heat is spin/vibration based and why should that drop at the same rate? Same problem in a solid. Anyway, this is well beyond the conversation I’ve been having with Doug and I don’t want to mess that up until we resolve his issue.

        1. Yes, well there’s no surface at the base of the nominal troposphere of Uranus. All the solar radiation is absorbed 350Km further up, so there’s no direct solar radiation at that base either. Your problem is that there is a gravitationally induced temperature gradient which is 95% of the “dry” value based on -g/Cp and there absolutely must be non radiative heat flow from the 60K TOA to the base of the troposphere which is 320K – hotter than Earth but 30 times further from the Sun. I am one of only two researchers in the world who (to my knowledge) has published a valid explanation for downward non-radiative heat transfer up the temperature gradient from cooler to warmer regions, yet very strictly acting in accord with the process described in statements of the Second Law of Thermodynamics. So if you can explain the downward heat flow in any other way, and also produce a study along the lines of that in my book, but instead proving water vapour warms to the extent the IPCC implies, rather than cools as I showed, then you or any reader here could be the first to do so and thus win the $5,000 reward that no-one has in nearly 8 months since my book as published.

        2. Jeff asks “why should that drop at the same rate” Because of the Equipartition Theorem. The equal sharing of kinetic energy among the degrees of freedom takes place during molecular collisions. In free flight between collisions gravitational potential energy exchanges with translational kinetic energy. This is where Roderich Graeff got his physics wrong (having admitted he had no formal education in such) and he multiplied the g/Cp gradient by the degrees of freedom. Never-the-less his 800 or more experiments did show clear evidence of a temperature gradient in nearly every one.

          You still don’t seem to understand that the gradient is the state of thermodynamic equilibrium. I’m sure you can understand that a density gradient develops. That also is a direct consequence of the process described in the Second Law, and it is one and the same state of thermodynamic equilibrium.

          THE DENSITY GRADIENT AND THE TEMPERATURE GRADIENT ARE BOTH THE SAME STATE OF THERMODYNAMIC EQUILIBRIUM AND THAT IS THE MOST IMPORTANT THING IN THE WHOLE OF THE CLIMATE DEBATE THAT YOU NEED TO UNDERSTAND, ALONG WITH ITS CONSEQUENCES. DOES RAIN FALL IN THE MIDDLE OF A LAKE AND THEN ONLY DISPERSE TO THE NORTH? THINK!

          1. I’m not going to reply to your future arguments until you sort the basic arguments. There is no point in having a complex discussion when the simplistic cannot be resolved.

          2. I have sorted the basics. You forgot that I explained (supported by empirical evidence) that inter-molecular radiation between carbon dioxide molecules reduces the temperature gradient, and thus the surface temperature.

          3. Strictly speaking you need more information about solar intensity and the height of the atmosphere. I’ve taken these to be as on Earth and thus pointed out that, in a nitrogen atmosphere, it is possible for direct solar radiation to raise the surface temperature to existing levels. That’s not possible on Venus or at the base of the nominal troposphere of Uranus. Nor is it possible on the real Earth with the reduction in solar radiation due to reflection and absorption of about half that incident solar radiation.

            On planets where the solar radiation reaching the surface is not sufficient to support the surface temperature, then the gravitationally-induced temperature gradient takes over and the overall level of the temperature plot is determined by radiative balance. The surface temperature will be warmer than that temperature somewhere in the troposphere because of non-radiative transfer of heat up the thermal plane from where it was absorbed from incident solar radiation at various altitudes in the troposphere. You get no useful information from your thought experiment unless you know whether direct solar radiation is determining the surface temperature (as on Mercury and our Moon) or the gravito-thermal effect.

          4. PS: And (as I also explained) on the real Earth the increase in mean emissivity (due to water and water vapour compared with a rocky planet) also reduces the mean temperature at which the Earth radiates, even though the theoretical radiative temperature (based on assumed emissivity of 1.000) remains the same. So Earth is not in the category of rare planets without significant radiating atmospheres where direct solar radiation determines the surface temperature.

            Obviously one molecule of carbon dioxide in 2,500 air molecules can have very little effect in the real Earth. I stick by what I said about CO2 cooling by 0.1 degree, but even if it warmed by a similar small amount, the cooling effect of water vapour would completely override that (especially if WV increases as CO2 increases giving negative feedback) and, at the same time, that cooling effect smashes the greenhouse conjecture.

          5. So, if you just suddenly change a planet from one in which solar radiation is determining the surface temperature (as with an Earth with a pure nitrogen atmosphere and a lower mean emissivity because of no water or water vapour) to a planet where the gravito-thermal effect is determining temperature and the atmosphere is radiating, then whether or not you get a higher surface temperature depends somewhat on the height of the atmosphere. So, just because you do on Venus, does not mean you do on Earth because the initial surface temperature due to radiation could well have been higher than that supported by the gravito-thermal effect.

            There is too much unknown information to be definitive about the end result, and, above all, it is a complete red herring which gives no information as to the effect of marginal variations in carbon dioxide levels in the real watery Earth.

          6. Wow. You have stated that the emission temp is the same, you have stated that the emission temp is on average from a higher than ground altitude on the CO2 planet, you have stated there is a gradient.

            Duck is cooked.

          7. Because the temperature gradient on Earth is determined firstly by non-radiative processes and is then reduced a little by inter-molecular radiation, all the air molecules play a part in the first process and carbon dioxide is only 1 in 2,500 of those air molecules. Then the radiating molecules (mostly water vapour which varies between about 1 in 100 to 1 in 25 and completely dominates carbon dioxide in both numbers and radiating bands) reduce the gradient depending on their numbers but only up to a reduction of about 35%. Ocean water and water vapour also reduce the mean radiating temperature for the whole temperature profile, so they cool in two ways.

          8. Doug, stop being a (what is a thing which intentionally pretends stupidity). We aren’t talking about Earth and you know it.

            We have a known temperature, an increasing slope with altitude from that temperature, and and increasing altitude with GHG. Warmer temps.

            You are self-contradictory and your theory has failed.

          9. [reply: I’m going to edit inline this time. – Jeff]

            Go back and actually read and study carefully what I have written, Jeff, noting especially all the comments from 7.10pm. [I read them] What the hell do you imagine your thought experiment implies for the real Earth which is so different in so many ways? [REPLY: I’m still trying to figure out if you have a theory, what it implies is something to be figured out afterward] Water and water vapour lower Earth’s actual radiating temperature compared with a rocky planet. [don’t care] They also lower the temperature gradient.[don’t care] Both processes have a cooling effect, so water vapour cools in the first place and has negative feedback as well. [don’t care] That means all the greenhouse conjecture must be wrong.[does not]

            Water gets less dense as you cool it below 4C.[don’t care] It also gets less dense as you warm it above 4C. What carbon dioxide does also depends on numerous other factors.[don’t care] Get real, man! [kay] Get onto talking about the real Earth where it is the gravitationally induced temperature gradient that determines surface temperatures, not direct solar radiation to the surface, let alone back radiation. [you have shown yourself unable to handle the most simplistic discussions of math or science, there is no purpose for talking about anything more complex] Show me your calculations for the surface temperatures of Earth and Venus![no need] Show me your empirical evidence for water vapour warming rather than cooling as it does.[don’t care] I have produced evidence. [No you havent! You have produced conclusions. Nobody can understand your foundation, including you as you are self-contradictory.] You have produced nothing but irrelevant thought experiments. [You have yet to answer the thought experiment or state in clear rational terms what would make it irrelevant. hell, you won’t even tell me where you disagree or agree. You only state conclusion. A conclusion which is self contradictory to your own writing.]

          10. In the real world if you add more water vapour more of it congregates below the mean radiating altitude than above, and so it lowers that altitude. It also lowers the mean radiating temperature because there is more radiation from more high emissivity molecules. It also lowers the temperature gradient (as is well known) by inter-molecular radiation (as is not well known) far more so than by latent heat release. Increasing carbon dioxide also lowers the temperature gradient by radiation more than its specific heat raises it, but its overall effect is minuscule. If the IPCC are correct about increasing CO2 causing increases in water vapour, then CO2 most certainly cools because of the overwhelming negative feedback from even a small increase in water vapour.

          11. Doug

            What you aren’t getting is that the GH effect is CAUSED by the combination of the radiative properties of the GH gases and the presence of a lapse rate. The Dry Adiabatic Lapse Rate is simply calculated and you are right that is is gravity related. The formula is simply

            Dry ALR = g/cp

            where g is the local gravity and cp is the specific gravity of the atmosphere.

            GH Gases determine the effective radiation to space altitude and thus force the temperature at that altitude. The Lapse Rate then sets the temperature of every other height in the air column relative to the temperature at that altitude.

            Imagine hypothetical planet that had a reversed lapse rate – air temperature get hotter as we go higher. This is physically impossible, but is a useful thought experiment. What would the GH effect do on such a planet? It would make the surface colder than above. And adding more GH gases would have a cooling effect on surface temperatures.

          12. in a nitrogen filled rocky Earth atmosphere the surface temperature could well be 300K as I have explained. Replace that atmosphere with 100% carbon dioxide and the mean radiating temperature drops because virtually all the incident solar radiation is absorbed and re-emitted in the upper atmosphere as on Venus, so the planet has higher emissivity. Sure the mean radiating altitude certainly rises, but we don’t know the net effect after the temperature gradient reduces and the mean radiating temperature reduces, and so the surface temperature could well drop to 295K for example, and you can’t prove otherwise. After all, the reduction in the gradient is similar on Earth and Venus. There is far less direct solar radiation getting to the surface, so if you think you can calculate the surface temperature from that you’re certainly out in the cold.

          13. You’re discussing a totally different paradigm, Glenn. The gravitationally induced temperature gradient is the state of thermodynamic equilibrium and it determines the supported surface temperature, not radiation. Think of Venus or Uranus. You don’t have to feed me the IPCC garbage which I’ve studied probably more than yourself and picked to pieces with the valid physics in my book.

          14. Glenn The key claim of the IPCC assumes water vapour is the main GHG doing most of their “33 degrees” of warming. I have shown from real world temperature data in my published study that water vapour cools. You can check the study and the data – it’s all in the Appendix of my book.

            The reasons an increase in water vapour in the real world cools are that (a) it doesn’t raise the mean radiating altitude, because more of it congregates below that altitude than above (b) it reduces the temperature gradient so the thermal plot rotates downwards at the surface end (c) it increase the mean emissivity and so the actual mean radiating temperature (not the theoretical one) is lowered and thus the whole thermal plot is lowered.

            Jeff has overlooked some of these facts in his over-simplified and irrelevant thought experiment.

            It’s time to move on, Jeff. What would you deduce if your carbon dioxide atmosphere produced a slightly warmer surface temperature anyway – say 310K rather than 300K for the nitrogen one? I suggest any deduction pertaining to a watery world where there already is a mean radiating altitude up in the troposphere would be quite irrelevant. I’m not agreeing with 310K, and you can’t prove it either.

          15. Doug, you have finally made a point. I’m sick of it, but since you finally made a point, I should probably reply to it.

            Surprisingly, you have demonstrated that you understand the conundrum you have created for yourself because you have now lowered the emission temperature AND the gradient. Unsurprisingly, you didn’t recognize that the change in gradient won’t help you because the N2 atmosphere emits from zero altitude. Also a planned part of the problem to keep our discussion clean.

            Of course by normal physics, you have unbalanced the energy of the CO2 planet such that it is collecting energy and warming up. So that gives me a new question, and a very reasonable question for someone like myself trying to understand your theory.

            Energy in must equal energy out as per the assumption of our planets. How does your planet have a different average emission temperature yet stay in radiative balance?

          16. Doug

            ” I have shown from real world temperature data in my published study that water vapour cools”

            Of course it does Doug. As I pointed out in other comments, the evaporation/condensation of water has a major influence on the lapse rate, making the real world lapse rate be 1/3rd less than it would be without water. So surface temperatures aren’t as hot as they would otherwise be.

            Now consider that water can both cool AND warm. Because there are two different mechanisms involved. The phase change properties of water have a cooling effect. And the radiative properties of water have a warming effect,

            Similarly clouds can both cool and warm. Clouds reflect sunlight by one mechanism – this is a cooling effect. Clouds contribute to the GH effect, this is a warming effect. Low level clouds have a net cooling effect because they reflect much more sunlight and don’t contribute as much to the GH effect since the lower atmosphere is warmer. High level clouds have a net warming effect since they are thinner so reflect less sunlight but they are in much higher colder air so their contribution to the GH effect is greater.

            Aerosols can warm or cool. They absorb infrared that strikes them and so are a part of the GH Effect – a warming effect. And they also reflect sunlight – a cooling effect.

            I think you are falling into a trap of only looking at one aspect of what something can do rather than all of its effects.

          17. Doug

            “You’re discussing a totally different paradigm, Glenn. The gravitationally induced temperature gradient is the state of thermodynamic equilibrium and it determines the supported surface temperature, not radiation. Think of Venus or Uranus. You don’t have to feed me the IPCC garbage which I’ve studied probably more than yourself and picked to pieces with the valid physics in my book.”

            Putting aside from the serious streak of arrogance in your tone Doug you might like to think of something. What you are calling ‘the state of thermodynamic equilibrium’ and I am calling the Lapse Rate is indeed gravitationally induced. Gravity is how the vertical pressure profile that creates the Lapse Rate is created. But this only determines the RELATIVE temperatures. That altitude x is y degrees different from altitude z.

            But it does not determine the absolute value of the temperatures. Something has to establish the baseline temperature at some altitude and then the lapse rate determines the temperatures at other altitudes as relative temperatures from that.

            Something has to anchor the entire temperature profile.
            And that something determines that the surface is at 15 C and at 5 km up is at -18 C rather than the surface is at 5 C and at 5 km up it is at -28 C. Saying something is the thermal equilibrium doesn’t say WHAT the equilibrium is.

            It is the radiative balance that determines what value is the equilibrium.

            Think of it like this Doug. Temperature can be described by a simple equation vs height

            T = L*H +b

            T is temperature
            L is Lapse Rate
            H is height.

            This equation describes a line with a slope of L. In fact it describes an infinite family of lines with slope L. In order to describe a single line we need to know the value of b. And b isn’t determined by lapse rate, gravito-thermal effects or any such thing. All those things can do is determine L.

            What determines b?

    2. Phil,
      The variation in insolation from the Sun at different solar inclinations (latitudes), the day night cycle, and the rotation of the planet, all cause atmospheric mixing. In addition, the surface solar absorption induced heating causes buoyant plumes as well as radiation up for both cases. In the case of the CO2 atmosphere, the atmospheric absorption of thermal radiation over some of the radiation wavelengths (and radiation from the atmosphere) results in additional energy transfer that is not directly radiated to space from the surface. However, the net transport of some of the absorbed solar energy by winds from lower latitudes to higher latitudes would result in a somewhat different surface temperature distribution than a planet with no atmosphere, and due to the non-linear radiation laws, could result in a different average temperature over the globe than the case with a pure Nitrogen atmosphere. There would definitely be enough mixing to maintain an average adiabatic lapse rate (pure conduction is a far slower heat transfer process than even modest convection, so it does not require much to mix enough to maintain adiabatic average lapse rate). It is clear that the CO2 atmosphere would radiate to space at a significant altitude, and the presence of the lapse rate would assure heating from the so called atmospheric greenhouse effect. The case with just Nitrogen would also have the adiabatic lapse rate, but essentially all of the radiation to space would occur from the surface, so the temperature would not be elevated from this cause. However, keep in mind the use of an average temperature is not an accurate basis for comparison for any case due to the non-linear radiation effects and redistribution of absorbed energy, but is a reasonable one for a comparison as described.

      1. Leonard Weinstein,

        “There would definitely be enough mixing to maintain an average adiabatic lapse rate…”
        “The case with just Nitrogen would also have the adiabatic lapse rate…”

        An atmosphere without vertical heat flow is isothermal and stratified. If an air mass is for one reason or another forced upward, it will be quickly damped through its adiabatic cooling.

        1. No it’s not isothermal, and you can’t prove it is. In an isothermal state there would be unbalanced energy potentials due to the additional gravitational potential energy at higher altitudes not being offset by a reduction in mean kinetic energy per molecule, that is, a reduction in temperature. So, if there were isothermal conditions, the process described in statements of the Second Law of Thermodynamics will ensure that entropy increases until the state of thermodynamic equilibrium is attained, that state then having a density gradient and a temperature gradient. Of course a pressure gradient is thus a corollary because pressure is proportional to the product of density and temperature. It seems you need to learn some thermodynamics my friend. You’ll learn such from my book “Why It’s Not Carbon Dioxide After All” on Amazon.

        2. Phi

          But don’t forget to consider conservation of mass. If a mass of air rises, a compensating mass of air will fall, being adiabatically heated. Lenard is right. We can’t just think about heat, we need to apply all the laws of thermodynamics and conservation of mass is critical.

          1. Glenn,
            In an isothermal and stratified atmosphere, vertical movements are effectively prevented. This is a common meteorological situation. Small oscillations are rapidly damped. All known observations confirm that there is no thermal gradient without thermal flux.

  3.  
     

    These two statements by Jeff are incorrect for the reasons documented in sufficient detail below. I make no apology for lengthy explanations as I consider it necessary to support my reasons with examples of empirical evidence, not just thought experiments, so silent readers should be disappointed if my explanation gets truncated. It is necessary to discuss the radiating greenhouse gas water vapour because that is the only one for which we can actually measure sensitivity. We can deduce from valid physics that other radiating molecules will act in a similar way, and thus we can discuss carbon dioxide in the context of supporting empirical evidence for water vapour..

    (1) “They experience the same solar input and all other features are the same.”

    (2) ” then the ground temperature of the CO2 planet must be warmer than the N2″

    Reasons:

    1: All other “features” are not the same. A planet’s surface receives considerably less direct solar radiation if there are absorbing molecules in that atmosphere. If there is water vapour then there are also clouds reflecting solar energy back to space. On Venus the atmosphere is nearly all CO2 and nearly all the incident solar radiation is absorbed on the way in. Guess where its energy ends up – nearly all of it in carbon dioxide molecules before it can get to the surface. The solar radiation reaching the surface is about 10% of what Earth’s surface receives, yet the temperature rises by about 5 degrees from 732K to 737K while the Sun shines for 4 months. If radiation were doing that, then there would have to be an input flux of about 14,000 to 16,000W/m^2 which is far more than the Sun delivers to the top of the Venus atmosphere. And if radiation were raising the temperature it would do it in a day or so and not take 4 months.

    2: No, the radiating altitude is a fiction, because radiation emanates not from a single altitude but from the surface and all altitudes. Water vapour, for example, lowers the mean altitude of radiation because most of it exists below 5Km. But the main point is that radiating molecules reduce the magnitude of the temperature gradient (as is well known for water vapour) so the temperature profile rotates and this leads to lower supported temperatures at the surface. Radiating water molecules reduce the insulating effect between double glazed window panes for the same reason. Jeff could have leaned all this from my book “Why It’s Not Carbon Dioxide After All.” What I say is supported by empirical data for water vapour, but the very slight cooling effect of carbon dioxide is of the order of 0.1 degree and not measurable.

    In effect Jeff is saying that radiating molecules like water vapour and carbon dioxide cause higher planetary surface temperatures. They don’t and he can’t prove from valid thermodynamics that they do, and nor can he present empirical evidence to support his contention. He presents no empirical evidence, for example, that more moist regions are considerably hotter than regions with only a quarter as much water vapour above them. In contrast, I present a sound and comprehensive study using 30 years of temperature data from three continents and proving the opposite, namely that water vapour cools as is to be expected from my hypothesis.

    I have also proved what I say about the carbon dioxide atmosphere, based on comparative actual measurements by probes dropped onto Venus and good estimates from spacecraft passing close to Uranus. The Venus temperature gradient is reduced in magnitude by at least 20% to 25% by all the inter-molecular radiation between carbon dioxide molecules. In contrast, with barely 2% of radiating molecules in the Uranus atmosphere (mostly methane) the gradient is still about 95% of the expected “dry” gravitationally-induced temperature gradient based on the quotient of the acceleration due to gravity and the weighted mean specific heat of the gases.

    So the more radiating molecules there are, the greater is the percentage reduction in the magnitude of the temperature gradient, and thus the lower is the supported surface temperature.

     

  4. My three comments above have been duplicated in full on Roy Spencer’s thread here and that’s pretty much all I need to say in response to this post. Anything else that Jeff may raise is probably refuted already in my book or my paper cited therein..

     

  5. Footnote:

    Jeff said “the emission temperatures of both are the same yet the average emission altitude of the CO2 planet is non-zero, then the ground temperature of the CO2 planet must be warmer than the N2 “

    The emission temperature of a planet is merely the mean temperature the planet and its atmosphere would be if its emissivity were 1.0. it is not necessarily the surface temperature. There can be the same radiative flux coming from a hotter surface that has lower emissivity. The emissivity of ocean water and water vapour (which do most of the radiating) is around 0.99. But a planet without water and water vapour has a somewhat lower emissivity and hence a higher surface temperature corresponding with the flux that relates to the theoretical radiating temperature. I repeat a comment on the other thread regarding this point…

    Regarding Jeff’s and the IPCC’s assumptions about the surface temperature in a GHG free atmosphere, there is no way you can prove that the surface temperature would be less than the existing 287.5K mean and thus prove water vapour or CO2 warms. You and the IPCC have forgotten that the flux you have assumed took account of the fact that about 30% of solar radiation is reflected by clouds, but there would be no clouds. So in fact something like 315W/m^2 would be the actual mean flux to the surface. Calculating the temperature correctly is no mean feat, as you should integrate over the whole globe using the T^4 relationship in the Stefan-Boltzmann equation. If you just assume that there is homogeneous solar flux on a flat Earth 24 hours a day then you get 287.5K from 315W/m^2 if you use emissivity 0.813 which may not be all that unrealistic. For example, this reference gives some emissivity values – note Basalt = 0.72, lime clay= 0.43, shale = 0.69, granite = 0.45, gravel = 0.28 fine snow = 0.82, soil = 0.38. I venture to suggest the surface temperature could indeed be 300K as I have been suggesting it would be before water vapour cools it. Until you determine the proportion of such materials on the Earth and do the full integration with a sophisticated multi-million dollar model that uses the actual mapped surface taking into account the latitude of the various materials etc etc you have absolutely no grounds for assuming water vapour (or any GHG) warms the surface You don’t even know for sure that a minute portion of the molecules in any gas in the periodic table would not be able to absorb some solar radiation and then warm adjacent molecules by non-radiative processes. In contrast, at least in the real world, I have produced a study showing water vapour cools. The full methodology and the data and source thereof are all in my book, so you or anyone can check such and/or do your own study trying to prove the opposite. Do you ever wonder why the IPCC has not published such a study? I suggest that whenever they try to do such they find water vapour cools and so they censor the study. But tell me if you can find one like mine but showing the opposite.

    I rest my case.
     

    1. Doug

      “if you use emissivity 0.813 which may not be all that unrealistic. For example, this reference gives some emissivity values – note Basalt = 0.72, lime clay= 0.43, shale = 0.69, granite = 0.45, gravel = 0.28 fine snow = 0.82, soil = 0.38. I venture to suggest the surface temperature could indeed be 300K”

      Doug you are quoting single emissivity values for different substances as if emissivity for any particular material is a constant value over all wavelengths. This isn’t true.

      Around the visible wavelengths absorptivity (and thus emissivity) varies hugely, from 0.9 or so for water to 0.2 or so for fresh snow.

      In the far infrared we are dealing with virtually all substances have emissivities greater than 0.95, often 0.97 to 0.98.

      So when considering the energy balance one need to take account of this. One approach is to factor the Earth’s total albedo out of the the incoming side of the equation to give a net energy absorbed by both the surface and the atmosphere – if you do just absorption by the surface your results will be wrong. Then use 0.98 as the emissivity of the emissions from the surface.

      So if incoming solar is 1365 w/m^2, divide by 4 to allow for the ratio of frontal area to surface area and then multiply by 0.7 to factor out albedo total solar absorbed is 239 w/m^2

      To radiate this divide by the SB constant, divide by 0.98 then take the 4th root and we get 256 K. Repeat this calc for emiissivity of 1.0 not 0.98 and we get 254.8. Not much difference.

      A planet at the Earth’s distance from the Sun, with an albedo (from whatever cause) of 0.3, must radiate at an equivalent temperature of around 255K.

  6.  

    So you have no proof of your thought experiment Jeff. An Earth without water and water vapour, for example, radiates mostly from a surface with lower emissivity, which must thus be at a higher temperature in order to deliver the radiative flux that corresponds with the planet’s radiating temperature (which is based on 1.000 emissivity) and, furthermore, it’s surface receives about 315W/m^2 of direct solar radiation rather than only 161W/m^2 after absorption mostly by water vapour and reflection mostly by clouds and ocean, snow and ice surfaces.

    Empirical evidence shows water vapour cools, Jeff, just as my hypothesis says it should, and quite the opposite of what the IPCC would like you to be gullible enough to believe.

    QED

    1. “Empirical evidence shows water vapour cools”

      Yep, it sure does Doug. Otherwise the GH Effect would be even stronger. See my other posts.

      Dry adiabatic Lapse Rate alone is 9.8 C/km So with an average radiating altitude of around 5 kilometers without water vapour’s cooling effect the GH effect would add around 49 C to the temperature. Instead, because the wet adiabatic or saturated lapse rate applies as well in part reducing the actual environmental lapse rate to 6.5 C/km and so around 33 C GH Effect.

      So water vapour does cool, reducing the magnitude of the GH effect by about 1/3rd.

      And there is a feedback mechanism that applies as the atmosphere warms, altering the lapse rate in part due to changes in water vapour content, although the size of thhis feedback, even its sign isn’t clear.

      There is a discussion here.
      http://www.astr.ucl.ac.be/textbook/chapter4_node7.html

  7. I’m not sure where to reply doug, there is a large pile here. Let’s start with this:

    (1) “They experience the same solar input and all other features are the same.”

    This is the assumption for the planets we are discussing. Now I realize that such a simplified assumption makes it hard to escape the conclusion you are avoiding, that is the basis of this conversation. If you say yes CO2 causes warming in this case but not in some other case, we can discuss that but you cannot write that the assumption is in error because it is the assumption.

    In this case the assumption is that neither planet absorbs or reflects more light to an outside observer, there is no water, so you cannot argue from that direction. You can stop rewriting different albedo’s for rock as that is not part of the question. It seems to me that you are pretending ignorance to avoid the problem.

    (2) ” then the ground temperature of the CO2 planet must be warmer than the N2″

    You answer — ” No, the radiating altitude is a fiction, because radiation emanates not from a single altitude but from the surface and all altitudes.”

    But that is the problem, I wrote “average emission altitude” to keep you from attempting this exact exit as it is complete sophistry. IF the AVERAGE emission altitude increases at all (even an inch) from N2 to a CO2 atmosphere, and your gradient is non-zero, meaning anything more than zero, then the surface temperature will be warmer in the CO2 planet.

    So you see, you cannot argue about the color of rock, or the reflectivity of water, 285C, or that it is covered somewhere in your book or that I am confused. None of that is part of the assumptions. It doesn’t matter, all you can argue is either why Jeff is correct in THIS case, why Jeff is correct in THIS case but it doesn’t apply to Earth for X reason, or that Jeff is wrong in THIS case for X reason.

    1.  
      Jeff

      The underlying assumption is surely that your two imaginary planets (one with N2 and one with CO2) receive the same flux of solar radiation at the very top of their exospheres (before albedo reductions) or, in other words, that they are the same distance from the same Sun.

      The fact that rock and most substances on a solid planet’s surface (without water) have emissivity somewhat less than 1.00 (and that of water) means that an assumed mean emissivity of 0.83 is not unreasonable. Yes clouds, water, ice and snow also reflect more, so both these points are relevant regarding surface temperature.

      So, the reduction in mean emissivity and the reduced albedo and reflection by the surface on a rocky planet could fully explain a temperature equivalent to Earth’s existing surface temperature without any need for any assumed carbon dioxide warming or any need for additional thermal energy to be delivered into to surface from a colder atmosphere, supposedly helping the Sun to raise the surface temperature each morning, which (as I explained to you Jeff in 2011 and in my March 2012 paper) would be a violation of the Second Law which applies to every independent process anywhere, including one-way radiation. In any event, an atmosphere cannot deliver more thermal energy out of its base than it received at the top, as is implicit in the K-T, NASA and IPCC energy diagrams which imply radiation always transfers thermal energy wherever it strikes something, which it doesn’t always do.

      Now, Jeff, you have an example of a carbon dioxide atmosphere on Venus, and that carbon dioxide ends up holding over 97% of the energy delivered by incident solar radiation that’s not reflected. So this energy is in the carbon dioxide molecules rather than the surface. In other words, the solar radiation reaching the surface of a planet with a carbon dioxide atmosphere is considerably reduced by the time it reaches the surface. (Carbon dioxide absorbs solar radiation mostly in the 2.1 micron band.) Once again I have empirical evidence on my side in that Russian probes dropped to the Venus surface made measurements from which they deduced that the mean solar flux there was between a mere 10 and 20W/m^2 and that was for only the sunlit side.

      You wrote more garbage when you said: “IF the AVERAGE emission altitude increases at all (even an inch) from N2 to a CO2 atmosphere, and your gradient is non-zero, meaning anything more than zero, then the surface temperature will be warmer in the CO2 planet.” It seems you need a geometry lesson, Jeff, because, as I have explained numerous times, radiating molecules radiate between themselves and reduced the temperature gradient. Once again, empirical evidence (which I have given in previous comments) proves that the gravitationally induced temperature gradient is reduced by the temperature-levelling effect of radiation. So the temperature plot drops a little at the surface and its gradient is reduced in magnitude. Sorry, Jeff, that’s not a warming effect at the surface.

      Radiating molecules reduce the magnitude of the temperature gradient (aka “lapse rate”) …

      (1) by about 35% on Earth (mostly by water vapour and a very little by carbon dioxide etc) as is well known

      (2) by about 20% to 25% (by my calculations) on Venus by carbon dioxide

      (3) By about 5% (by my calculations) on Uranus by a sprinkling of methane

      Now the fact is that we live on a planet with water vapour, and that’s the planet we are concerned about. If we find (as I did) from empirical evidence that water vapour cools by about 4 degrees for each 1% in the atmosphere rather than warms by more than 10 degrees for each 1% (as the IPCC implies) then the whole greenhouse conjecture is smashed as there is no reason to assume carbon dioxide would do the opposite of water, and even if it did, there would then be a larger negative feedback from additional water vapour if the IPCC are right about CO2 increasing WV..

      Water vapour leads to lower supported surface temperatures because ….

      (a) It reduces the magnitude of the gravitationally induced temperature gradient (just like carbon dioxide and methane do) and that causes the temperature profile to rotate downwards at the surface end, lowering the temperature supported by the base of the troposphere which is due to that gravitationally-induced temperature gradient.

      (2) It increases the albedo through cloud formation as well as reflection from water, snow and ice surfaces

      (3) It increases the mean emissivity of the Earth+atmosphere system, thus allowing the Earth to have radiative equilibrium with the Sun at a lower temperature than would a rocky Earth.

      In the interests of science, even though it may bring down Jeff Conlon and his “Air Vent” (and hopefully one day the IPCC) this comment is being posted on several large climate blogs.

      If anyone wishes to debate or enquire about my hypothesis (which is a whole new paradigm) there’s my book “Why It’s Not Carbon Dioxide After All” on Amazon to read first.

      1. Doug, you are rambling incomprehensibly.

        If the mean emission temperature of both planets are the same and the average emission altitude of the CO2 planet is higher, then the only way that the CO2 planet cannot warm is if there is no gradient between the emission altitude and ground. Since you agree there is a gradient, then there is a warmer surface. Case closed, cooked duck as they say.

        I had rather hoped you had considered your argument more carefully since the days of the non-working bolometer discussion, but unless your next comment gives some light on the obvious self contradiction you have created, and it is OBVIOUS to all other readers, I’m going to have to give up on you.

        Oh, and the name is Condon, hopefully you spell it right in your copy-paste spamming routine.

      2. Doug

        On your first point. How on earth does the fact that the molecules are radiating (and lets not forget that they are also absorbing) change the lapse rate?

        Is the observed lapse rate lower than the adiabatic lapse rate? Yes. ALR for the Earth is around 9.8 C/km whereas the observed rate is more like 6.5 C/km.

        But you are ignoring phase change – evaporation.and condensation. This produces what is known as the saturated or wet lapse rate produced by the energy transferred via the hydrological cycle. And this is closer to 5 C/km.

        The actual observed lapse rate at around 6.5 is a hybrid of these two factors.

        Interestingly on Venus the ALR is calculated as slightly above 10 which is what is observed.

        So a simple calculation Doug.

        The average altitude at which Venus radiates infrared to space – the TOA – is at around 50 km. The Stefan-Boltzmann balance temperature for a planet at Venus’s distance from the sun (108,000,000 km) and its bond albedo (0.9) should have a SB temperature of 228K

        And it has a lapse rate of 10.4 C.km

        So 50 km below the TOA altitude the temperature will be 228 +(10.4*50) = 748 K

        Mean surface temperature is around 737 K

        Not a bad match.

        The GH Effect is caused BY the presence of the lapse rate!

        GH Gases determine the average TOA altitude. Stefan-Boltzmann determines the average temperature at TOA. Then Lapse rate determines the rest of the atmospheric temperature profile around that.

        Doug. If you are going to produce any sort of coherent theory you need to consider 2nd Law, Conservation of Energy, Conservation of Mass, Phase change.

        1. Glenn,
          Just in the interest of scientific accuracy, I would say that your simple calculation is not actually simple at all, and, as a counter to Doug’s argument, ends up begging the question. The “average altitude at which [a planet] radiates IR to space” is not generally at the Top-of-Atmosphere (TOA).
          To find a value, typically, a black-body temperature or “brightness temperature” is calculated for the planet from S-B, and then the altitude is found which corresponds to that brightness temperature from the known vertical temperature profile. This then more or less guarantees that the surface temperature “predicted” from this value plus lapse rate will be correct.
          In the case of Venus, the BB temperature works out to be 184.2K . (I am not sure where you got your “SB temperature” from. The value of 184.2K is obtained using a Bond albedo of 0.9 and an emissivity of 1. Solar insolation is 2613.9 W/m^2.) This corresponds to an effective radiation height of 53.1 kilometres (well below TOA), using a lapse rate of 10.4 deg C/km and a surface temperature of 737K. Unsurprisingly, if we now reverse the calculation and use the BB temperature and lapse rate to calculate the surface temperature, then we will reproduce the surface temperature of 737K which we started with. This does not add any insight.
          There is unfortunately no independent way of calculating the effective radiative emission height without knowing the temperature profile, so as a predictor or explainer of surface temperature, it’s not helpful. I would also add that it does not exist as a measurable physical entity – it is more a conceptual curiosity. The physical reality is that different wavelengths emit at different intensities from each vertical layer – average emission height varies by wavenumber. While one can find a definition which uniquely relates the flux profile to an elevation (e.g. the height where the outgoing flux contribution from below equals the outgoing contribution from above) there is no guarantee that this elevation will also correspond to the elevation where the BB temperature corresponds to the physical temperature.

          1. Paul

            The term TOA is used in the climate sense as the effective radiating height, not the literal top of atmosphere.

            And yes, TOA is a conceptual altitude. At each different wavelength the altitude will actually be different. And the temperature associated with TOA will also be different for each wavelength because of those different altitudes. And the TOA altitude isn’t precise. It is a band of altitude where the atmosphere goes from optically thick to sufficiently optically thin However it is still a reasonable approximation to think of a median height where radiative balance occurs.

            You are right that it is hard to calculate TOA by the above description using anything other than a Radiative Transfer Code. I was citing that approximate figure as one I have read for that height, not based on the SB temp and lapse rate. To calculate the TOA height you do need the pressure and temperature profile for the atmosphere – pressure being the more important factor since it has the bigger impact on determining when the atmosphere becomes optically thin at any wavelength.

            However, it isn’t true that we don’t know that profile. Probes have determined some of these values. Perhaps not as exactly as here on Earth but still usefully so. Thus we have an independent basis for estimating TOA, even TOA for each wavelength. It just needs serous number crunching, a full RTC and those profiles and is beyond the scope of the sort of back-of-an envelope calculations possible on a blog. All we can do is use indicative arguments.

            But the determination of TOA, perhaps a bit imprecisely, is still something that is possible independent of SB/Lapse rate arguments. So too here on Earth where the available data is much better. It is the capacity to calculate TOA independently that makes this not a form of circular reasoning.

      3. Doug,
        You are indeed rambling incomprehensibly. I have just posted the following on Bishop Hill where you left some of your droppings.

        “@Doug Cotton,
        Instead of spewing cr*p all over perfectly respectable blogs, can I suggest that you return to the Air Vent and answer Jeff’s question. If you cannot, and I am certain that you cannot, then please accept that your position is untenable, thank Jeff for the education, and move on. It will leave you with a lot more dignity than the path you have adopted. “

        1. I repeat …

          It’s time to move on, Jeff. What would you deduce if your carbon dioxide atmosphere produced a slightly warmer surface temperature anyway – say 310K rather than 300K for the nitrogen one? I suggest any deduction pertaining to a watery world where there already is a mean radiating altitude up in the troposphere would be quite irrelevant. I’m not agreeing with 310K, and you can’t prove it either.

          Your starting point, Jeff: You assume 2,500 molecules of carbon dioxide in 2.500 molecules produces, say, 10 degrees of warming. That came from 300K by a nitrogen atmosphere to 310K in a carbon dioxide atmosphere. So 1 molecule in 2,500 produces 0.004 degrees of warming, and if we double it we get a sensitivity of 0.004C.

          1. what?

            and no you finally answered the question above and you did not agree with standard physics so it led to a new question. All perfectly reasonable.

        2. I’m not conceding the 310K. My best estimate is that the nitrogen only atmosphere would indeed have a surface temperature of 300K and that pure carbon dioxide would reduce that to around 295K but if I’m wrong, even though neither of us can prove it, my point is that, let’s say the radiating altitude was 7Km and the gradient 8C/Km, that gives us 255K + 56 = 311K. And that’s a sensitivity less than 0.005C for doubling existing CO2 levels. You can play around with other figures but the sensitivity is still minuscule.

          1. The N2 planet emits directly from the surface at zero altitude – per your previous agreement.
            the CO2 planet emits from a higher average altitude – per your previous agreement
            Both planets have a temperature gradient with altitude — cooler being higher.

            Both planets must emit at the same temperature as seen from space to maintain radiative balance. You have just stated above that in order to avoid the obvious warming the CO2 planet in our example would experience, there is ACTUALLY a lower emission temperature, which is a NEW addition to the Doug Cotton theory as explained here. It is a fairly KEY point I would think and probably should have gone on page one, chapter one of your book.

            How is the emission temperature lower on the CO2 planet, yet the radiative balance maintained?

          2. Your understanding of how they calculate a planet’s radiating temperature is seriously lacking, and you have not digested the explanation I have already given, which indicates to me you are not concentrating on my explanations one little bit. If emissivity is lower the actual mean radiating temperature is higher. That’s why the rocky planet with nitrogen is around 300K – that’s why I quoted those emissivity figures – remember? PLUS, the solar radiation is nearly double what the surface of the real world receives (315W/m^2 compared with 161W/m^2) – remember? Try using an on-line Stefan Boltzmann calculator some time. You’ll find all the above already in previous comments, so stop wasting my time. I have work to do now, as I still do each afternoon until 6pm Sydney time in my part time business supporting three kids, so you have time to sit back and think. What the hell is it worth wasting time over discussing whether sensitivity is plus 0.005 or minus 0.005?

          3. Already here before you Douglas!

            Emissivity of CO2 is much much lower than any rock you can imagine. that means higher temps not lower. Uh oh… You may want to pick a different direction to explain why emission temps are lower.

            ————-

            I’m going to take this comment back. Emissivity of CO2 in Earth is much lower but in a pure CO2 atmosphere, things change and without definition we must make assumptions. However, the question here stated that the planets experience the same incoming energy.

          4. Incidentally, the only references ot emissivity I find for CO2 at even unreasonable pressure are lower than what we would find for most rock. If you make the claim that emissivity of CO2 is higher, I do expect a reference.

          5. Doug

            Nasif Nahle?????

            “The temperature introduced in the calculations is 300.15 K (27 °C):”

            Gee that looks very familiar Doug. Are you and Nasif channeling the same guru.

            Is his derivation of 300.15 any better than yours?

            If you want to convince people, why not try citing the mainstream literature Doug. If someones only claim to fame is the blogosphere maybe one should be a little wary of their views.

          6. “the temperature of CO2”

            Um…, Arghhh…

            Doug. WTF!!!

            CO2 is one component of the atmosphere. The Atmosphere has a temperature. Individual molecules within it don’t.

            Your comments are becoming steadily more erratic as time goes by.

        3. I don’t consider some of what the IPCC states to be standard physics. If you ever accuse me of not using standard physics then cite references (which you never do for your assertive statements) and spell out in detail which law of physics you are referring to, what that law says, or what you think it says (with references and hyperlinks) and why you think I am saying something to the contrary. You won’t succeed on that front, my friend, in any significant matter and relevant physics.

          1. How is the emission temperature lower on the CO2 planet, yet the radiative balance maintained?

          2. Oh, and Loschmit, Bolometers, Radiation, global warming etc. . all non-standard. But how about if you answer the question above instead?

          3. I did in several previous comments, repeated above at 9.34pm. Over and out.

          4. Loschmit (sic): Based on Second Law

            Bolometers: Based on standard radiation equations, measuring rate of cooling if object is at lower temperature

            Radiation: One way radiation obeys Second Law which specifically refers to a process, not a combination of independent processes. Water does not run uphill in one process because it runs further downhill in another independent process. The Second Law is what tells us that can’t happen because entropy does not decrease in any independent process.

            Global warming: Natural cycles are discussed by many scientists these days. The correlation with the scalar sum of angular momentum of Sun and planets is compelling evidence that planetary orbits regulate these 1000 year and 60 year cycles. Did you ever look at the plot of this AM at the foot of the Home page on my earth-climate dot com website? It’s been there for nearly four years.

          5. I can argue with almost every comment you made above but I won’t be sidetracked just because you are losing.

            How is the emission temperature lower on the CO2 planet, yet the radiative balance maintained?

          6. Because, as on Venus, the surface plays a far smaller role in the emission to space when you fill the atmosphere with CO2 that acquires over 97% of the energy in incident solar radiation on Venus. Virtually all radiation and emission takes place in the upper troposphere and above, so that region does in fact act very close to a black body with emissivity perhaps just a little below 1.0 and certainly higher than Earth’s surface emissivity would be without water. In general, I’m not arguing about the 255K figure for Earth. I’m just explaining why the rocky Earth with N2 only would be around 300K, not 255K. Now that does not leave you much scope for any significant temperature increase above 300K with a pure carbon dioxide troposphere say 11Km high on average. The gradient can’t exceed 9.8C/Km and we know from Venus it would be at least 20% less than that. Whatever you stretch it to, by the time you divide by 2,500 to get the effect of concentrations of CO2 in Earth’s atmosphere your sensitivity is minuscule – orders of magnitude less than IPCC estimates, and that’s the main point I’m interested in proving.

          7. A temperature of 300K is supported by the known radiation of about 315W/m^2 at Earth’s surface (with no reflection or absorption of incident solar radiation) and a mean emissivity of 0.6858710562414266 which you could not prove to be an unrealistic figure. I’m never in a tight spot my friend, but this must be over and out.

          8. My best estimate was based on a mean CO2 radiating altitude of 6Km and gradient 7C/Km giving 255 + (6 x 7) = 297K with obvious error margins. In the real world I have estimated all the carbon dioxide cools by 0.1 degree.

          9. Because, as on Venus, the surface plays a far smaller role in the emission to space when you fill the atmosphere with CO2 that acquires over 97% of the energy in incident solar radiation on Venus.

            Venus has an albedo of 0.67 which is why it is so bright when you look at it. Only 33% is absorbed.

            After that, you are talking again about Earth and not our sample experiment so there is no comment.

            how tight is the spot —
            Knights of nee tight

          10. If you had concentrated on my previous comments I was obviously talking about the percentage of radiation entering the Venus atmosphere after reflection near TOA. That would not be all that different from what would enter at the top of Earth’s CO2 atmosphere. You love to side-track onto irrelevant points don’t you. I really don’t care if what reaches the Venus surface is 90% of TOA flux or more.

          11. Doug

            Your use of a mean emissivity of 0.6858710562414266 in a comment above is way off as I discussed earlier.

            The mean emissivity of virtually every substance on the earth’s surface is more like 0.97 to 0.98 in the far infrared. I suspect you are using averages of the emissivity of substances in the visible range as a basis for your 0.68… number. Redo the calculation with 0.98 and see what you get.

          12. Glenn please get with what we are discussing – a waterless, carbon dioxide less, life-less blob of a planet presumably with rock over all its surface. I have given you the source of emissivities that I’ve quoted. What is your estimated breakdown of the various types of rock surface. It’s 51 years since I last studied geology, and my grandfather who was a professor of geology ain’t around to ask.

          13. http://www.optotherm.com/emiss-table.htm

            Basalt 0.70
            Granite Natural 0.95
            Gravel 0.90 – 0.95
            Gypsum 0.85 – 0.95
            Limestone 0.95 – 1.00
            Quartz 0.90
            Sandstone 0.70
            Shale 0.70
            Soil Dry 0.90 – 0.95
            Slate 0.70 – 0.80

            These two sources differ substantially in their values for Granite and you haven’t mentioned many other rocks.

            And since at present you aren’t currently discussing a GH Effect free earth then you need to make sure you don’t imply that you are. If you want to do a calculation for non-GH, non-earth fine. But a non-GH, real Earth would be more relevant.

            So lets assume that 70% of this waterless planet is covered by rocks with the same emissivities as the corresponding water here on earth.
            Water Replace with
            Liquid 0.90 – 0.95 Dry Soil , Limestone or Quartz
            Ice 0.95 – 1.00 Limestone
            Snow 0.80 – 1.00 Gypsum & Limestone

            High 0.9’s still looks like the reasonable number for an earth analog.

          14. “A temperature of 300K is supported by the known radiation of about 315W/m^2 at Earth’s surface (with no reflection or absorption of incident solar radiation) and a mean emissivity of 0.6858710562414266 which you could not prove to be an unrealistic figure”

            Say What??

            But there is absorption of incident radiation by the atmosphere. Mainly by oxygen. And you aren’t discussing your hypothetical N2 only non-earth. You said the Earth. And if you ignore reflection from the surface then you push your figure up again.

            And you said “as I have explained, the surface temperature could well be around 300K with a rocky lifeless Earth without water vapour and carbon dioxide? “. No mention of it being without oxygen!

            So define exactly what sort of planet you are talking about so you can handle atmospheric absorption properly, include reflection from the surface and try again Doug. Or state VERY CLEARLY when your examples have NOTHING to do with the Earth.

            Because Doug, if this sort of imprecise thinking is how you have formulated your ideas, I’m not surprised you get so much wrong.

          15. Doug,

            I accept the number 0.9 for this discussion, we shouldn’t get sidetracked with minutae. If the planetary emissivity on our planets is the same as the CO2 gas (which is very generous to your case), what happens to the surface temp.

          16. Doug,

            I stand corrected, it isn’t the Knights of nee, twas the black Knight I refer to.

            Your argument is completely lost unless you can explain something entirely new.

  8. Doug,

    I have a question. First some observations I have an IR gun that I use to take temperature measurements. When I am sailing in the Bahamas and point it at clear sky, the gun generally reads a little above 0˚ C day or night. When I am at my home in Idaho (6000′ elevation) the clear sky readings have a much broader range but they are always much below 0˚ generally about -20˚. Fully overcast skies read a few degrees lower than surface temperature in both locations.

    At night in the Bahamas when the sun goes down, the temperature doesn’t change much at all with cloudy skies or clear, midnight swims are enjoyable. At night in Idaho when the Sun goes down the temperature plummets down with it, often around 20˚ or more except when it is completely overcast. Going swimming at night in Idaho is an extremely chilling experience.

    So my question is why does the temperature drop at night in Idaho, but doesn’t drop in the Bahamas?

    1. In my study of 30 years of temperature data from three continents (in which I proved water vapour cools) I explained in the documentation of the methodology that I excluded regions which (like the Bahamas) are within 100Km of an ocean or a large inland water body because of the temperature levelling effect of such water bodies. We see it in Singapore where the temperature varies only between about 25C minimum and 31C maximum every day of the year. So of course the Bahamas have fairly uniform temperatures because they are affected by non-radiative heat transfers between them and the air above the ocean, if not by sea breezes also. Of course Idaho is inland and not thus affected. And it is at a higher altitude which also leads to colder temperatures, and of course it is much further from the Equator..

    2. A more interesting question is why does the rate of cooling in Idaho slow down in the early pre-dawn hours? Get out and measure it each half hour through the night (as I have done) and see if I’m right. The reason is that, once the temporary heat from the day has dissipated at a rate of, say 2 or 3 degrees per hour, there comes a point at which the supporting temperature at the base of the troposphere (determined by where the the projection of the thermal plot from higher in the troposphere meets the surface) kicks in. After that the whole troposphere would have to cool retaining the same temperature gradient because that gradient is the state of thermodynamic equilibrium. That’s why the Venus surface only cools by 5 degrees in 4 months and cooling slows down or even stops on calm nights in the early pre-dawn hours. This supports what I say about the gravitationally induced temperature gradient.

      1. Ahh, I think I see your problem Doug (and it is a common problem). If you compress air into a tank it gets hotter. Everyone agrees on that. If you take a long insulated tube of air and stand it up from the surface of the earth the bottom portion (the compressed portion) gets hotter (gravitationally induced temperature gradient). Similarly with regard to albedo if you warm two objects one white and one black the black one gets hotter faster.

        The thing is that at equilibrium the compressed air in the tank becomes isothermal with its surroundings, the same with the tube of air (your gravitationally induced temperature gradient) and the heating of the two objects with different albedos, at equilibrium they are the same temperature.

        The problem is that the ‘climate’ is a process. It isn’t static. A snapshot of initial conditions like the gravitationally induced temperature gradient (whose effect was gone over 3.5 billion years ago) is less than meaningless. What is interesting is the gravitationally induced pressure gradient where current processes like radiation and conduction warm the denser portions more than lower pressure areas. That is the AGW greenhouse effect.

        Of course the real greenhouse effect is the Oceans absorption of SW radiation and its primary method, evaporation, of emitting that energy. The atmospheric contribution by comparison is a rounding error. Comparisons with Venus and Mars, any of the other planets really, are not relevant for that reason.

  9. Just one thought led me to log in again. When you replace your first 10% of nitrogen with carbon dioxide, you may well see the mean radiating altitude jump to, say 5Km. But does the next 30% take it out of the troposphere to 20Km or 100% take it to 50Km? What the hell do you think you could deduce about sensitivity when, as I have explained, the surface temperature could well be around 300K with a rocky lifeless Earth without water vapour and carbon dioxide? Emissivity of sand = 0.76, granite 0.45, basalt 0.72, See this reference and tell me your estimate of a mean without any wood or vegetation of any kind: http://www.engineeringtoolbox.com/emissivity-coefficients-d_447.html

  10. Yes it seems even the best of us get misled by the IPCC some of the time – like regarding emissivity of CO2.

    The following is worth quoting in full regarding such emissivity of carbon dioxide. Note also the reference to real radiative equilibrium temperature of Earth as being 300.15K …

    the source is: http://www.biocab.org/emissivity_co2.html

    “The IPCC team of experts has changed the radiative forcing so many times that the IPCC team have had to admit that the numbers are not real. The real values for the emittance or “radiative forcing” have been provided by the heat transfer science and thermodynamics.

    Now the IPCC team has found that the radiative forcing changes erratically, induced by the climate changes. The last assertion from the IPCC team is an assumption without scientific support, and it contradicts the physics of heat transfer because the radiative forcing capability of any system depends of its physical characteristics, like specific heat, mass, enthalpy, etc. not in the state of climate. The climate is not driven by the radiative forcing of the elements in the atmosphere, but by the Sun and the oceans.

    Knowing the real values of emissivity, total emittance and absorptivity of the carbon dioxide obtained by other scientists by experimentation and observation of nature, the formula derived from the Stephan-Boltzmann’s equation adopts the following form, introducing values from the real world:

    ΔT = (0.423 W/m^2) [ln ([CO2] current / [CO2] standard)] / 4 (5.6697 x 10^-8 W/m^2*K^4) (300.15) ^3

    For example, the real radiative equilibrium temperature of Earth is 300.15 K (27 °C), and we want to know the anomaly caused by carbon dioxide, which concentration in the atmosphere was 381 ppmv. If the standard concentration of CO2 in the atmosphere is 280 ppmv (another subjective number because the real one was fixed by scientific associations and boards, and its value is 350 ppmv), the anomaly in the temperature of the lower troposphere (the layer of air just above the ground and in contact with the surface with not more than one meter thick) caused by CO2 (Partial Pressure from 381 ppmv [CcdL] = 0.00034 atm-m) under a total atmospheric pressure of 1 atm is:

    ΔT = (0.423 W/m^2) [ln (δ CO2 current / δ CO2 standard)]/4 (5.6697 x 10^-8 W/m^2* K^4) (T^3)

    ΔT = (0.423 W/m^2) [ln (0.000681 Kg/m^3 / 0.0005 Kg/m^3)] / 4 (5.6697 x 10^-8 W/m^2* K^4) (300.15 K) ^3

    = (0.423 W/m^2)(0.308) / 4 (5.6697 x 10^-8 W/m^2*K^4) (27040520.253 K^3) = 0.130284 W/m^2 / 6.13 W/m^2*K = 0.02 K

    Thus, the anomaly of the lower troposphere temperature caused by the increase of CO2, on June 15, 2007 at 18:05 hrs. (UT) was 0.02 K, which is equal to 0.02 °C. The maximum efficiency of the whole atmosphere was 1-[[Tatm/Tbb)^2] = 1 – [311.75 K/300.15 K)^2] = 0.08. A really low efficiency for a heat engine.

    Thus, carbon dioxide is not able to cause the global warming experienced again in the last two centuries.”

  11. Glenn and Jeff:

    As I said in an earlier comment, if the solar radiation reaching a planet’s surface is insufficient to maintain its observed temperature, then the gravitationally-induced temperature gradient takes control. So emissivity arguments may be irrelevant. The composition of a rocky Earth could possibly be similar to the Moon. “The crust of the Moon is composed mostly of oxygen, silicon, magnesium, iron, calcium, and aluminum. There are also trace elements like titanium, uranium, thorium, potassium and hydrogen.” [source]

    As you know, the Moon gets very hot (over 120C) and so too could your Nitrogen Earth in regions where the Sun is overhead. Then the radiation is about 1,260W/m^2 rather than the 24 hour global mean of 315W/m*22. Much of the thermal energy in the surface at such temperatures will transfer by non-radiative processes to the atmosphere and some of that energy will remain trapped in the system as it raises the temperature of the whole troposphere whilst retaining the temperature gradient of 9.8C/Km. There is not sufficient time for the whole system to cool sufficiently at night before the Sun warms it again the next day. It should not be surprising that the surface temperature could easily be maintained around 300K. Adding carbon dioxide would be like adding holes in the blanket, releasing some of the extra thermal energy by radiation to space.

    However, Jeff, you should not assume that the emissivity of CO2 (even lower than the low value I thought it had) determines its temperature. If you understood the Stefan Boltzmann result, coming as it does by integrating over the Planck function, it should be obvious that the relatively few thin spectral lines of CO2 come nowhere near filling in all the area under the Planck function, and so CO2 acts nothing at all like a black or grey body and SBL calculations are irrelevant.

    1. Doug

      “The composition of a rocky Earth could possibly be similar to the Moon”
      You are jumping around all over the place. The whole point of considering a GH free Earth is as a thought experiment. Not as an exploration of lunar geology

      ” So emissivity arguments may be irrelevant. ”
      Not to the point being discussed which is what a hypothetical GH Effect free Earth would be like. Without GH gases all emissions from the planet come from the surface. So what the gravitationally induced temperature gradient (i.e. Lapse Rate) may be actually doesn’t matter since it is radiation from the surface that determines radiative balance in this context. So surface emissivity actually matter totally in the topic under question.

      If you want to talk about what a planet with a GH effect is like, fine Doug, but that is a change of topic. Please don’t blur them together.

      1. Glenn: Please see first my comment 13.

        With respect, you haven’t come to grips with the fact that the gravitationally induced temperature gradient is the state of thermodynamic equilibrium – which is of course what the Second Law says will evolve. The consequences may surprise you. Consider, for example, why the base of the nominal troposphere of Uranus is hotter than Earth’s surface, even though there’s no direct solar radiation getting down there, no surface and no convincing evidence of net energy outflow due to cooling or internal energy generation. The explanation is in my book, some of which you can “Look Inside” here and two positive reviews you could read, but better still, spend an hour reading the book. I’m offering free copies to Jeff and those writing here if you email me a postal address to its.not.co2@gmail.com and, in that neither Jeff nor you yet have an understanding of the “heat creep” process, and its significance, I feel I must now stop wasting my time until you pay me the courtesy of reading there what I am saying in a level of detail that cannot be presented in short comments here.

      2. Footnote: If you use emissivity of 0.95 then the mean of 315W/m^2 yields 276.5K (about 3C) but that’s very approximate because it does not take account of the T^4 relationship. The maximum radiation of 1260W/m^2 gives 391K (118C) which is very close to the Moon’s maximum of 120C. If now Jeff’s 100% carbon dioxide atmosphere replaces the N2 atmosphere it will absorb some of the incident solar radiation in the 2.1 micron range, as happens on Venus, and so less solar radiation reaches the surface, and so the temperatures are lower – if you are correct in believing it’s all about radiation.. You cannot add the back radiative flux to the solar flux and use that in S-B calculations. The back radiation from a colder atmosphere cannot help the Sun achieve a higher maximum than that 118C. That conjecture is a complete travesty of physics from the IPCC.

        1. ” if you are correct in believing it’s all about radiation.. You cannot add the back radiative flux to the solar flux and use that in S-B calculations.”

          Doug, haven’t you taken in a single thing I have said? Where did I say it’s all about radiation? I have been discussing repeatedly how atmospheric mixing generates the lapse rate and the GH effect is produced by the combination of the radiative balance AND the Lapse Rate. Even the incoming rdiation is completely absorbed before the surface, the mixing associated with the Lapse Ratte will still create a temperature profile all the way to the surface.

          And Back Radiation being absorbed by the surface isn’t a travesty of physics. It is standard physics and it doesn’t come from the IPCC it comes from standard thermodynamics. Including the standard thermodynamics I was taught as a young engineer nearly 40 years ago. And please don’t quote Claes Johnson’s bit of absurd mathtubation at me.

        2. Doug (and others)

          You might find the following an interesting read (although it is written in very 19th century English and is a little turgid by modern standards)

          http://books.google.com.au/books?id=8LIEAAAAYAAJ&printsec=frontcover&dq=editions:PwR_Sbkwa8IC&hl=en&sa=X&ei=h6DgT5WnF46e8gSVvbynDQ&redir_esc=y#v=onepage&q&f=false

          THE MECHANICAL THEORY OF HEAT
          WITH ITS
          APPLICATIONS TO THE STEAM-ENGINE
          AND TO THE
          PHYSICAL PROPERTIES OF BODIES

          BY
          R. CLAUSIUS
          1867

          (yes,that Clausius)

          Contains several memoirs including the Fourth Memoir on page 111

          ON A MODIFED FORM OF THE SECOND FUNDAMENTAL THEOREM (the 2nd Law of Thermodynamics) IN THE MECHANICAL THEORY OF HEAT

          Read the footnotes on Pages 117 & 118

          “In the first place, the principle implies that in the immediate interchange of heat between two bodies by conduction and radiation, the warmer body never receives more heat from the colder body than it imparts to it…..
          …On considering the results of such processes more closely, we find that in one and the same process heat may be carried from a colder to a warmer body and another quantity of heat transferred from a warmer body to acolder body without any other permanent change occurring. In this case we have not a simple transmission of heat from a colder to a warmer body, or an ascending transmission of heat, as it may be called, but two connected transmissions of opposite characters, one ascending and the other descending, which compensate each other. …
          …Now it is to these compensations that our principle refers; and with the aid of this conception he principle may also be expressed thus: an uncompensated transmission of heat from a colder to a warmer body can never occur.”

          Clausius himself articulated the allowed bidirectional nature of heat flows between colder and warmer bodies and that it is the net flow (the ‘uncompensated transmission’) that the law is about.

          1. Glenn,
            Yes, Clausius introduces two streams just to say they are not treated independently by thermodynamics. When climatologists will understand this subtlety, we’ll be halfway to the solution.

  12. All the calculations using a quarter of the solar flux (further reduced if applicable by 30% for albedo and 20% for absorption) are in effect treating the Earth as a flat surface receiving a quarter of the actual solar radiation for 24 hours a day. The excuse is made that this gives a reasonable estimate in the real world with its stabilising atmosphere and oceans. But in Jeff’s imaginary Earth with 100% nitrogen atmosphere supposedly not absorbing or emitting anything at all (somewhat questionable) the temperature variations would be somewhat larger and I have indicated temperatures could reach 100C or more as they do on the Moon. With such variation the T^4 relationship in the S-B calculations becomes significant, so far more energy is radiated out at the high temperatures leaving less that needs to be radiated at lower temperatures. Some energy transfers by conduction to the atmosphere during the day, then back to the surface at night, raising the minimum temperature. So, all in all, it is entirely plausible that the mean surface temperature could well be over 300K. With carbon dioxide emitting so few spectral lines, and only able to slow radiative cooling of the surface (not non-radiative cooling into the atmosphere) it is (as I said in 2012) like “a picket fence (just a few spectral emission lines) from CO2 up against the flood of all frequencies coming from the surface.” [source]

    So what do we have Jeff? Independent estimates of the total effect of current levels of CO2 being from 0.1 degree of cooling to 0.02 °C of warming, as in the above quote in Comment 11. You can calculate what you think your thought experiment proves in that regard. In addition, we have empirical evidence proving water vapour cools, in line with what my hypothesis says it should do. So any warming by carbon dioxide would be more than offset by negative feedback by water vapour. If all that doesn’t prove my main contention that the whole greenhouse conjecture is false, then the physics in my book does. QED

    [REPLY, I assume your avoidance of answering a very simple question regarding emission altitude means that you recognize now that CO2 does in fact cause warming.

    or can we expect—no my boy, it’s Just a flesh wound!

    –Jeff]

    1. Doug

      ” But in Jeff’s imaginary Earth with 100% nitrogen atmosphere supposedly not absorbing or emitting anything at all (somewhat questionable) ”

      Data on Nitrogen emission/absorption here: http://www.cfa.harvard.edu/hitran/
      Plots of spectral intensity here: http://www.spectralcalc.com/spectral_browser/db_intensity.php
      (select N2 and plot over say wavenumber 500 to 2000. Repeat for CO2 for the same wavenumbers)

      “and I have indicated temperatures could reach 100C or more as they do on the Moon”

      ‘could’!!!
      How so rigorously mathematical of you Doug! Even though you haven’t put forward one calculation to support this contention. Doug you idea of analysis seems to be that if I wave my hands around and make connections between disparate concepts using just words that I have done a mathematical proof.

      QED. Quietly Evading Discussion

    2. Yes, as per the calculations below, in the real world I concede a doubling of carbon dioxide warms by about 0.008C if in fact its median altitude is about 1Km above the median altitude of water vapour. In general the median altitudes do not increase with increasing concentrations, because the density gradient remains the same. So I don’t know whether the median altitude of carbon dioxide is in fact 1Km higher than that of water vapour. If it is the same, then there is no warming effect from doubling existing concentrations of carbon dioxide. So if you can present me with information about the relative median altitudes of the existing concentrations of carbon dioxide and water vapour and then, if that for carbon dioxide is more than 0.2Km above that for water vapour, then and only then would its warming effect start to exceed that of the cooling effect that is due to reducing the temperature gradient. Discussion of changes between 0% and 0.04% carbon dioxide are irrelevant in the absence of water and water vapour because a world like that could have maximum temperatures around 120C which I suggest is a far greater problem than variations in the mean. It’s lucky we have water. We wouldn’t be here discussing the issue if we didn’t. And without carbon dioxide as well, the world would be a rocky planet which I still say would have emissivity more like 0.8 and thus a mean temperature more like 288K than the IPCC estimate of 255K.

  13. @Glenn Tamblyn

    Glenn,
    Thanks for your response. I know that this is a side-show to the main event, but I still think your argument is circular.

    We are not using common definitions, which doesn’t help. You stated:
    “The term TOA is used in the climate sense as the effective radiating height, not the literal top of atmosphere.”

    Normally, when the TOA is defined in terms of radiative balance, it is the level at which the downward shortwave radiative flux is negligibly different from the solar constant and where there is negligible downward longwave flux (other than that due to the sun, which is small).
    In terms of Earth atmosphere, that places the TOA in the low to mid-stratosphere.
    On the other hand, the ‘effective emission level’ or ‘effective radiating height’ is typically defined as the level at which the climatological annual mean tropospheric temperature is equal to the emission temperature: (OLR/σ)1/4, where σ is the Stefan–Boltzmann constant. More loosely, it is the height at which the physical temperature of the planet corresponds to the brightness temperature of a black-body emitter at equivalent intensity.
    In terms of Earth atmosphere, the effective radiating height varies latitudinally between 4 and 5 kms – i.e. within the troposphere and well below the level of the TOA.

    My main point remains that knowing the brightness temperature of a planet does not allow you to estimate the effective radiating height and lapse rate, and hence the surface temperature of the planet. For this, you have to have some independent way of estimating the vertical temperature profile of the planet – which is begging the question. If you run a probe and take sample measurements of temperature, pressure and composition at different elevations, then certainly you can calculate the effective radiating height (and lapse rate). You can then calculate the surface temperature from the brightness temperature. Unless you have made a gross error, you should find that it is consistent with the temperature values which you measured, but it proves nothing other than the self-consistency of the calculations. I know of no means for estimating the effective radiating height, which does not involve already having some estimate of the vertical temperature profile. Your estimate of (your definition of) TOA at 50kms comes from where exactly?
    For the avoidance of doubt, I accept completely your statement: “The GH Effect is caused BY the presence of the lapse rate!”
    I just reject the validity of this particular argument which you presented to Doug Cotton.

    1. Yes Paul, I know that’s how the IPCC authors define the radiating altitude, but it’s irrelevant, as I explain in my book and in my comment below. The median radiating altitude is where the thermal plot pivots and that’s about 3.5 to 4Km, not 5Km. The IPCC forgot the T^4 relationship it seems. Water vapour reduces the gradient and, as empirical evidence proves, it cools by about 12 degrees rather than warms, because the thermal plot rotates downwards about the pivoting (median) altitude. That single fact smashes the greenhouse.

  14. It’s time to talk about the real world folks. I concede that carbon dioxide can potentially raise the median radiating altitude by a very small amount, but at the same time its radiating properties have a temperature levelling effect that works against the gravitationally induced temperature gradient. What we are interested in (and the world’s politicians ought to be interested in) is a realistic sensitivity figure in the vicinity of current concentrations of water vapour and carbon dioxide.

    Now, this median radiating altitude is what I call the pivoting altitude in my book, because the whole plot of temperature against altitude in the troposphere rotates about that altitude – downward at the surface end as the temperature levelling effect of radiation increases with increasing concentrations of radiating molecules. Where the plot meets the surface in calm conditions in the early pre-dawn hours is roughly the minimum surface temperature at that time of the year for the existing concentration of water vapour, that being usually from a little less than 1% to 4% in the lower troposphere. The median altitude is that where half the energy radiated upwards comes from below that altitude and half from above. Because water vapour concentration peaks in low altitudes (about 3Km) and the surface itself radiates, the median radiating altitude is probably in the vicinity of 3.5 to 4Km where the troposphere is at a mean height of about 11Km.

    Now, carbon dioxide may have a higher mean altitude than water vapour, but it is unlikely to be more than 4.5Km because obviously density is greater at lower altitudes. I am being generous in assuming its median altitude is 1Km above that of all the other radiation. I doubt that it is that much. But if it were, then let us assume the mean percentage of water vapour is 1.5% and that of carbon dioxide doubles from 0.04% to 0.08%. In fact carbon dioxide radiates in far fewer bands than water vapour and its effect on the overall mean radiative flux is thus much less for any given concentration in the atmosphere. Let’s say it has 10% the effect of water vapour. If I’m way out you can adjust the calculations. But even just comparing 10% of 0.04% to 1.5% of water vapour, and using a mean of 1Km above that of water vapour, I calculate that would raise the median altitude by the fraction 0.004/1.5 of 1Km which is 0.00267Km. However, from Venus data we know that 98% carbon dioxide reduces the temperature gradient by about 20%. The higher altitude would raise the temperature by about 0.01 degree over 3.5 to 4Km, and the reduction in the gradient by 0.04% carbon dioxide would lower it by perhaps 0.008% of 6.5C/Km , that is about 0.0005C/Km or about 0.002C in 4Km. So I concede the net warming effect by doubling carbon dioxide would be 0.01 – 0.002 = 0.008C. Big deal. Worth spending billions?

    1. Doug (and others)

      Go to the University of Chicago’s website here http://forecast.uchicago.edu/modtran.html which lets you generate the outgoing long wave radiation spectrum that will occur for different conditions. The default settings are for a tropical atmosphere, no clouds, looking down from 70 km up. Look at the shape of the ‘notch’ formed by CO2. Contrast that with the atmospheric temperature profile they have used on the right.

      Let you eye move inwards from either side of the notch to wards it’s centre. As you move down the walls of the notch you are passing over Planck curves for steadily lower temperatures. This corresponds to ever higher altitudes on the corresponding temperature vs height plot on the right. By the time you reach the flatter region at the bottom of the notch you are at the tropopause. Temperatues have stopped dropping. move inwards to the central notch – now you are above the tropoause. Move up the central notch to the top. Now you are moving up into the stratosphere where temperatures are rising. The ‘temperatures’ on the OLR pectrum don’t match the profile exactly – the bottom is only around 220K where the temperature profile drops below 200K – because the radiation isn’t generated at a precise altitude but over a band of altitudes where it starts to become optically thin.But you are seeing the vertical temperature profile reflected in the shape of the OLR spectrum.

      And the height associated with the central peak for CO2 isn’t 4.5 km Doug, it’s more like 30 to 40 km. Whereas the heights associated with the walls of the ‘notch’ vary enormously, from near zero to 20 km.

      Now try some what-if’s.
      – Turn CO2 (set it to zero ppm) and Water (set water vapour scale to zero) off separately to see what happens when you only have one or the other.
      – Try different latitude bands, paying attention to how the temperature profile on the right changes
      – Try using different cloud models to see how much of an impact that has
      – Try setting CO2 quite high – say 4000ppm to make the change visible – and see what changes.
      – Try varying the lookdown altitude. As you start to drop below 30km features from the spectrum start to disappear, with the spike at the centre of the CO2 notch disappearing first. As you drop below an altitude you can no longer see a feature generated above that.

      If you want a sense of how CO2’s absorption/emission can generate such widely varying behaviour at different wavlengthsgot to the spectralcalc site here http://www.spectralcalc.com/spectral_browser/db_intensity.php. Plot the spectral intensity for CO2, for wave numbers from 500 to 1000 cm^-1. Look how varied it is – the default vertical scale is logarithmic – so there is a variation of over 6 orders of magnitude.

      I think the spectroscopists and atmospheric physicists that deal with this have a little, little bit more understanding of this subject than just Doug doing some back-of-an envelope calculations.

      And if you doubt whether Modtran is reliable (or any other Radiative Transfer Code for that matter) then why don’t you drop the authors of the program an email and query them about it. That would be the US Air Force. They are the owners of many of th patents behind ModTran. For many years any developments of MdTran were signed-off on by the Commandant of the USAF GeoPhysics Laboratory at Hanscom AFB.

      If you want to query the data in th HiTran spectroscopic database, please contact the folks at the USAF Cambridge Research Laboratory at Harvard, they are the people behind it.

      I hope this gives some sense of just how amateurish and simplistic any back of an envelope calculations done here actually are. And maybe read the comments of someone like Doug in that light – ‘the IPCC got things wrong’ type of stuff. LOL.

    2. Glenn and others:

      You must be aware of the density gradient in Earth’s atmosphere. There must be considerably more molecules of any of the gases in our atmosphere in the lower troposphere than in the upper troposphere, let alone the stratosphere and mesosphere where density is very low indeed. Furthermore, the only effect that radiation from carbon dioxide can have is to slow that portion of surface cooling which is itself by way of radiation. Standard physics tells us that the rate of slowing gets greater and thus more effective as the temperature difference between the surface and the radiating molecules reduces. So molecules near the surface slow the radiative cooling much more than those that are considerably colder near the tropopause. The cooling effect depends on the relative area under the Planck curves, so a genuine black body with full spectrum radiation is far more effective than carbon dioxide with its limited spectral lines, the strongest of which corresponds to temperatures colder than -80C as found only in the mesosphere. So the strongest radiation from carbon dioxide is also the most ineffective because it comes from a region where the temperature difference between it and the surface is about 100 degrees.

      Furthermore, when you understand the whole new 21st century paradigm pertaining to the supporting temperatures at the base of a planet’s troposphere (due to the gravitationally induced temperature gradient) you will realise that the rate of cooling does not affect the minimum daily temperatures which are supported when cooling slows down and often stops in the early pre-dawn hours. Nor can radiation from the colder atmosphere help the Sun to raise the surface temperature to a higher maximum in the middle of the day. Ultimately, all that matters is where the temperature plot meets the surface. But the issue is, how does the necessary energy get there, and that’s what the “heat creep” process is all about, as my book explains with totally valid physics I assure you.

      [OT: Be the first to see this video just uploaded of my 8 year old lawn bowler https://www.youtube.com/watch?v=viB48QW-AvY&feature=youtu.be ]

      That’s all folks.

    3. “It’s time to talk about the real world folks. I concede that carbon dioxide can potentially raise the median radiating altitude by a very small amount, but at the same time its radiating properties have a temperature levelling effect that works against the gravitationally induced temperature gradient. “

      Baby steps Doug.

      So now can you finally admit that CO2 in at least some circumstances will cause warming?

      The height increases, the lapse rate is not inverted.

      1.  

         

         

        No Jeff – I only agreed conditionally: you have not produced any evidence that the median height of CO2 is more than 0.2Km above the median height of water vapour. I see no reason why it should be, because the density gradient affects all molecules of all gases equally,

        I have pointed out that any conceivable effect is absolutely minuscule.

        I have evidence that water vapour cools. Its median altitude (and so its radiating altitude) does not rise, but it reduces th etemperature gradient and thus the surface temperature. My comprehensive study of real world data confirms that it cools. Where is any contrary evidence, Jeff?

        No raising of any radiating altitude raises the minimum surface temperature. It may just mean that the warmth of the day lasts a few seconds or minutes longer into the night, but the cooling still slows far more due to conduction into nitrogen and oxygen molecules, and the cooling even stops at the supported temperature.

        Until you people read and understand the new 21st century paradigm explained in my book you are way off track.

        Jeff cannot prove that the increase in the 30 years up to 1998 was anything more than the combined effect of the 1000 year cycle still rising and the 60 year cycle rising for those 30 years. The latter is still half way down, so slight net cooling will continue until about 2028. Eventually 500 years of cooling will start, probably after the year 2059.

        I repeat: water vapour cools: I have solid empirical evidence. You have never presented evidence to the contrary, so I rest my case.

        As I said, that’s all folks.

        Bye.

         
         

        1. Doug

          “No Jeff – I only agreed conditionally: you have not produced any evidence that the median height of CO2 is more than 0.2Km above the median height of water vapour. I see no reason why it should be, because the density gradient affects all molecules of all gases equally,”

          Doug, GO AND LOOK AT THE SPECTRAL DATA AT spectralcalc.com I linked to above. The evidence you are asking Jeff for is right there. The emission altitude for CO2 varies over a vertical range of 30-40 km depending on which wavelength you are looking at. As long as you are only discussing densities, gradients etc you really have zero understanding of what you are talking about. Just stop and think Doug. Emission/aborption depends on density AND the probability that a molecule will absorb. This probability varies enormously across the differing wavelengths in a molecules absorption band. Enormously.

          If you don’t understand the full subject matter of the topic you are pontificating about then you might want to consider the possibility that you don’t know what you are talking about. That you haven’t discovered something new, but rather are making the old mistakes that the professionals made and corrected a long time ago.

          You don’t have a 21st century paradigm Doug, actually much more like a 19th century one.

          Questions, which you should know the answers to if you actually have even a modest grasp of the subject matter:

          Why are molecules that do not have a Molecular Dipole unable to act as GH gases?

          Why is any molecule that is made up of two identical atoms (such as N2 and O2) unable to be a GH gas?

          Why does the fact that water vapour is able to sustain a permanent molecular dipole mean that it can absorb and emit infrared radiation through both vibration of the molecule and rotation of the molecule, giving it a much broader absorption spectrum? In contrast CO2 for example can only absorb/emit due to molecular vibrations because it can only sustain a transient molecular dipole.

          Why does the temperature of the gas alter the absorption properties of the gas through doppler shift?

          Why does the absorption properties of a gas change substantially with pressure?

          Doug. If you don’t know the answers to these questions, and perhaps more importantly don’t understand why they are relevant, how can you be so sure you understand enough about this subject.

          Donald Rumsfeld and his ‘unknown unknown’s is coming strongly to mind here.

          1. Well if you think CO2 molecules at 30 to 40Km altitudes are somehow raising the maximum daily temperature of Earth’s surface then you need to explain how they cause extra thermal energy to transfer from the stratosphere into the surface either by radiative or non-radiative processes whilst the Sun is warming that surface each clear sunny morning. For a start, the temperature gradient is opposite in sign up there, but it’s colder than the surface, so any radiation between there and the surface is transferring thermal energy out of the surface, not into it. All the carbon dioxide in Earth’s atmosphere could not possibly raise the radiating temperature by more than 1.5 metres, that having negligible consequences.

          2. The role of radiating molecules in the troposphere is to help outward energy flow by inter-molecular radiation and direct radiation to space. Radiation only transfers thermal energy from warmer to cooler regions, and all one-way passages of radiation obey the Second Law throughout the Universe. Radiating molecules reduce the magnitude of the gravitationally induced temperature gradient in a planet’s troposphere, as you know happens for water vapour. The reduction is usually less than a third, and can be as small as 5% to 10% such as in the Uranus troposphere where the radiating altitude is close to TOA where the temperature is about 60K. That 60K layer is the heat source for the whole planet, and what you don’t understand is how the temperature gradient sets the slope of the thermal profile that is anchored by that 60K layer. There is non-radiative “heat creep” up the thermal profile, and that’s what the new paradigm is all about. That’s what you need to understand, my friend.

      2. “No Jeff – I only agreed conditionally: ”

        so you are saying no/yes? I can’t understand?

        ———————-
        This is your theory Doug, if you cannot explain just the basic concept and you cannot answer even the most basic questions, it is doomed to the failure it now seems to deserve. You can write conclusion after conclusion that flies in the face of standard physics, but unless you can explain the foundation of whatever is different in your theory from standard (and childishly obvious) physics.

        I realize you have theorized yourself into a complete corner where you must agree that CO2 can cause warming in some instances. This goes against all of your ranting pontification style conclusions, however, it is where your understanding has taken you. No amount of “faith” can change the answer. Now I admit that you haven’t admitted Earth is the same situation but were we to continue this type of discussion, for say another month or so because your limited understanding of physics cannot handle the complexity, you would come to the same conclusion that CO2 MUST raise the emission altitude on Earth as well. You would then recognize that even a strong negative feedback which doesn’t seem to exist in the more complex situation of earth cannot bring the stability point below zero. It is literally impossible mathematically for a stable system to end up cooler when heat is added.

        I reject your ever-changing theory on the grounds that it has no foundation in science.

      3. DOUg 12:26am: “Where is any contrary evidence, Jeff?”

        Jeff et. al. will be interested to know the contrary evidence is shown pp. 365-6 sec. 7.1 of Dr. Craig Bohren’s 1998 Atm. Thermo. text. I’ve pointed this out to DOUg in the past and the basic accurate science was, ahem, not well received by DOUg.

        Atmospheric radiation at the ground depends on vertical water vapor density and temperature profiles for which Prof. Bohren runs thru the column wv calculations at a desert site and non-desert site. The process shows precisely where DOUg gets it wrong in his methods described to be in his book. The science proving DOUg wrong was available in 1998.

        I agree DOUg’s ever changing comment theory should be rejected on the grounds it is not tied to a foundation in science. DOUg’s book in part was proven wrong specifically in 1998. Possibly the 2nd edition will show the corrections now they have been pointed out, there is always hope.

  15. Others have no doubt made the same point (and I realize Jeff, that you are focused on a slightly different nuance), but it is worth pointing out occasionally, just to keep us all honest, that increasing CO2 does not *cause* an increase in the global average temperature (however we want to calculate that “average,” which is a separate issue).

    This is a basic, empirical, observed fact. All we have to do is look at the CO2 numbers (increasing as they have been for decades), and compare that to the temperature record, whether we want to look at the recent 15-18 year “pause” or the cooling in the early middle half of the last century. In either case we had (i) an increase in CO2, along with (ii) a decrease or stagnation of the global average temperature; both at the same time. Ergo, CO2 does not cause the global average temperature to rise. This is a simple fact. It does not depend on long, drawn-out, detailed analyses of radiation balances; it does not depend on input parameters, various forcings, adjustments, volcanoes, clouds, water vapor and the like. It is simply an observed fact that an increase in CO2 does not *cause* an increase in global average temperature.

    Now, it is certainly possible for us to argue, as most are wont to do, that “Yes, there have been times when CO2 has risen and temperature has not, but that is due to clouds, volcanoes, particulates, heat hiding in the oceans, or [pick your own explanation du jour].” That may be true. It may be spot on. It may be the most accurate and well documented fact in all of science (it isn’t even close, but bear with me for a moment). But this “explanation” of why temperatures failed to rise with increasing CO2 during certain periods is essentially equivalent to saying: “Increasing CO2 causes the global average temperature to rise. Except when something else prevents it.”

    Which, dear reader, is also equivalent to saying: “Increasing CO2 would cause the global average temperature to rise if the Earth’s dynamic system were something other than what it is.”

    Again, I am not disputing here CO2’s classification as a greenhouse gas. I am not questioning CO2’s ability to absorb and re-emit radiation. For the moment, I am not even arguing against laboratory experiments demonstrating that a temperature rise occurs with increasing CO2 under lab conditions.

    I am simply focused on the basic, observable, empirical, logical point that increasing CO2 cannot be said to “cause” the global average temperature to rise. At most what can ever be said is the following: “CO2 is a contributing factor toward an increase in the global average temperature during periods in which we have additional contributing factors a, b, and c, and in which we don’t have countervailing factors x, y and z.”

    I haven’t spent a lot of time parsing Doug’s arguments or those similar to his. But I do know that many people who question CO2’s role in increasing the global average temperature are simply arguing that the role of CO2 is poorly understood and that, therefore, the above maximally-strong statement about CO2’s role should be softened to:

    “CO2 might be a contributing factor toward an increase in the global average temperature during periods in which we have additional contributing factors a, b, and c, and in which we don’t have countervailing factors x, y and z.”

    CO2 “might be,” instead of “is” a contributing factor. That is all; that is the dispute; that is the disagreement — at least for many people (again, setting aside Doug’s specific arguments for the moment).

    This ‘might-be-versus-is’ approach deserves scrutiny and may even be worthy of vociferous debate. But taking the “might be” approach, given our current state of knowledge, is hardly an outrageous tentative position for someone to take.

    1. The ‘increase’ in global temp vs what it would have been is irrefutable.

      Were it going to drop for other reasons that are more powerful than CO2’s influence is not something I or anyone else has knowledge of. I hope to god that CO2 is enough to stop the next ice age but I don’t believe it can come close.

      1. Jeff, thanks for your thoughts.

        However the claim that “the ‘increase’ in global temp vs what it would have been is irrefutable” is simply not supportable.

        Can you tell me what the global average temperature would have been over the past several years if CO2 had remained constant from 1998 on? Of course not. And no-one else can either. We are dealing with a chaotic system with numerous factors and feedbacks, some positive, some negative. We can run models all day, but they have not shown themselves to have any skill. Thus, we are left with rather general claims, based on rather general observations.

        “I hope to god that CO2 is enough to stop the next ice age but I don’t believe it can come close.”

        In other words, an increase in CO2, per your opinion, will not necessarily *cause* global temperatures to increase. That is precisely the point. People love to say “All things being equal, an increase in CO2 will cause an increase in temperatures.” Fine. But that isn’t reality. That is sterile, simplistic, lab-based talk. In the real world we know from extensive and repeated experience that all things are never equal, certainly not over any appreciable length of time. Volcanoes, clouds, cosmic rays, solar flares, variations in albedo, winds, vegetation, land use, forest fires, dust storms, plus all the long-term variations and cycles and trends, and on and on . . . The simple fact is that we have no solid idea whether global temperatures will continue to rise over, say, the next 50-100 years. No-one does. Anyone who claims otherwise is selling something. And that is true regardless of whether CO2 continues its slow steady march past 400, then 450 and onward. CO2 is at most *a* factor in global temperatures (and we cannot even be sure precisely how much of a factor; possibly just a bit player). It is most definitely not *the* cause.

        1. “Can you tell me what the global average temperature would have been over the past several years if CO2 had remained constant from 1998 on? Of course not.” – Best estimates are available but that is all they are. I’m not particularly convinced of them either.

          It is however, non-zero positive and that IS irrefutable.

        2. “That is sterile, simplistic, lab-based talk. ”

          Agreed. In fact I agree with most everything you have written, except things like – possibly just a bit player. It is absolutely a player, and that is all I’m ever saying here. Not how much or whether the Earth will get hotter than today.

          1. Jeff:

            Thanks for your thoughts. I think we’re largely on the same page and I don’t want to beat a dead horse, but I was just thinking a bit more about this topic and believe the following merits consideration:

            When we say that an increase in CO2 “must” warm the atmosphere to some degree, it seems we are saying it because CO2 is a greenhouse gas (which, by definition, we presume, means it must add some warming — however much that may be). Yet what does it mean to say that CO2 is a greenhouse gas? There are three possibilities:

            1. CO2 is a greenhouse gas because it absorbs certain wavelengths of radiation and re-emits that energy.

            This is a definition that everyone would agree with, can be verified in the lab, and is largely irrefutable.

            2. CO2 is a greenhouse gas because increasing its concentration warms the air in a closed container in laboratory conditions when convection is prevented and all other feedbacks are eliminated.

            This is a definition that, again, can be verified in the lab and presumably everyone would agree with the observation.

            3. CO2 is a greenhouse gas because increasing its concentration in Earth’s atmosphere leads to an increase in atmospheric temperature.

            In contrast to 1 and 2, this is not something that can be carried out under lab conditions. The only “experiment” we have is our actual climate system. And that climate system has not resulted in increasing atmospheric temperatures with increasing CO2 in at least two multi-year periods over the last century. Then there is the matter of the ice cores which show CO2 increase trailing temperature increase, not leading. So the only live “experiment” we have is, arguably, contradictory to #3, or at the very least inconsistent with #3 to the extent that we need to point to other factors or revise definition #3 in order to make it hold true.

            So how do we deal with this observational data that doesn’t match with #3? There are two primary logical possibilities:

            (a) We can say that in those periods in which temperatures flatlined or fell, that temperatures would have fallen even more without the increase in CO2. That is certainly possible. It might be the case. Yet, it is based, we must confess, not on our live “experiment” for #3, but rather on #1 and #2. In other words, our claim for #3 is based, not on evidence for #3, but on evidence for #1 and #2. That is at best, indirect evidence, and this is a very significant logical point. We might be convinced, we might believe in our heart of hearts that the Earth’s dynamic, open-ended, convection-driven, heavily-mixed, feedback-dominated climate system will respond to an increase in CO2 the same way a static, closed, convection-absent, non-mixed, feedback-absent laboratory system responds. And we might be right, but we need to acknowledge that (i) our argument is based, not on evidence for #3, but evidence for #1 and #2, which is at best indirect, and (ii) there is a non-zero possibility we could be wrong, and that Earth’s dynamic system might react quite differently to increases in CO2 than our static laboratory system.

            (b) We can rework definition #3 to say that “all things being equal” an increase in CO2 concentration in Earth’s atmosphere will lead to an increase in atmospheric temperature. This is a very common approach many people like to take. However, what does “all things being equal mean?” Quite frankly, it means that we ignore the dynamics of the system; we ignore feedbacks, convection, winds, changing albedo, vegetation, land use, cosmic rays and all the other things that make up the dynamic system. At this point, our new definition is quite true; indeed, everyone would agree with it. But not because it teaches us anything substantive. Rather, because we have basically redefined definition #3 to be equivalent to #2 — meaning, we would be arguing that in a static, non-dynamic, non-real-world system, increasing CO2 leads to an increase in temperature.

            So the only ways we can account for the real-world CO2 and temperature data for #3 and still keep the ‘increasing-CO2-warms-the-atmosphere’ story is either to (a) make some pretty hefty assumptions, or (b) redefine ourselves out of the real world and back into the lab.

            Again, #1 and #2 are quite true. But #3 is the one that is relevant and the one that we are interested in and we need to acknowledge that in Earth’s real-world dynamic system we are making some not-insignificant assumptions to get to that point. We could be right that increasing CO2 will necessarily warm the atmosphere. It is possible that we are spot on. But it is definitely not irrefutable.

          2. My answer to your long post is simple — yes.

            In my newer post, I spent some time renewing my understanding of oceanic heat content. It has always been the primary key to my own skepticism of AGW doom, and it is all anyone will ever need to be skeptical of the RealClimate style claims of uncontrolled warming. It just doesn’t make any sense.

    2. CO2 is a known contributing factor and when it is not recognized as one, skeptics look like they don’t understand science.

      But actually, I’ll do you one better. The ocean is beyond concerning to me. The giant body of water with an average temp of under 4C can absorb all the heat in the atmosphere with barely a measurable change in contained energy. 4C is way too cold for my warm blooded ass and it will only take a small change in thermal transfer for that tiny little black hole of energy sink to turn every bit of CO2 warming into nothing.

      People talk about asteroids, worries about AI, nuclear war, giant volcanoes, the real monster we should be worried about is that giant body of extremely cold water that dominates our planet. ‘ Course I’m not a world renown politician, I’m just a blogger.

      1. Jeff

        “the real monster we should be worried about is that giant body of extremely cold water that dominates our planet”

        Let me put some context to that comment.

        Human energy consumption is at a rate of around 17 trillion watts.
        Total geothermal energy flows from within the earth at around 44 trillion watts.
        Total energy accumulation in the oceans, measured by the ARGO system, is currently at around 250 trillion watts.
        Total energy arriving from the Sun, after allowing for albedo is around 121,826 trillion watts.

        If the energy from the sun could not be radiated out to space and all instead remained and accumulated here then result would be enough to:

        – Boil Sydney Harbour dry in 12 seconds.
        – Boil the oceans away in around 900 years
        – Melt the entire Earth’s crust in 5,000 to 10,000 years.

        Obviously that energy can escape to space. But there is still a restriction on that flow in the form of the GH effect. So I would have though that anything that involves adjusting the control valve regulating that outflow is something we would want to be very conservative about.

        That 250 trillion watts accumulating in the oceans, obviously isn’t coming from anywhere here on earth; there is no energy source big enough to supply it.

        And the world hasn’t stopped warming. There is still at least 250 trillion watts worth of warming. And if all that energy that is currently going into the oceans had all gone into the air instead, air temperatures would be rising at 15 C/decade. If the Earth were a desert world, with only very, very shallow seas and not huge oceans there would be absolutely no question mark about the impact of CO2 – its effect would be blatant and immediate.

        Are you really that confident that the oceans are the thing we should be so worried about?

  16. Not reading – just writing my daily circular – take it or leave it Jeff my boy …

    The “greenhouse gas” water vapour cools by about 12 degrees in total, a fact I have proved from empirical temperature and precipitation records from three continents and over thirty years. Carbon dioxide cools for the same reason, but only by a minuscule amount. Each reduces the gravitationally induced temperature gradient because of their radiative properties. The overall state of thermodynamic equilibrium has the environmental temperature gradient.

    Until those other than myself can explain how the necessary energy gets down to the base of the nominal troposphere of Uranus, they have no correct understanding of the energy flows on Earth. It’s all explained in my book “Why It’s Not Carbon Dioxide After All.”

    1. “Not reading …”

      You just summed your problem up SOOOO succinctly Doug!

      Without reading how will you ever learn?

      “Until those other than myself can explain how the necessary energy gets down to the base of the nominal troposphere of Uranus, ”
      SIGH

      Energy transfer through atmospheric mixing Doug. Hot air rises, cool air falls, conservation of mass is an absolutely brutal task master.
      If there is energy above and vertical mixing occurs this can transport energy lower. Fluid mechanics is the dominant mode of heat transport within most fluids.

      1. i read your first line. I’ve done my thousands of hours of reading, thanks, and of thinking. Now I’m at the teaching stage, and you and Jeff have a lot you could learn from me, because you can’t correctly explain the energy flows on Uranus and Venus, whereas I and one other author have done so..

      2. You’re actually describing the “heat creep” process, but the “mixing” is not due to wind or fluid mechanics of any form. That doesn’t happen in the Uranus troposphere where there’s no surface at the base thereof and no solar radiation down there either. The solar energy striking Uranus is absorbed only near TOA (where it’s about 60K) and then the absorbed thermal energy has to get down 350Km to the base of the troposphere where it’s 320K (hotter than Earth) and you need to understand how this non-radiative heat transfer happens up the temperature gradient as it restores thermodynamic equilibrium. You’ll read how and why in my book. ​

      3. Jeff and others:

        This is still on topic, because solar radiation (and possibly associated cosmic ray flux) is the main process that regulates what are purely natural climate cycles on Earth. Carbon dioxide sensitivity is 0.0C to one decimal place, and the reason for that can only be understood if you understand what really supplies the required thermal energy to the base of a planet’s troposphere and into any surface there. That is not radiation from a colder troposphere.

        Glenn Tamblyn wrote “Energy transfer through atmospheric mixing Doug. Hot air rises, cool air falls, conservation of mass is an absolutely brutal task master.
        If there is energy above and vertical mixing occurs this can transport energy lower. Fluid mechanics is the dominant mode of heat transport within most fluids.”

        The whole concept of fluid dynamics relates to what physicists call “forced convection” that being mass energy transfer forced by an external energy source such as wind.

        But to really understand the underlying thermodynamics we have to consider an ideal troposphere without any such wind and without a surface and with solar radiation only being absorbed near its top because such incident radiation is strongly attenuated by absorption in the atmosphere by radiating (and absorbing) molecules like water vapour, carbon dioxide, methane etc.

        The nominal troposphere of Uranus provides the best example in our Solar System of such an ideal troposphere, and its temperature is about 60K at the top and 320K at the base of that troposphere (350Km below) where there is no surface.

        We must consider the difference between night and day. At night (on the dark side) there will obviously be energy loss to space, and that is radiated from the uppermost layers of the troposphere. The whole troposphere cools by non-radiative heat transfer towards the top, and the thermal gradient in the troposphere remains the same, while the temperature profile drops a little at all altitudes. On Venus this drop is about 5 degrees.

        What actually happens during this cooling? Well there is indeed upward non-radiative heat transfer by molecular collision (and some diffusion) upwards. This causes some expansion purely because some net movement of molecules occurs upwards to less dense regions. The expansion itself is not what causes the cooling. It is the internal loss of translational kinetic energy as molecules rise and gain gravitational potential energy in lieu.

        The troposphere is “ideal” and so what happens in one location happens all over that dark hemisphere and all net molecular movement (due to kinetic energy transfers in collisions) is upwards. So the height of the troposphere must increase slightly.

        Now, by day the reverse happens. The troposphere is warmed at the top, but because the temperature gradient is the state of thermodynamic equilibrium, some thermal energy transfers downwards by non-radiative heat transfer (not forced convection) away from the new source of slightly warmed air – perhaps 61K on the sunlit side instead of 59K on the dark side, for example. The downward transfer of energy absorbed at the top is restoring thermodynamic equilibrium with its associated temperature gradient. In short, the process is the reverse of the night time process as far as non-radiative heat transfer processes are involved, and of course the height of the troposphere lowers during the day, so there is no gain or loss of mass.

        Radiation cannot transfer thermal energy from cold to hot, but non-radiative processes can do so in a gravitational field, and that is a major breakthrough in 21st century physics pertaining to our understanding of energy transfers and temperatures in planetary tropospheres, surfaces, crusts, mantles and cores throughout the Solar System and no doubt beyond.

      1. And you still can’t explain why water vapour actually cools in the real world, as confirmed by real temperature records.

      2. No he doesn’t Jeff because he does not understand thermodynamic equilibrium, and nor do you, let alone the corollaries we can deduce from such.

      3. No he doesn’t as I’ve explained in the second comment above still in moderation and likely to be deleted because you and Glenn have no valid response. The energy has to get down there, and you can’t explain how, though I have.

      4. “Read more” ??? I think the quotes from the IPCC website and the rebuking thereof in my book is sufficient. But if you can point me to a study like mine which, instead of showing water vapour cools, shows that the sensitivity to water vapour is over 10 degrees of warming per 1% then, guess what, I’ll read more.

  17. It takes a high level of knowledge and understanding in a field of research, along with a high level of intelligence to be one of the first two scientists in the world to make such a major breakthrough as we have, But when you realise that it is a whole different paradigm which in fact correctly explains the observed temperatures in the physical Solar System, you know you are right, and colleagues who listen and study what you say (instead of making their own arrogant assumptions) confirm such.

    1. And a high level of narcissism for an intellectually challenged individual like yourself to admit such knowledge.

      1. Yes Jeff, I claim a better understanding of atmospheric thermodynamics than yourself or any of your disciples (students?) writing here.

    1. So what happened to Glenn’s comment? I’ll respond if he reposts and if you don’t consider me too naughty to do so, Sir Purrrr!

  18. You could learn from these reviews, Jeff my boy, the above being by a professor of physics …

    “Doug Cotton shows how simple thermodynamic physics implies that the gravitational field of a planet will establish a thermal gradient in its atmosphere. The thermal gradient, a basic property of a planet, can be used to determine the temperatures of its atmosphere, surface and sub-surface regions. The interesting concept of “heat creep” applied to diagrams of the thermal gradient is used to explain the effect of solar radiation on the temperature of a planet. The thermal gradient shows that the observed temperatures of the Earth are determined by natural processes and not by back radiation warming from greenhouse gases. Evidence is presented to show that greenhouse gases cool the Earth and do not warm it.”

    John Turner B.Sc.;Dip.Ed.;M.Ed.(Hons);Grad.Dip.Ed.Studies (retired physics educator)

  19. I have been deleting Doug’s comments here now. He won’t stay on topic whatsoever and I’m quite tired of his ramblings. If he is responding to someone else, rather than leaving blanket nonsense as he is prone to do, I will leave it up.

    1. Patrioticduo

      You can read copies of my comments to Jeff on Roy Spencer’s thread. Feel welcome to discuss the physics with me there.

    2. What moderator on any other thread dictates the line of discussion and restricts comments to those that answer his questions and play along with his line of argument (which is the IPCC’s anyway about higher radiating levels which most of us know about and, in my case, can pinpoint the faults with) in the fashion that young Jeffrey Conlon does as he “teaches” his grandmother to suck eggs?

      [REPLY: The name is Condon. You can’t even read accurately enough to get that right]

      1. Sorry about the typo, Jeff. I have typed your name correctly many times when referring to you on other climate blogs and I’ll make sure I get it right on my new website next year which will be responding to all the false physics posted on WUWT, SkS, Judith Curry’s, Roy Spencer’s and your blog also if it continues pushing the IPCC “fissics” about increased radiating altitudes somehow supplying extra thermal energy to the warmer surface from the electro-magnetic energy in backradiation, which in fact is all used up as part of the surface’s quota of outward radiation as per its Planck function. This happens because electrons are raised between quantum energy levels, but immediately re-emit an identical photon without ever having that EM energy converted to thermal (kinetic) energy in molecules. And this is indeed due to resonance because the surface can emit such identical photons, just as Prof Claes Johnson pointed out.

        In a previous comment on the other thread you showed some belief in the fact that gravity forms a temperature gradient, but you have not yet realised the world-shattering consequences of such.

        Perhaps this explanation of what is of course in more detail in my book will help you and your readers, and it gives the explanation as to why more moist regions on Earth have lower (not higher) mean daily maximum and minimum temperatures than do much drier regions at similar latitudes and altitudes.

        People (well climatologists anyway) don’t realise that the Second Law is about all forms of energy, not just the kinetic energy in molecules which gives them their temperature. The Second Law says thermodynamic equilibrium (not just thermal equilibrium) will evolve, and that includes mechanical equilibrium.

        So, when a density gradient forms in a gravitational field that is an example of the Second Law in operation, and when a lake levels out again after rain falls on a small section of it, that also is the Second Law operating as entropy approaches the maximum level that is accessible within the constraints of the system.

        I believe Josef Loschmidt (a brilliant physicist from the 19th century) realised this about the Second Law, and he deduced correctly that we need to consider the gravitational potential energy in molecules when determining the maximum entropy state, that is the state of thermodynamic equilibrium. This leads to the inevitable conclusion that we only eliminate unbalanced energy potentials (maximising entropy) when the sum of molecular kinetic energy and gravitational potential energy is homogeneous. This means that there is always a propensity to form a temperature gradient in a gravitational field. However, radiating molecules do radiate to each other and have a temperature levelling effect that means that the overall state of thermodynamic equilibrium (taking this radiation into account) has a less steep gradient. In water it is almost eliminated, but not in Earth’s outer crust for example.

        Now, the important point is that, when thermodynamic equilibrium is attained (being the same as what climatologists like to call hydrostatic equilibrium, even though they don’t really understand why it is equilibrium at all) then all net non-radiative heat transfer stops. If we add thermal energy at the bottom (for example, the Sun warms the Earth’s surface after dawn) then non-radiative heat flow starts again with net upward transfers. But if instead, we add new thermal energy at the top, as at dawn on Venus and Uranus, then new energy transfer starts again with a net downward direction, this being what I call “heat creep” up the temperature gradient.

        And this is what really supplies the necessary thermal energy to supplement the Sun’s energy entering Earth’s surface, not back radiation which cannot transfer thermal energy from a colder troposphere to a warmer surface.

      2. PS:

        As for the IPCC’s radiating altitude (and supposed positive feedback from water vapour) it should be obvious that, because more than half the water vapour congregates below an altitude of 4Km, adding more water vapour lowers the mean radiating altitude.

    3. So do I have right of reply or don’t I Jeff my boy. You don’t have to get tired. You could leave it to your followers to raise issues with me, like jimc below.

      Your response will be on my future website that will be well publicised with some of the multi-million dollar profits from my various businesses over the years …

      My reply to this smarty on Roy Spencer’s thread is relevant …

      jimc is a beggar for punishment:

      I have shown with valid proof spanning three chapters of my book and supported by evidence that water vapour cools in a comprehensive study in the Appendix of that book, that the process described in the Second Law of Thermodynamics dictates that there is a propensity to maximise entropy and, in so doing, to reach the state of thermodynamic equilibrium. This is how the density gradient forms and the temperature gradient forms at the same time, as has also been proven in over 800 experiments this century using well insulated sealed cylinders rather like your box.

      As I requested, if you wish to make assertive statements like yours above, then provide proof that the state of thermodynamic equilibrium can exist in a force field and yet have isothermal conditions and thus unbalanced energy potentials which of course would not be the state of maximum entropy. In other words, what you propose would need the Second Law not to work and would need homogeneous density as well.

      And, by the way, a Ranque Hilsch vortex tube does not have isothermal conditions in a centrifugal force field – not by a long shot. Its temperature gradient is also the acceleration due to the force field divided by the weighted mean specific heat of the gases passing through it. Funny how the Second Law is valid everywhere, now isn’t it jimc?

      “The law that entropy always increases holds, I think, the supreme position among the laws of Nature. If someone points out to you that your pet theory of the universe is in disagreement with Maxwell’s equations — then so much the worse for Maxwell’s equations. If it is found to be contradicted by observation — well, these experimentalists do bungle things sometimes. But if your theory is found to be against the second law of thermodynamics I can give you no hope; there is nothing for it but to collapse in deepest humiliation.“

      —Sir Arthur Stanley Eddington, The Nature of the Physical World (1927)

  20. Jeff I’ve emailed Glenn and also copied my reply to Roy Spencer’s thread which I feel obliged to do as I’m not going to have my time wasted by your deletion of comments. Why is it that you feel you can’t allow on topic debate between me and other readers here? Why do you feel you have to control all comments and restrict such to answering your questions that are just progressing along the standard IPCC explanation – as if I didn’t know that (LOL) – and thus not thinking outside the square, as you need to do. You could learn a lot from my response to Glenn, but I predict you won’t try to do so.

    1. Doug & sock puppets 10:23pm: “Why is it that you feel you can’t allow on topic debate between me and other readers here?”

      because IMO Doug 6:23am writes that he does:

      “…claim a better understanding of atmospheric thermodynamics than yourself or any of your disciples (students?) writing here….”

      So why publish this debate? 6:23am will always be the same pointless return debate from Doug. And is. Renders debate discussion worthless; wastes space & time in threads. Doug NEVER gives a credible cite to support his conjecture by any author other than himself.

      1. I remember asking you Ball4 many months ago about how the energy gets down to the base of the Venus and Uranus tropospheres, but you still can’t answer, can you. Nor can you explain why empirical data on Earth proves water vapour cools.

      2. Doug 12:08am – Provides the perfect examples of my 3:39pm comment’s correctness.

        I had replied with papers showing Doug or sock puppet how energy gets down to the base of the Venus and Uranus troposphere. I did the work to look them up and read them. Doug didn’t. Doug simply replied they were wrong without reading them or providing cite or experiment to show a well founded argument why they were wrong. Again, Doug invoked the “…claim a better understanding of atmospheric thermodynamics than yourself or any of your disciples (students?) writing here….” worthless response.

        “Nor can you explain why empirical data on Earth proves water vapour cools.”

        I did the work to cite text and page number (as again in this thread 8:26am) of the correct empirical data reduction process & exactly where Doug makes mistakes in the calculations. Doug once again invoked the “…claim a better understanding of atmospheric thermodynamics than yourself or any of your disciples (students?) writing here….” worthless response.

        It doesn’t matter what any poster tries to discuss with Doug, they always get the ““…claim a better understanding of atmospheric thermodynamics than yourself or any of your disciples (students?) writing here….” worthless response.

        There is IMO no benefit to publishing Doug’s “…claim a better understanding of atmospheric thermodynamics than yourself or any of your disciples (students?) writing here….” worthless responses. The proprietor trying to put science advancement into the discussions is correctly editing out Doug’s worthless (spam) comments to the benefit of all commenters.

        More blog proprietors should do so but it takes time and I observe the wary commenters do not fall into Doug’s trap anymore.

        1. My belief is that I have given Doug more interactive time than any person in blogland. I’ve attempted serious discussion while being properly skeptical. His theory and his ability to discuss his theory, are beyond lacking. They don’t even exist in a functional form.

          I am sorry that I subjected readers to it but it was a distraction for me. There is something macabre about the human psyche that is beyond my understanding. When feelings and belief overwhelm logic, bad things regularly happen.

  21. Doug,

    I moved your spam flags to a different section of the comment filter in wordpress and didn’t see your emails collecting in a pile there. I have released the ones written on this thread but prefer to contain your unphysical theory to this space as it detracts from reasonable discussion.

    I may read some of them later but haven’t taken the time yet.

    1. Yes well you can’t prove it’s unphysical: that’s just your guesswork because you don’t even know what it is in my book. Nor can you explain with your false radiative forcing physics why it’s so hot at the base of the Uranus troposphere (because there’s no solar radiation down there) or why water vapour cools. Anyone who falls for James Hansen’s nonsense non-science is very gullible, Jeff.

        1. I proved the greenhouse radiative forcing conjecture “unphysical” by showing real world temperature data confirms water vapour cools. As you add more water vapour more of it congregates below the “radiating altitude” (as does more carbon dioxide) because of the density gradient. So that altitude is lowered and furthermore we all know that water vapour reduces the temperature gradient. You have produced no empirical evidence to support the GH conjecture. You have not proved water vapour warms or produced any valid SBL calculations that gel with the observed surface temperatures on Earth or Venus and so you don’t come anywhere near qualifying for the $5,000 reward, because the first requirement is a study with similar methodology to mine but showing water vapour warms, and warms to the huge extent that would be necessary if it were in fact causing most of the claimed “33 degrees” of warming as in IPCC documentation on their website. Critical to the IPCC “explanation” is the addition of back radiative flux to solar flux when doing SBL calculations for Earth’s surface temperature. The combined requirement for 288K (using emissivity 0.95) would be a mean of 370W/m^2 of thermal energy transferring out of the base of the atmosphere and into the surface. But, after 30% of solar radiation is reflected by the atmosphere, the mean solar radiation entering the atmosphere is about (0.7 x 1360)/4 = 238W/m^2. So how does the atmosphere add 55% to the incident energy and deliver more into the surface?

          1. In going from zero carbon dioxide to 0.04% carbon dioxide the radiating altitude increases to perhaps 4Km. In going from there to 100% carbon dioxide the absorption of incident solar radiation by carbon dioxide in the 2.1 micron band and inter-molecular radiation causes significant surface cooling because far less solar radiation reaches the surface and the temperature gradient is reduced, just as we see in going from 1% to 4% water vapour.

            Now you have several questions of mine to answer and empirical evidence contrary to mine to produce. I have proved water vapour cools in the real world. What have you proved with any similar study? Oh, you mean to say you haven’t done one!

          2. I have no idea how you keep applying numbers to non-numeric problems or changing the problem to percentages but no you haven’t finished yet.
            Since the N2 atmosphere had a zero emission altitude and both emit at the same temperature and they both have a decreasing temperature with altitude , you must now admit that the CO2 atmosphere has caused surface warming.

            THE most basic geometry Doug or rewrite radiative physics. Can you admit it?

          3. I have shown in this comment that (even if the emissivity were still as high, namely 0.95) for a rocky planet the mean temperature with an N2 atmosphere would be 282K which is close to what it is in the real world. If the emissivity were less than 0.88 (which it very well could be for rock) then the mean temperature would be 14C which is what the IPCC tells us it is now. So there would be no scope for any warming. You would have to prove to me that the emissivity of a rocky planet without water or any plant life would have emissivity greater than 0.88 for there to be any scope for any warming what-so-ever between your N2 atmosphere and the current real world atmosphere. If you are going to argue with my Stefan-Bolyzmann calculations then produce your own.

          4. You are assuming carbon dioxide emits with the same intensity as the surface with the same broad spectrum of frequencies filling in all the space in the area under the Planck curve. It does no such thing and does not act like a blackbody. Hence the radiating temperature that a blackbody would be is irrelevant. And no, as I have shown, the actual temperature of the surface depends on its emissivity. Surely you are not claiming that the Stafan-Boltzmann equation is “unphysical” also.

            You are ignoring the fact that the real world obviously …

            (1) has a higher emissivity at its surface due to water, grass etc, meaning its temperature would be lower than a rocky planet emitting the same flux, and that lower temperature is directly related to the existence of water and rain and also vegetation that is supported by carbon dioxide.

            (2) has an atmosphere which absorbs and reflects 52% of incident radiation, so that the Sun’s radiation cannot possibly raise its surface to anywhere near the observed temperature.

            All your radiation calculations go out the window in the real Earth, the real Venus and the real Uranus Jeff. If it were 163W/m^2 of direct solar radiation that were determining Earth’s surface temperature, it would be a mean of 236K. And don’t reply with crap about back radiation helping the Sun to warm the surface each morning, when the only effect of all radiation between the warmer surface and the colder troposphere is an outward flow of thermal energy day and night.

            For planets with significant atmospheres, radiation to the surface is not the primary determinant of the temperature at the base of the troposphere or any surface: the gravitationally induced temperature gradient is.

          5. “You are assuming carbon dioxide emits with the same intensity as the surface with the same broad spectrum of frequencies filling in all the space in the area under the Planck curve. It does no such thing and does not act like a blackbody. Hence the radiating temperature that a blackbody would be is irrelevant”

            It is less efficient than rock meaning even higher emission temperatures in the balanced environment of the assumption. I can’t believe it takes you over a week of effing thinking to figure out the most basic facts. It’s like you are a special ed student. Leave the numbers out, recognize the gradient, realize you have proven beyond any doubt that CO2 has caused warming.

            All the while stomping around about your “superior” knowledge. What a joke.

          6. Do you get it yet, Jeff? There is no warming to be done between your nitrogen atmosphere and the real world if the emissivity of a rocky surface is 0.88. (For that emissivity and solar flux of 340W/m^2 without reflection or absorption we get 287.3K.) If emissivity is less than 0.88, there is cooling to be done. From this table the emissivity of basalt is 0.72, clay 0.39, shale 0.69, gravel 0.28, soil 0.38, sand 0.76, You’re going to be hard pressed to prove a weighted mean above 0.88.

            [REPLY: still no numbers in our basic thought problem. You don’t even have an actual “theory” do you? — Jeff]

          7. For goodness sake, Jeff my boy. The emissivity of polished silver is 0.01. Does that mean if you place it in, say, 300W/m^2 of direct sunlight it reaches a temperature that a blackbody would, namely 852K? Lesson 1: to work out the temperature you need to know how much flux is actually absorbed. Polished silver reflects 99% and absorbs 1%. Carbon dioxide also probably absorbs something like 1% because of its limited spectral lines, but whatever the figure is, it is nothing like the 100% that a blackbody would. A rocky planet is much more like a black body, in fact what we call a grey body, or gray body for you.

            On reflection I was wrong about 0.04% carbon dioxide raising the weighted mean radiating altitude to 4Km. It might be more like 4 inches. It is water vapour that plays the major role in raising the median radiating altitude to about 3.5Km to 4Km by my calculations. If 0.04% carbon dioxide raises it a further 4 inches (or whatever) then any warming effect is probably negated by the cooling effect of a lower temperature gradient and more green vegetation increasing emissivity and more water vapour which we know from real data causes cooling and thus negative feedback. The net effect of all the 0.04% carbon dioxide is ±0.1C of warming or cooling at the most. More than that we cannot prove.

            [REPLY, the planet does the absorbing which the CO2 doesn’t. The CO2 does a portion or all of the emitting. I feel little bad for talking you into disproving your own theory but with your constant nonsense on nearly every blog I read……My Id made me do it! ]

          8. I’m really not interested in arguing with someone who fails to realise that empirical data proves him wrong. Water vapour cools – real data proves it. That single fact wipes out the GH conjecture.

            There never has been any need for the IPCC’s “33 degrees” of warming, because a rocky planet receiving twice as much solar radiation at its surface would have been hotter in the first place than the real world is now.

            My hypothesis, of which you have absolutely no understanding because you don’t deign to read the book, does explain all temperatures on all planets and moons, and, more importantly, the required energy flows. The IPCC conjecture is that the surface would have been 33 degrees colder with an atmosphere of pure nitrogen, and I have proven that assumption, and thus their conjecture, to be totally incorrect. Those who still believe it are just plain gullible.

          9. “the planet does the absorbing which the CO2 doesn’t. The CO2 does a portion or all of the emitting.” You don’t say. At least we agree on something.

            That’s why the real surface of Earth only receives (according to NASA) 48% of the flux which the surface of the nitrogen only planet receives. And because the latter would have a rocky surface with emissivity less than 0.88 it would get hotter than the existing Earth’s surface with emissivity about 0.95, so carbon dioxide, methane, water vapour etc cause net cooling.

            [REPLY, the question is not about Earth Douglas. You keep stuffing numbers into a non-numeric problem and arguing with them.]

          10. Doug,

            You need to understand basic physics of thermodynamics before your theory can be accepted. You don’t even recognize when you contradict math. It really isn’t fair to you to play here because I’m sure you mean well and you are closer than you were.

            Now to be clear, to my understanding you have absolutely disproven your own “no” global warming theory. I have seen nothing in your discussions which suggest an “explained” deviation from radiative physics yet you regularly make critical statements which do deviate. It seems pretty obvious that you are unaware of the problem at this point. Like Loshmidt, this does not preclude you from making a good point.

            If you like, I would like to move on from AGW for a bit and talk Loschmidt because I do find that interesting. I would like to see if you have recognized any of the paradoxes it creates and have the answers I cannot find.

            Imagine two perfectly insulated gas filled thin cylinders vertically oriented in a gravitational gradient. The cylinders are joined together at the top and bottom ends but the ends are blocked with a very thin layer of aluminum which conducts heat but prevents flow of the gas in each cylinder. Because you like numbers, the cylinders are 1Km tall oriented vertically in Earth gravity. One cylinder is filled with Hydrogen having a Cp of 14.32Kj/Kg-K the other cylinder having nitrogen at 1.04 KJ/Kg-K (http://www.engineeringtoolbox.com/specific-heat-capacity-gases-d_159.html). There are equal kilograms in each cylinder – say 1000 Kg. I specify thin because the thin cylinders don’t allow much convection. So little, we can reasonably ignore it. Of course if you say we shouldn’t ignore convection, we can discuss it as well.

            IGNORE phase changes. The cylinders are in gas phase.

            So Loschmidt predicts a G/Cp gradient. I assume we can agree that Loschmidt predicts that the Nitrogen will have a higher temp at the base and a colder temp at the top. The heat transfer then is equal from N -> H at the bottom and H->N at the top. The problem is that the flux in energy continues unstopped….

            How does your theory resolve this problem?

          11. You wanted estimated figures. Sure!

            N2 rocky planet.

            Mean solar radiation reaching surface; 340W/m^2
            Estimated emissivity: 0.65
            Temperature: 310K

            100% CO2 atmosphere:

            Mean solar radiation reaching surface 280W/m^2
            Estimated emissivity: 0.65
            Temperature: 295K

            Cooling effect of 100% CO2: 15 degrees.

            Cooling effect of 0.04% CO2: 0.006 degree

          12. ” I have seen nothing in your discussions which suggest an “explained” deviation from radiative physics”

            Yes well 20W/m^2 of direct solar radiation reaching the surface of Venus (assumed emissivity 0.9) gives (by radiative physics) at temperature of 141K.

            My hypothesis is such a different paradigm pertaining to the “heat creep” process that it gives the correct temperature in the vicinity of 735K. You have no concept what-so-ever of what I have written in the book, and I’m not here to breach copyright and reproduce the book.

            ——

            Likewise the explanation of your Loschmidt paradox is in my book, I do research topics before writing about them, Jeff, and ensure that I can and do rebuke all published attempts to counter my hypothesis in what I write. You have wasted far more than an hour of my time which you could have saved by spending an hour of your time reading my book.

            Point 1: In physics, as I have explained, convection is an all-embracing term including diffusion and kinetic energy transfer by molecular collision – even right through your aluminium panels. There is most certainly a redistribution of kinetic energy in a sealed insulated cylinder, as shown in over 800 experiments this century which you should know about.

            Point 2: You have described a single combined system which the Second Law of Thermodynamics tells us will only have increasing entropy until it reaches its (new) state of thermodynamic equilibrium for the combined system.

            Point 3: If you understood thermodynamic equilibrium you would understand that there are no unbalanced energy potentials in that state. Hence what you postulate regarding endless circulation of energy is not what the Second Law indicates will happen, and thus ” if your theory is found to be against the second law of thermodynamics I can give you no hope; there is nothing for it but to collapse in deepest humiliation. —Sir Arthur Stanley Eddington, The Nature of the Physical World (1927)

          13. Doug, your first reply is complete nonsense as it contradicts the higher emission altitude. If you were to even do the most basic calculation for radiative emission for what you have written, you would find the same.

            If you don’t want to play with Loschmidt, then I am fine with that.

          14. I did “basic calculations” using the Stefan Boltzmann calculator you can easily find on line. Give me your calculations. It’s pretty obvious (from Venus) that a carbon dioxide atmosphere absorbs a lot of incident solar radiation and hence prevents that radiation warming the surface. So, if I play along with the incorrect assumption that the gravitationally induced temperature gradient does not exist, then I get ridiculous results for the surface of Venus and close to absolute zero for the temperature at the base of the nominal troposphere of Uranus. That surely is evidence that the gravito-thermal effect is a reality. As I have said many times, using SBL an asphalt paved Earth (emissivity 0.93) receiving a mean of 163W/m^2 of solar radiation (as per NASA) would have mean temperature 235.79K. Because this is obviously wrong, the whole concept that we can worked out planetary surface temperatures with radiative physics alone is wrong. I’ll leave it to you to think about why you can’t just assume that increasing the radiative altitude (by perhaps 1 metre with 0.04% carbon dioxide) does not necessarily cause the surface temperature to rise. Even if there were water on Earth, removing all the carbon dioxide would eliminate vegetation thus reducing emissivity and increasing temperature.

          15. In summary, Jeff, however you look at it, the impact of 0.04% of carbon dioxide being doubled to 0.08% is infinitesimal, probably being less than ±0.1 degree and maybe less than ±0.01 degree of warming or cooling. You can put your own figures into this calculation:

            Start with the real world and assume there is 2% water vapour and 0.04% carbon dioxide. Assume the radiating altitude is 4.5Km and temperature gradient 7C/Km. Imagine replacing the 98% of other air molecules with CO2. The troposphere is a mean of 11Km high. It is unlikely that the radiating altitude would rise above 7Km. So multiplying the CO2 concentration by about 2,500 raises the radiating altitude 2.5Km. So just doubling it raises that altitude a mere 1 metre. Applying the temperature gradient, that 1 metre represents 0.007 degree of warming. But there is a cooling effect because carbon dioxide absorbs some incident solar radiation in which the 2.1 micron photons have about 5 times the energy of the 10 micron ones coming up from the surface. There is also a cooling effect due to the gradient being reduced by inter-molecular radiation. And there is a cooling effect due to greater expanse of vegetation which, in general has higher emissivity than soil and rock.

    2. Yes Jeff. but you know it’s not particularly scientific just to make assertive statements about some argument being “unphysical” and that kind of comment, (like Ball4’s) is just water off a duck’s back to me. The reason I know I’m right is because empirical evidence wherever I look confirms it. Every planetary troposphere exhibits the expected temperature gradient and water vapour on Earth consistently leads to cooler temperatures. If you think you can prove me wrong you need to start with empirical evidence and a satisfactory explanation of the thermal energy flows and temperatures even on other planets, because physics is universal. I can do that for data even from below the Earth’s surface, as well as Uranus, Venus etc.

      Regarding the debate back in 2011, the Second Law relates only to independent processes. There may be “participating systems” but the implication is that these are dependent. The Second Law tells us, for example, that water will flow up one side of a siphon if (in a dependent participating system) it flows further down the other side. Likewise we can argue about heat transfers, but it is rare for there to be dependent heat flows in different directions if there are no physical constraints, such as a bent tube. This is why Claes Johnson is right and my March 2012 paper is also, explaining the mechanism by which even one-way radiation obeys the Second Law and so back radiation does not deliver thermal energy into a warmer surface helping the Sun to raise the temperature higher. Hence there is no valid explanation using radiation alone for planetary surface temperatures where there is a significant atmosphere.

    3. Two simple SBL calculations show you that there must be other energy flows into Earth’s surface in the real world. In your nitrogen only atmosphere, let’s say the emissivity is still 0.95 (although I have suggested a rocky planet would be more like 0.8) and the surface receives a mean of a quarter of the 1360 W/m^2 at TOA. SBL gives a temperature for 340W/m^2 of nearly 282K. Now introduce the real atmosphere and, as per NASA, the surface receives only 48% of the incident radiation, so it receives 163W/m^2 and the temperature by SBL (using emissivity 0.95 again) is far colder, namely 236K.

      1. Co2 has a lower emissivity than the surface of the planet. It is a bit complicated in that you need to look at Kirchoffs law and recognize that emsisivity and absorbtivity are related to the spectral wavelengths in question, so I expect you to miss that point. If we assume a planet has a 0.95 emissivity, CO2 will have a far lower emissivity in the temperature ranges you define.

        Per Stephan Boltzman, in order to maintain the same energy output (the planet is in energy balance) a lower emissivity = higher temperature and that is why you have proven global warming exists Doug.

        You did it yourself but don’t realize it yet.

        1. A gas is not a blackbody. If you used the very low emissivity of CO2 to calculate the temperature at the top of the Venus troposphere you would get a result far, far hotter than reality.

          1. It doesn’t need to be a blackbody, it just needs to have a lower emissivity than the ground over the wavelengths being considered … which it does.

            We can talk about pressure broadening of wavelenghts a long time after you recognize the simple stuff. You proved CO2 warming with your own words, what are you going to do about it?

  22. Maybe this will help people understand. When we have thermodynamic equilibrium we have a temperature gradient in which, if we consider two horizontal planes of molecules separated by the mean free path, then the difference in mean KE equals the difference in gravitational PE. When a molecule with mean PE and KE at the top level moves downwards between the layers, by the time it collides with one on the lower level its KE has increased and its PE decreased so as to match one with mean KE and PE at the lower level. Hence when they collide they have the same KE and so the combined KE after the (assumed elastic) collision is the same as before, causing no further warming or cooling. The opposite happens for molecules moving upwards between the two planes. Hence we have thermodynamic equilibrium. And this also demonstrates why the density gradient is not altered once that same state of thermodynamic equilibrium evolves, simply because there’s no further redistribution of KE during collisions.

  23. There has been much speculation about whether a temperature gradient will spontaneously develop in an isolated column of gas. You can decide for yourself at this website:

    http://physics.weber.edu/schroeder/md/InteractiveMD.html

    The website runs a molecular dynamics simulation* of gas molecules in a 2-D box with and without gravity. The molecules are colored according to their kinetic energy. In the present of gravity, a density gradient develops. At the same time, one can see that the frequency of collisions with the wall (ie pressure) is greater near the bottom than the top. When the Temperature is about 3 and Gravity is 0.1, there is a nice distribution of colors. 300 Atoms in the largest box (Volume = 10000) is a good place to start. Time step 0.003. 25 Steps per frame.

    The temperature is raised by clicking on the faster and slower buttons (or more gradually with the -1% and +1% buttons). When you turn on or up the gravity, molecules begin falling – which increases their kinetic energy and temperature. If you increase the temperature using “Faster”, the temperature goes up immediately and then falls off as some kinetic energy is converted to potential energy.

    Skeptics can watch one atom bouncing inside the box as expected.

    From Wikipedia: “Molecular dynamics (MD) is a computer simulation of physical movements of atoms and molecules in the context of N-body simulation. The atoms and molecules are allowed to interact for a period of time, giving a view of the motion of the atoms. In the most common version, the trajectories of atoms and molecules are determined by numerically solving the Newton’s equations of motion for a system of interacting particles, where forces between the particles and potential energy are defined by molecular mechanics force fields.

    1. It’s not talking about the gravitationally induced gradient. It reads “This web app simulates the dynamics of simple atoms and molecules in a two-dimensional universe. The force between the atoms is weakly attractive at short distances, but strongly repulsive when they touch.” Sure, but these forces cancel out and do not affect the mean kinetic energy at any given altitude. Read the assumptions in “Kinetic Theory” in the Wikipedia item. The Ideal Gas Law (derived from Kinetic Theory) assumes elastic collisions with large distances between molecules relative to the size of the molecules. To be sure the force of gravity is much less than intermolecular forces at close range, but it applies over a far greater distance, namely the mean free path. The gravitationally induced temperature gradient is evident in all planetary tropospheres and its magnitude gels with expectations.

      If you assume the gravitationally induced temperature gradient does not exist then you have no explanation for the temperatures observed, let alone the energy flows required to maintain those temperatures.

      Even the raising of the radiating altitude concept (promulgated by the IPCC as if it’s the only factor at play) implicitly accepts the gravito-thermal effect. It just assumes the surface will somehow get warmed because the temperature plot rises there, but it provides no explanation of the energy flows, which explanation you will only find in my book and nowhere else in world literature to my knowledge.

      1. Real gases in real atmospheres have a weak attraction (van der Waals force). This is the force that causes gases to condense at low temperature. Some of the preset simulations involve liquids and solids. The Lennard-Jones (12-6) potential is the standard force field for modeling van der Waals force and is the best fit to experimental data. If the hard spheres used in the Kinetic Theory of Gases actually developed a temperature gradient and the molecules in this simulation did not (which seems highly unlikely), the simulation would still be the correct answer for what happens in a real atmosphere. (A real atmosphere also involves polyatomic molecules and radiative energy transfer, but you and Jeff have agreed the thermo-graviational effect should occur in an argon atmosphere.)

        Turn on the gravity, watch the molecules fall and heat up in the bottom of the container, and then watch the resulting transient kinetic energy gradient dissipate … with your own eyes. You are watching real physics that is difficult to study in the laboratory. (Perfectly insulating walls don’t exist. Gravity can’t be turned on and off. Etc.)

        Before you complain about the simulation being 2-D, I’ll point out the Boltzmann equipartition theorem predicts (1/2)kT of kinetic energy in each degree of freedom. The molar Cp for an ideal monoatomic gas in 2 dimensions will be 2R rather than (5/2)R. If a temperature gradient equal to g/Cp developed in a 2D simulation, the gradient would be 5/4 bigger than in 3D. Since the gradient appears to be non-existent, the lack of a third dimension appears to be irrelevant.

        DC wrote: “If you assume the gravitationally induced temperature gradient does not exist then you have no explanation for the temperatures observed, let alone the energy flows required to maintain those temperatures.”

        You sound just like the IPCC. a) Since our models can’t explain global warming without the anthropogenic enhanced GHE, GHGs MUST have caused the observed warming. Since we can’t easily explain some lapse rates without the thermo-gravitational effect, the thermo-gravitational effect MUST EXIST. If any energy source (SWR or even radioactive decay) heats the bottom or lower portion of an atmosphere faster than the heat can escape as thermal IR through an optically thick atmosphere, buoyancy-driven convection and an adiabatic lapse rate will develop.

        1. Yes Frank, you don’t have to “teach” me about the existence of van der Waals forces or the equipartition theorem. You think GHG must have caused the warming, but real world temperature data can be used to prove that water vapour cools, as in my published study in my book Why It’s Not Carbon Dioxide After All.

          What staggers me is that you realise from the model that the temperature gradient does in fact evolve at the molecular level (as I also explain in the book) and yet you then claim it is due to (upward only) buoyancy driven convection. You just can’t think outside the square of climatology propaganda. Because of the process described in statements of the Second Law of Thermodynamics, gravity forms a density gradient and, simultaneously, a temperature gradient. The pressure gradient (and thus any buoyancy) is just a corollary. In the troposphere of Uranus it is blatantly obvious that the temperature profile has been built up from the 60K region at the top of the troposphere, for that is the only region in which solar radiation is absorbed. Yet it’s hotter than Earth’s surface at the base of that troposphere, 30 times further from the Sun. Once again, I have explained how the thermal energy gets down there, and you won’t read how in any published paper to my knowledge.

          So, yes, I have explained in the book, based on valid thermodynamics (and as only one of two authors to do so to my knowledge) how the additional required thermal energy gets down into planetary surfaces by non-radiative heat transfer. You cannot explain the Uranus temperatures, or the fact that the Venus surface actually rises in temperature by 5 degrees during its 4-month day by assuming back radiation from the colder atmosphere is transferring extra thermal energy into the surface in order to raise its temperature. Nor can you explain Earth’s surface temperature without counting this non-radiative heat transfer.

          1. Doug: You raised the issue of the weakly attractive force between the molecules in the simulation. If you recognized this attraction as van der Waals force, you had no reason to object to it – unless you were trying to confuse matters.

            When I first encountered the thermo-gravitational effect, I was impressed by Feynman’s argument that perpetual motion could be driven by spontaneous development of temperature gradient in an insolated column of (http://www.feynmanlectures.caltech.edu/I_40.html) and by DeWitt’s modification using two equally-tall isolated columns filled with gases with different heat capacity. Nevertheless, I sought a more direct physical explanation and wasn’t very satisfied with what I could do on my own and read at SOD and elsewhere. I posted a comment (http://scienceofdoom.com/2012/08/12/temperature-profile-in-the-atmosphere-the-lapse-rate/ April 6, 2014 1:46am) suggesting that molecular dynamics simulations could provide an experimental answer. Despite the fact that the correct answer appeared obvious (perpetual motion isn’t likely), I thought that such experiments would be useful. When the subject came up here, a new google search lead me to a 2014 paper about the above website.

            You say that: “gravity forms a density gradient and, simultaneously, a temperature gradient. The pressure gradient (and thus any buoyancy) is just a corollary.” You may have causation wrong. The weight of the gas (a force produced by gravity) creates a pressure gradient in a column of gas. The pressure gradient and local temperature determine the density at any location in a system without a fixed volume (like an atmosphere). Pressure alone can’t create kinetic energy and thereby temperature. PdV is energy; P alone is not.

            The modest amount of energy released by radioactive decay inside the earth creates buoyancy-driven convection inside the mantle, because heat can’t diffuse through the earth quickly enough. Any energy source in or below an atmosphere that is very opaque to thermal infrared can induce buoyancy-driven convection if the energy can’t escape quickly enough as radiation. On earth, net thermal IR allows only about 54 W/m2 to escape to space. The remain 100 W/m2 is carried away by convection. In my amateur opinion, the 20 W/m2 of sunlight that reaches the surface of Venus is probably more than can escape through its atmosphere without the assistance of convection. And I doubt we understand enough about Uranus to draw any reliable conclusions

    2. Sadly the model doesn’t appear to give data for the mean KE in, say, the top quarter compared with the bottom quarter. If it did we would have evidence of the temperature gradient. Of course the density gradient is obvious, and we can deduce that the temperature gradient evolves at the same time. What you can see, however, if you click DATA and introduce gravity (sliding the bar) then, each time you PAUSE, the sum of KE and PE remains close enough to the same, regardless of the mean KE. This is the condition of homogeneous (PE+KE) that I have been talking about and which directly implies there is a temperature gradient when this state of thermodynamic equilibrium evolves.

      1. I agree completely that one needs to abstract quantitative data from the simulation before a conclusive answer is available. However, if one has a temperature of about 3 and the gravity is set to 0.1, the distribution of colors in both the top and bottom is similar even though a clear density gradient has developed. If you start with a temperature of about 1.2 and zero gravity and then turn the gravity up to 0.1, the system equilibrates to a temperature of 3 because of the additional energy added to the system when every molecule acquires some gravitational potential energy. The average molecule falls, creating a density and pressure gradient, but the kinetic energy appears to be re-distributed upward by collisions.

        E appears to be the total energy of the system – it varies with the number of molecules and increases the instant gravity is turned on. You can draw bonds between atoms and E will probably include the rotational and vibrational energy. I don’t know whether these are classical or quantized (and it doesn’t matter for monoatomic gases.) T appears to be kinetic energy per molecule.

        1. If you reject the Kinetic Theory explanation of the temperature gradient (based on the Second Law process) then you might as well also reject the Ideal Gas Law which is derived from Kinetic Theory. So you’re back to Square One with no explanation of the -g/Cp gradient which is found in all planetary tropospheres, reduced a little by intermolecular-radiation which has a temperature levelling effect that must also be taken into account in determining the final state of thermodynamic equilibrium..

          1. I don’t reject the Kinetic Theory of gases or the 2LoT. However the 2LoT doesn’t apply to individual molecules – which have kinetic energy but not a temperature. A slower-moving molecule can collide with and transfer kinetic energy to a faster-moving molecule. If all collisions transferred energy to the slower moving molecule, we wouldn’t have a Boltzmann distribution of molecular speeds. A photon can also transfer energy from a slower-moving molecule in an excited state to a faster-moving molecule. However, when you talk about energy transfer between groups of molecules colliding frequently enough to have a temperature, then net energy transfer by either mechanism is always from hot to cold.

          2. Frank said: “net energy transfer by either mechanism is always from hot to cold.”

            No it’s not. You are not quoting the Second Law of Thermodynamics: you are quoting a corollary of that law which has certain prerequisites, notably the prerequisite that all forms of energy other than kinetic energy remain constant. In regard to gravitational potential energy, that happens only in a horizontal plane. Thus the corollary that you quoted applies only in a horizontal plane without phase changes or chemical reactions, and it thus irrelevant for what we are considering in a vertical plane in a planet’s troposphere, crust or mantle for example..

            Next time, Frank, save my time by reading the book first and referring to the Second Law of Thermodynamics as I have clearly done myself. The detailed proof of what I said in the paragraph above spans two chapters of the book.

            Jeff

            This really is enough irrelevant discussion that is not in any way discussing my hypothesis in the book. If you want to be the first climate blog in the world to run an appropriate post on the breakthrough complete paradigm shift in understanding planetary atmospheric, surface, crust, mantle and core temperatures then I am prepared to write a lengthy post on the content of my book and I will be able to handle each and every attempted rebuke thereof by referring to the content of the post and valid physics, pinpointing any and all errors in the physics presented by anyone, such as in this comment by Frank.

          3. Come back, Frank, Jeff or anyone when you can explain how, when virtually all the solar radiation reaching Uranus is absorbed by a methane layer high up in the atmosphere (at a temperature around 60K) the required thermal energy then gets to the base of the troposphere (where it’s hotter than Earth’s surface) and continues to maintain the temperature there despite upward radiation and upward convection especially on the dark side.

          4. Doug,

            Further examination of your question is moot if you cannot apply your theory to simple situations. Your question implies a complete lack of understanding of a blanket but again, how can we begin if YOU cannot even understand what you have written?

            A really nice temperature profile calculation left by galloping camel – http://faculty.washington.edu/dcatling/Robinson2014_0.1bar_Tropopause.pdf. I believe it answers your question anyway.

            But this thread was created to get you to EXPLAIN how your NEW AND DIFFERENT theory would act with CO2 added to the atmosphere. You accidentally? talked yourself into a position where ANY reader must conclude the surface of the CO2 planet warmed. Instead of explaining yourself, you got petulant and moved on to conclusions in far more complex situations.

            I was really disappointed that you were not even good enough to recognize your failure. The way you walked into admission of CO2 warming and didn’t even react was quite telling. You claim a complex understanding yet can’t even reach beyond high-school physics. Even that may be generous to you.

            Again, I gave you more opportunity than any other blog, and like before, you have failed miserably.

            I agree, 200 comments is enough. I am not going to read your nonsense book — ever. After 200 comments, if there is ANY rational core for your theory, it is well hidden in the muck and mush of a fuzzy mind.

          5. You continue to ignore points (2) (3) and (4) in #30 which explain why radiating gases have a net cooling effect despite raising the radiating altitude, which they don’t do anyway from the existing level..

            You continue to ignore the empirical evidence that water vapour cools. So if you applied your “logic” about radiating altitude being the sole determinant of surface temperature you get the wrong result for water vapour.

            You continue to ignore the fact that I showed that the surface could well be warmer than the existing temperature with a nitrogen atmosphere, because more solar radiation strikes it and its emissivity would be lower.

            You continue to ignore the non-radiative transfer of thermal energy from colder to warmer regions on Uranus and Venus, and on Earth. So you leave yourself with no explanation of reality, no idea as to what keeps the core of our Moon well above the maximum surface temperature and no understanding of the link between “heat creep” up the temperature profile and the gravitationally induced temperature gradient.

            Try using moist air when you build your next mansion with double glazed window panes. After all, you think water vapour acts like a blanket as you were taught in elementary school I suspect.

          6. See Doug, it doesn’t matter that water vapor cools in your mind, because there is NO WATER on the planets of our example. I don’t “ignore” the rest because the example is what the question was.

            You proved warming in the example. Until you realize the basics, how can we discuss something more complex?

          7. What I proved from real world temperature data is that water vapour cools, and that has to do with its radiation reducing the gravitationally-induced temperature gradient. That’s is why the surface must be warmer without water on the planet, probably being close to 300K.

            The Second Law of Thermodynamics is well within the scope of a Science degree majoring in physics, but oh how poignant was this comment in March 2012 ..

            Because the import of the consequence of the radial temperature gradient created by pressurizing a spherical body of gas by gravity, from the inside only, is that it obviates the need for concern over GHG’s. And, because this is based on long established fundamental principles that were apparently forgotten or never learned by many PhD’s, it is not something that can be left as an acceptable disagreement.” [source]

  24. The process described in statements of the Second Law of Thermodynamics (namely that entropy increases to a maximum wherein the state of thermodynamic equilibrium is attained) is precisely what forms the density gradient. I’m sure Tim Folkerts and other alarmists would agree that there is indeed a density gradient in a planet’s troposphere. The density gradient remains stable for one reason only: the molecules at the base have more mean KE and greater density.

    If you rotated a horizontal sealed insulated cylinder of gas to a vertical position, the density gradient forms and at the same time the molecules at the bottom end up with more mean KE simply because more molecules have fallen than have risen, and that’s what also forms the density gradient. A pressure gradient is merely a corollary because pressure is proportional to the product of temperature and density.

    I am posting this comment on four or five other popular climate blogs because I’m sick and tired of the false physics promulgated by this Tim Folkerts alarmist, despite the numerous times I have demonstrated the error in his thermodynamics. He, like many others, should be held to account in a court of law, and some time next year, with funds from companies pooled for class action here in Australia, I hope to organise such and make the criminals who mislead the world pay for their actions. Yes I’m angry because of the waste of money and huge loss of life involved resulting from the biggest hoax in the history of the world.

    1. I don’t think so. It is interesting that they are starting to recognize the tweaks to the data, but they probably won’t mean much more than political points. Hopefully Australia keeps heading away from the CO2 regulation nonsense but left-wingers think in terms of regulation only, anything else is uncivilized.

  25. The problem with the model is just the impression it gives regarding the relative size of the molecules compared with the mean free path. The relative distance over which the van der Waals forces have any significant effect is very, very much less than the mean free path of air molecules in Earth’s troposphere. So, at any instant, relatively few molecules are being affected by these forces, and even where they are, the overall net effect tends to average out. So, when we are determining whether or not there are unbalanced energy potentials, for the vast majority of the molecules we can ignore the van der Waals forces (assuming no phase changes) and consider just the kinetic energy and gravitational potential energy. Unless the mean sum of these two is homogeneous we don’t have thermodynamic equilibrium, as can be understood from the following thought experiment from my book : …

    Imagine there are two horizontal planes of molecules in which every molecule in each plane has the same KE, being less in the top plane and more in the bottom plane. Obviously all the molecules in the top plane have more gravitational PE than those in the bottom plane. Assume the distance between the planes is the mean free path, so it is reasonable to imagine one molecule moving vertically downwards and striking a molecule in the bottom plane. If and only if the mean sum of (KE+PE) is the same in each plane, then, by the time the molecule collides with that in the bottom plane it has gained just the right amount of KE so as to match the KE in the molecule in the bottom plane. The reverse happens with an upward passage of some other molecule from the bottom plane to the top plane. In other words, all collisions everywhere are such that the molecules have matching KE when they collide. It is not hard to understand why this is then the state of maximum entropy, because there is no further re-distribution of KE happening and no unbalanced energy potentials. So this state of thermodynamic equilibrium has a temperature gradient and we calculate it in two lines by equating PE change (M.g.dH) with KE change, that being the energy required to raise the temperature (M.Cp.dT) giving the gradient dT/dH=g/Cp.

    1. Doug – you have literally proven CO2 warming and missed it. Then when it is explained to you what you did, you missed it again. Then when it is explained that you missed it twice, you ignore it.

      Imagine there is a person with an idea
      The idea is beyond anyone else
      So complex, so perfect, so simple
      It cannot be explained, it cannot be refuted
      So new it is old, so unique it is standard
      So powerful that it doesn’t do anything different
      If we have faith, it will exist!
      Even the slayers will be slayed.

      1. I’ve proved with empirical data and valid science that water vapour cools. Hence there is negative feedback countering the feint possibility that all the carbon dioxide in Earth’s atmosphere warms by about 0.01 degree.

        The slayers have been slayed. Postma’s radiative physics, Miatello’s and Pierre Latour’s is all invalid and does not explain, for example, how the warmed methane layer at the top of the Uranus troposphere (where virtually all the incident solar radiation is absorbed and it’s colder than 60K) then transfers thermal energy by non-radiative processes to the warmer regions below, even to the 5,000K small solid core (55% the mass of Earth) and the energy has been trapped there by gravity over the life of the planet. None of the slayers understand this or have any alternative explanation. They are all stumped. They can’t even understand why water vapour cools, or why the core of our Moon is so hot.

    2. Doug wrote: “The problem with the model is just the impression it gives regarding the relative size of the molecules compared with the mean free path. The relative distance over which the van der Waals forces have any significant effect is very, very much less than the mean free path of air molecules in Earth’s troposphere. So, at any instant, relatively few molecules are being affected by these forces, and even where they are, the overall net effect tends to average out.”

      Changing the size of the box or the number of molecules changes the mean free path. Raising the temperature minimizes the importance of van der Waals interactions (but makes it harder to see the spectrum of kinetic energies, a problem that would be eliminated without with the ability to calculate the variation of temperature with height.

      Doug also wrote: “Imagine there are two horizontal planes of molecules in which every molecule in each plane has the same KE, being less in the top plane and more in the bottom plane. …”

      The nice thing about molecular dynamics simulations is that you can specify such initial conditions. Put a temporary barrier between the top and bottom. Specify the temperature and gravity in each half of the simulation. Stop the simulation and remove the barrier. Watch what happens. For example, chose preset Figure 7 (expansion), a gas expanding through a small hole into a vacuum. Do more of the rapidly moving molecules find their way though the hole, leaving the temperature of the remaining gas left cooler? What is the mean kinetic energy of the molecules that have expanded into the vacuum?

      Even better, start a simulation with a volume of 1024. Crank the temperature up to about 5 and let it equilibrate. Pause, expand the size of the box to 10000 and then resume the simulation. When we expand the size of a box containing one or a few molecules, kinetic energy is conserved. When we do the same thing with a couple hundred molecules, their average kinetic energy decreases! I’m sure you understand PdV work (the answer to this dilemma), but you probably can’t explain it in terms of the physics of individual molecules. Why do a large group of molecules behave differently than a single molecule? The advantage of molecular dynamics simulations is that the laws of physics are applied on a microscopic scale and we can observe the macroscopic thermodynamic output (pressure and temperature) without having to worry about PdV work, entropy or statistical mechanics.

      So molecular dynamics, not speculation about what might be happening on Uranus, provides a method for accurately studying a possible thermo-gravitational effect. It hasn’t been done properly yet. Eventually it should be done in 3D. Maybe you will be found to be right. Maybe you will find that a large collection of colliding gas molecules doesn’t behave as you expect a few molecules to behave. Energy is transferred from slower-moving molecules to faster-moving molecules all of the time, but heat only flows from hot to cold. The emergent behavior of large numbers of molecules is probably connected with the rational that the thermo-gravitational effect would permit perpetual motion. My hypothesis is that emergent behavior develops as Boltzmann distribution kinetic energies develops and we can say that the gas has a well-defined temperature. However, well controlled experiments are far more valuable than speculative hypotheses or speculation about what happens on other planets that we can’t properly study.

  26. Jeff, for the benefit of silent readers (if any) some may not have realised that the “heat creep” process described in my book (being non-radiative heat transfer up the temperature profile towards the hotter surface) does of course only support planetary temperatures in real tropospheres that have sufficient molecules that absorb incident solar radiation. In other words, as on Venus, Uranus and the real Earth, some incident solar radiation is absorbed in the troposphere and is then dispersed in all directions so as to restore the state of thermodynamic equilibrium which you and I, and Frank now, understand forms at the molecular level in accord with the process described in statements of the Second Law of Thermodynamics, not by imaginary “parcels” or “pockets” of air that are held together by nothing.

    That is not to say that the gravito-thermal effect will always determine all planetary surface temperatures throughout the universe, because some planets may have tropospheres that are not high enough, or not absorbing enough incident solar radiation for the heat creep process to raise their surface temperatures to more than that to which the direct radiation from their star would do. That’s why we calculate the temperature for the imaginary pure nitrogen atmosphere by SBL rather than by using the gradient. In the case of pure carbon dioxide, we also get a higher temperature using direct radiation, so we use that rather than the calculation based on the gradient. There is also much more doubt as to what temperature the Solar radiation could raise a pure CO2 troposphere, but it is certainly not what you get using SBL with very low emissivity.

    There are several factors which affect Earth’s surface temperature (in addition to the solar flux, cosmic ray density and any other factors from outside the Earth’s system) namely …

    (1) the mean radiating altitude (possibly raised by about 1 metre by the 0.04% of CO2 in Earth’s atmosphere)

    (2) the temperature gradient (lowered by inter-molecular radiation between carbon dioxide molecules, thus cooling the surface

    (3) the level of incident solar radiation which determines how much above the supported minimum the mean daily maximum may be (lowered by CO2 due to absorption in the 2.1 micron band in the solar radiation)

    (4) the emissivity of the surface (increased by CO2 due to expansion of vegetation) thus cooling.

  27. Jeff – I think 200 comments on this thread is enough. Only Teofilo Echeverria and myself have written about this “heat creep” downward non-radiative heat transfer by conduction, diffusion and advection (ie by convection) so it’s not surprising that you have trouble understanding the significance of it and why it does away with any need for the false concept that thermal energy transfers from the colder atmosphere to a warmer surface, because it doesn’t – it transfers the other way and so does not help the Sun to achieve hotter temperatures.

    It’s time for you Jeff, either to read my book or continue on your merry way, totally unable to explain heat transfers on Uranus and all planets with significant atmospheres.

  28. What a laugh Jeff. Your own link to this document confirms what I have said about the temperature gradient being calculated the same way in all planetary tropospheres, but reduced by up to about a third. They also acknowledge that it is maintained by convection (just as I have said) but they gloss over the fact that the convection transfers heat towards the lower regions on the sunlit side and the other way on the dark side. I quote:

    “Tropospheric convection follows a dry adiabat adjusted by an empirical scaling factor typically around 0.6–0.9 to match an observed mean moist adiabat in each atmosphere.”

    [REPLY: They use radiation and greenhouse gasses and adiabatic convection to calculate the temperatures. Your challenge was to do it another way. There is no discussion of the Loschmidt calculation whatsoever. Interestingly, the stratospheric temperatures are also calculated. You need to read more carefully. Hell, you just need to read more. While I don’t need your money, I think you owe them a check for 5K.]

    1. (1) Quote from their document to support your contention, Jeff. What I quoted was about convection transfers, not radiation. As usual, you missed my point.

      (2) There’s no significant solar radiation in the depths of the Uranus troposphere.

      (3) My book also explains why the stratosphere temperatures are higher than that at the tropopause..

      By the way, one condition for the $5,000 reward is to produce a study of real world temperature data with methodology similar to that of my study, but which, instead of proving water vapour cools by a few degrees for each 1% concentration, shows that it warms somewhere near the extent of over 10 degrees per 1% concentration, as is implied by the IPCC radiative greenhouse conjecture. Then another condition is to be able to show with valid physics in some way different from my explanation, how the required thermal energy gets down into the warmer regions of typical planetary tropospheres where the original absorption from solar radiation was in higher cooler regions.

      1. So what are you saying, they didn’t carry the zero or didn’t use the correct number of significant figures?

  29. So, in conclusion Jeff, in that you have not proved that the emissivity of a rocky planet with a nitrogen-only atmosphere would be greater than 0.8716 (this emissivity leading to the existing mean temperature of 288K with the mean of 340W/m^2 solar radiation that would reach the surface through a transparent atmosphere) you have not proved that even all the existing water vapour and carbon dioxide would have any further warming to do.

    1. What is this number .8716 from? No — never mind Doug, I did not specify emissivity. You need to stop adding numbers as straw men to be proven wrong.

      So do you believe that if the emissivity of CO2 of our waterless example were lower than the emissivity of the rocky planet of our example, it would cause warming?

      Answer carefully, I’m three moves ahead of you.

      1. One step at a time: The number 0.8716 is what the Stefan-Boltzmann equation tells us would give the existing 288K surface temperature from the 340W/m^2 solar radiation reaching the surface through a nitrogen-only atmosphere. All I am talking about is solar radiation reaching the surface and supporting its temperature as in standard blackbody Stefan Boltzmann calculations. Such calculations work with the planet Mercury, so they would work with a transparent atmosphere on Earth. You can Google “Stefan Boltzmann calculator” and use the tutorvista one.

        Unless you can prove the emissivity would be higher, then there is no warming. If it’s lower there is cooling when we add the existing greenhouse gases. (We don’t use the Loschmidt gradient to determine surface temperatures in cases where no solar radiation is absorbed in the atmosphere – why would we?)

        Your argument falls apart if the original temperature is warmer than the existing world temperature, because that means all the existing greenhouse gases cool. Given the emissivity of various rocks, I maintain that the rocky planet without water or vegetation would indeed have lower emissivity than 0.8716 and thus be warmer than the existing temperature (more like 310K as I showed in a previous comment) so there is only cooling to explain for all the existing greenhouse gases in the real atmosphere. It’s up to you to explain why one greenhouse gas (CO2) would warm whilst the main one, water vapour has been proven to cool from real world empirical data. If you end up deducing that all the carbon dioxide in Earth’s atmosphere (0.04%) warms, then quantify your result. I’m happy to agree with warming or cooling of the order of ±0.1 degree or less. Obviously such is irrelevant and not worth spending billions of dollars on trying to alter.

        1. Doug, I’ve made no argument, I’ve only asked questions and pointed out that you have proven global warming exists. So ignoring all of the nonsense, and there is a lot of it, the question was —-

          So do you believe that if the emissivity of CO2 of our waterless example were lower than the emissivity of the rocky planet of our example, it would cause warming?

          1. The emissivity of CO2 is a next-to-useless figure when trying to calculate the temperature of any thin layer of the atmosphere through which most of the solar radiation passes, just as we can’t determine easily what temperature solar radiation mostly passing through the thin surface layer of the ocean would affect its temperature. (These layers do not act like black or grey bodies.) Yet the IPCC authors think they can do so, and not only that, they add the back radiation which, as is well known, does not penetrate more than a few nanometres into the ocean. Then they fudge the back radiation figure so they get a total that gives them 288K with emissivity 0.95 and hence a total solar and back radiation flux of 370W/m^2 of which solar flux is only 163W/m^2. That’s laughable, because their atmosphere is supposedly somehow delivering 370W/m^2 when, after albedo considerations (say, 30%) is 55% more than the total solar flux of 238W/m^2 entering at the top. So the atmosphere is a good energy generator is it?

            Now if you want to prove your 100% CO2 atmosphere makes the Earth’s surface hotter than it would be with a pure nitrogen atmosphere, then you need to calculate the surface temperature for each, because the solar radiation reaching the surface is about 340W/m^2 if it passes through N2, but I don’t know and you don’t know what it is by the time it is partly absorbed by CO2. Nor do you know what the radiating altitude would end up being, but I suggest no higher than 7Km in a region where the troposphere is 11Km high. What do you suggest? What then is your calculation for doubling CO2 from 0.04% to 0.08%. Show me your computations and input data, Jeff, or you are not talking physics as far as I’m concerned. I’ve agreed to one reason for warming, and added three other reasons for cooling. I quantified the effect as less than 0.1 degree of cooling in my book. Let’s see you quantify any warming, because it is nothing like the 1.3C I read somewhere here.

          2. First I have no idea what you are referring to — ” I’ve agreed to one reason for warming,” Since the beginning I have been trying to figure out your theory. After numerous meandering answers, like the ones above, you agreed completely with a comment which represented standard global warming theory. Then you went on like nothing had happened.

            The question is very simple…

            So do you believe that if the emissivity of CO2 of our waterless example were lower than the emissivity of the rocky planet of our example, it would cause warming?

            Let’s save you typing time — if the answer is YES — type A. If the answer is NO — type B.

            For craps sake Doug, focus!!

          3. I proved global warming existed in my March 2012 paper and quantified the long term trend as being 0.05 degree per decade as in that paper which you can read with this link. I also showed on my website back in 2011 that natural climate cycles correlate exceptionally well with the 934 year and superimposed 60 year cycles in the inverted plot of the scalar sum of the angular momentum of the Sun and all the planets (as on my earth-climate dot com) and greenhouse gases have not added any warming, there being no hockey stick as shown on my second climate website climate-change-theory dot com – so what’s your point? I proved from real data that water vapour cools, so even if you think you can prove all the existing CO2 warms by 0.1 degree, then there would be negative feedback from water vapour. Where’s your quantification, Jeff my boy? The only figure (1.3C sensitivity) that you quote is way out because it’s based on false assumptions about the starting point, namely the surface temperature with a pure nitrogen atmosphere, which I say is close to 288K if not warmer. It is not the 255K used bythe IPCC to calculate sensitivity. You for some strange reason dismiss the issue without explaining why you think you should.

            Personally I find it hard to calculate a temperature difference if I don’t know one of the two temperatures involved. /sarc

          4. If you’re still “trying to figure out my theory” you could read this review of my book …

            Most Helpful Customer Reviews

            8 of 9 people found the following review helpful

            Valid physics well supported by empirical evidence. Excellent and ground-breaking.

            By Dr Alex Hamilton on May 1, 2014

            Format: Paperback

            The fallacies in the greenhouse conjecture are exposed rigorously and backed up by a comprehensive study (in the Appendix) which compares rainfall and temperature data for locations on three continents. The study concludes convincingly that the wetter regions do indeed have lower daily maximum and minimum temperatures than dry regions at similar latitudes and altitudes. This supports the hypothesis in the book which shows that so-called “greenhouse gases” (mostly water vapor and a little carbon dioxide) do in fact reduce the lapse rate and thus lower the “supported” temperature at the surface. In other words, water vapor cools and so does carbon dioxide, the latter by only a minuscule amount.

            The book discusses how and why surface cooling slows down almost to a halt in the early pre-dawn hours as the supported temperature is approached. This slowing down process is well known, but the concept of the supporting temperature (due to a temperature gradient autonomously induced by gravity) was not understood, even though this “gravito-thermal” effect was originally proposed in the nineteenth century. Modern day physics can now be used to prove the Loschmidt effect is indeed a reality, as this book shows.

            As a physicist, I can honestly say that the physics is indeed mainstream and valid in all respects. It discusses the maximum entropy conditions that evolve as the state of thermodynamic equilibrium is approached, and then goes on to develop a real break-through hypothesis of “heat creep” which, when we consider what happens on Earth and other planets with atmospheres, we see must be the process which explains how the necessary energy gets into the surface of Venus to raise its temperature during its sunlit period. Indeed all planetary temperature data, even that below any surface, can be explained by the hypothesis in this book, which is indeed a totally new paradigm that completely demolishes the old greenhouse conjecture that was based on mistaken understanding of the laws of physics.

            I would expect “warmists” and “lukes” alike to attack the reviews of this book, but the astute reader will realise that is just their normal mode of approach to all such matters. To them science matters not – just their vocation or other pecuniary interests in maintaining the status quo. They would do well to consider the final comment in Chapter 1: “One wonders how many lives may have been saved had such funds been devoted instead to humanitarian aid.”

          5. Nope, I changed my mind.

            You have the information required to answer the question. If the CO2 has a lower emissivity than the planet surface, would it warm or cool the planet relative to the N2 version of the exact same waterless planet.

            You know the answer Doug.

            Baby steps.

          6. None of any radiation from CO2 or anything that radiates from the colder atmosphere can do anything but slow the rate of radiative cooling of the warmer surface. It cannot slow non-radiative cooling. It cannot help the Sun’s radiation raise the maximum daily temperature. In that the CO2 blocks some of the incident solar radiation, the surface temperature would be cooler than with a nitrogen atmosphere which allows the full blast of the Sun’s radiation. I can also calculate the CO2 using the gravito-thermal effect and, in this particular case, get about the same result.

            So, you still don’t seem to realise that I answered that question back on October 30th. I’ll copy my answer here, but feel free to put in your own figures …

            The previous comment read ….

            You wanted estimated figures. Sure!

            N2 rocky planet.

            Mean solar radiation reaching surface; 340W/m^2
            Estimated emissivity: 0.65
            Temperature: 310K

            100% CO2 atmosphere:

            Mean solar radiation reaching surface 280W/m^2
            Estimated emissivity: 0.65
            Temperature: 295K

            Cooling effect of 100% CO2: 15 degrees.

            Cooling effect of 0.04% CO2: 0.006 degree

          7. Jeff, you asked: “If the CO2 has a lower emissivity than the planet surface, would it warm or cool the planet relative to the N2 version of the exact same waterless planet.”

            The emissivity of CO2 has nothing to do with the temperature to which radiation heats it because it is not a black or grey body and each thin layer is almost completely transparent. But, none-the-less, the troposphere will not be warmer than the surface, so radiation between the surface and the troposphere is always cooling the surface because when radiation transfers thermal energy it always transfers from warmer to cooler objects, so the only transfer of thermal energy between the surface and the troposphere is coming out of the surface. Hence that transfer has nothing to do with the maximum temperature to which the Sun can raise the surface each clear day. The Sun does very well at that with the transparent nitrogen atmosphere, and so that surface will be around 290K to 310K by my estimates. After some considerable time (maybe years) the carbon dioxide troposphere would warm until the whole surface plus atmosphere is in radiative balance with the Sun. I would estimate the pivoting altitude to be in the vicinity of 5Km at the most for 100% carbon dioxide. (I estimate about 3.5 to 4Km for the existing atmosphere because water vapour congregates at lower altitudes. The 5Km would be a maximum for CO2 I would think, because it will still be more dense in the lower troposphere and the total mean height is only about 11Km. Hence, if it is 255K at 5Km and if the temperature gradient is about 25% less than the -g/Cp value (as it is on Venus) then the gradient would be 7.5C/Km giving a supported temperature (due to “heat creep” by non-radiative processes up the thermal plane restoring thermodynamic equilibrium) and so we have 5 x 7.5 = 37.5 degrees above 255K and thus a temperature of 292.5K. I can’t honestly say whether that would be warmer or cooler than the nitrogen atmosphere which I estimated between 290K and 310K depending on surface emissivity. However, if 100% carbon dioxide warmed from my minimum estimate of 290K by a mere 2.5 degrees, then the sensitivity for doubling the existing 0.04% to 0.08% would be about 2.5/2500 C which is 0.001C.

            No matter what realistic figures you put in you are going to end up within the range ±0.01C for sensitivity for a doubling of existing CO2. As far as I’m concerned that’s fine and I have proved that there is no radiative greenhouse effect doing anything remotely like what the IPCC claims.

          8. There’s not a word of math or physics in your last comment. Where is your calculation of sensitivity to carbon dioxide? Let’s see your assumptions, data and calculations. I’ve provided mine. You show me what one molecule of carbon dioxide in 2,500 can do to Earth’s temperature when it gets a mate and so carbon dioxide doubles in concentration to two molecules in 2.500 other air molecules that are storing 99.96% of the thermal energy in Earth’s atmosphere.

          9.  

            OF VITAL IMPORTANCE:

            All the models calculate sensitivity based on the false assumption of “33 degrees” of warming from about 255K to 288K. That claim is clearly spelled out in the glossary on the IPCC website from which I’ve quoted in my book and you can read free with the “Look Inside” feature on Amazon. I have shown that the 255K figure is false for a nitrogen only atmosphere where the surface temperature would be around 290K to 310K for a realistic emissivity range. Incredible as it is, the IPCC discussion of the 255K figure “forgets” that about twice as much solar radiation would be reaching the surface because there would be no clouds reflecting radiation back to space and no absorption of incident solar radiation. This glaring oversight, if corrected for, leaves no warming to be done because the existing mean temperature of 288K is actually cooler than that for the nitrogen atmosphere.

             

          10. Actually, I deserve a snip for that, so I will later. The problem is that you have just mixed up so much physics that it is unparsable. It isn’t my intent to mock, but I do need to move on from this conversation.

            Things I have a problem with are when you claim standard physics, yet demand the Loschmidt gradient. You fail to note that the Loschmidt gradient isn’t different by equation than the dry adiabatic lapse rate, and cannot parse the difference. You mix up radiative heat transfer bulk vs individual property, cannot understand or describe emissivity in any physically normal manner, and on top of that, you cannot stay on topic long enough to discuss any of the NUMEROUS problems your theory creates.

            By the way, theory is far too generous a word for the mush you have described here. You openly admit that your theory isn’t different than standard physics, at the same time denying back-radiation does transfer heat energy AND all the while claiming that Loschmidt’s rejected theory is the right one!! It is self contradictory.

            You admit an altitude change for emission, you recognize that emissivity determines temperature, you recognize that LOWER emissivity results in worse radiation yet you cannot admit CO2 will warm the planet of our example. It is a mess Doug. You have proven CO2 warming with your own words, yet you cannot allow yourself to recognize it.

            I need to move on from this, you can have the last word.

          11. Jeff you’re really out of your depth. If you had read the book you would realise that we need an understanding that the environmental (not the dry) temperature gradient is the state of overall thermodynamic equilibrium. Only then can we understand the downward non-radiative heat transfer I called “heat creep” and how the required energy gets into and raises the temperature of the Venus surface. Back radiation cannot raise a temperature of something hotter than its source. Fullstop.

            I have debunked the greenhouse “33 degrees of warming” (as in the IPCC website) because the 255K is totally wrong for the nitrogen only atmosphere. The incident mean radiation is 340W/m^2 and the mean emissivity of the rocky planet is certainly not 1.00 and if it is 0.85 then the temperature would be 290K and if 0.90 then the temperature would be 286K – these being just 2 degrees either side of the existing mean of 288K, so there’s nothing like 33 degrees of warming, and most probably there’s cooling because emissivity for the rocky planet is fairly sure to be less than 0.85. I’m just using Stefan Boltzmann calculations. What do you think is wrong with my input data or the calculations?

      2. Only three moves? That’s nowhere near enough against an expert chess player with a son who represented Australia in the game, and beat his grandfather (Prof Leo Arthur Cotton) who played against world champion Purdy.

        1. Wow, you play chess? That’s actually pretty cool Doug.

          Well you should be better at figuring out when you’ve lost. Three moves is more than enough when the answer is “Mate”.

  30. You continue to ignore points (2) (3) and (4) in #30 which explain why radiating gases have a net cooling effect despite raising the radiating altitude, which they don’t do anyway from the existing level..

    You continue to ignore the empirical evidence that water vapour cools. So if you applied your “logic” about radiating altitude being the sole determinant of surface temperature you get the wrong result for water vapour.

    You continue to ignore the fact that I showed that the surface could well be warmer than the existing temperature with a nitrogen atmosphere, because more solar radiation strikes it and its emissivity would be lower.

    You continue to ignore the non-radiative transfer of thermal energy from colder to warmer regions on Uranus and Venus, and on Earth. So you leave yourself with no explanation of reality, no idea as to what keeps the core of our Moon well above the maximum surface temperature and no understanding of the link between “heat creep” up the temperature profile and the gravitationally induced temperature gradient.

    Try using moist air when you build your next mansion with double glazed window panes. After all, you think water vapour acts like a blanket as you were taught in elementary school I suspect.

  31. “The moon has an iron-rich core with a radius of about 205 miles (330 km). The temperature in the core is probably about 2,420 to 2,600 F (1,327 to 1,427 C). The core heats an inner layer of molten mantle, but it’s not hot enough to warm the surface of the moon.” [source]

    Considering how quickly the surface of the Moon cools on the dark side, where does the energy come from to keep the core so hot with no apparent cooling off?

    The process whereby thermal energy is supplied to the core of the Moon requires a sound understanding of the Second Law of Thermodynamics and thermodynamic equilibrium.

    When our friend Jeff Condon can explain the Moon’s core temperature with valid physics, chances are he’s read my book.

    1. Doug, solids and liquids don’t exhibit your gravity gradient BUT you are avoiding the real story.

      Douglas Cotton believes in CO2 warming!

      1. Of course solids do. That is a necessary fact for the core of the Moon to be so hot, and something that Loschmidt explained. However in water, because of the high radiation properties, the inter-molecular radiation has a temperature levelling effect which tends to eliminate the gravitationally induced gradient. Please don’t continue to make assertive statements without any discussion of the physics. Solid have molecules which move a little and then collide. If that didn’t happen, there would be no conduction.

      2. Not net warming by CO2. Maybe in (1) below but cancelled but the other points which it seems I need to repeat …

        There are several factors which affect Earth’s surface temperature (in addition to the solar flux, cosmic ray density and any other factors from outside the Earth’s system) namely …

        (1) the mean radiating altitude (possibly raised by about 1 metre by the 0.04% of CO2 in Earth’s atmosphere) – yes warming if that was all that happened.

        (2) the temperature gradient (lowered by inter-molecular radiation between carbon dioxide molecules, thus cooling the surface

        (3) the level of incident solar radiation which determines how much above the supported minimum the mean daily maximum may be (lowered by CO2 due to absorption in the 2.1 micron band in the solar radiation and thus cooling)

        (4) the emissivity of the surface (increased by CO2 due to expansion of vegetation) thus cooling.

  32. Good news Doug! Or is it bad news? The molecular dynamics simulation has the ability to record position and velocity data at any instant. Sorry I’m so slow recognizing that the developer must have included this capability somewhere and slow at reading instructions. Would you like first crack at testing your thermo-gravitation hypothesis?

    1. I’m busy. But note that the model does not include any adjustment for inter-molecular radiation, so you should only expect to get the “dry” gradient. You would need to emulate Earth’s gravity and the gases in Earth’s atmosphere – and be able to set the height of the box to at least 100 metres in order to see about a one degree difference.

      1. One can adjust the strength of the gravitational field so that the density of the gas in the box drops as much as 10-fold between the top and bottom of the box. That models the difference between the surface and the tropopause, which is more than adequate to do the job. If there is a detectable density and pressure gradient, a temperature gradient from a thermo-gravitational effect should also be detectable.

        In this simulation, the gravity can creates a density gradient in the gas over a distance that is as short as 100 or 10 mean free paths. Near the surface of earth, the mean free path is about 1 um. So we can watch individual molecules gain and lose significant amounts of potential energy between collisions. More importantly, we can watch the thermal diffusion move heat from the bottom of the box – where it is generated from falling molecules soon after gravity is turned on – to the top of the box. Thermal diffusion (which is faster than molecular diffusion) can and does equalizes temperature everywhere without any net vertical movement (PE change) in the molecules. Heat diffuses vertically in solids without any molecules moving vertically. Since the model is only 10 to 100 mean free paths from top to bottom, the vertical rate of thermal diffusion of heat is very fast.

        If we were to “turn on gravity” in a perfectly insulated 10 km vertical column of atmosphere on earth or any other planet, the gas will fall and gain kinetic energy near the bottom. That heat WILL thermally diffuse through the column, just not as fast as in these simulations – because the heat has about 10^6 times further to go to reach the same equilibrium state as we see in these simulations. Since thermal diffusion is so slow, most things are not isothermal on large scale. Neither thermal diffusion nor radiation can dissipate the energy the surface of the earth receives quickly enough to prevent a temperature gradient from developing. Thermal diffusion can’t dissipate the heat generated inside the earth by radioactive decay inside the earth quickly enough to prevent a temperature gradient from developing – which causes buoyancy-driven convection in the mantle. There is no way to prevent thermal diffusion by collisions and it doesn’t require ANY net vertical motion. The state of thermodynamic equilibrium is isothermal for this reason. However, thermal diffusion is so slow that we rarely see things on a planetary scale that are isothermal.

        1. Frank (and Jeff):

          You cannot convince me that the whole of the 350Km high troposphere of Uranus has not reached a state that is very close to thermodynamic equilibrium, having, as it does, a temperature gradient about 95% of the -g/Cp value. Thermodynamic equilibrium cannot have unbalanced energy potentials. Extra gravitational potential energy at the top would cause unbalanced energy potentials. The Ranque Hilsch vortex tube develops very significant temperature differences in a huge centrifugal force field, and the cross-sectional temperature gradient is very close to the quotient of the acceleration due to that force field and the specific heat of the gas. So whatever your model shows, something goes wrong with it when it represents heights in the kilometres.

          You wrote “Neither thermal diffusion nor radiation can dissipate the energy the surface of the earth receives quickly enough to prevent a temperature gradient from developing.” You can’t prove that. It’s fiction.

          The temperature gradient doesn’t necessarily happen from a heated surface upwards. It can just as easily happen from a heated layer at the top of a planet’s troposphere downwards.

          You need to think about what happens on Uranus where there’s no convincingly evidence of net energy loss at TOA, no surface at the base of its troposphere and no solar radiation reaching down there through 350Km of the nominal troposphere, let alone through thousands of kilometres to the small solid core (55% the mass of Earth and at a temperature of about 5,000K) all supported by weak solar energy 30 times further from the Sun than we are. You also need to think about how the surface of Venus rises in temperature by 5 degrees in its day, and why the core of our Moon is so hot. You’ll find reasons in my book.

          Isothermal conditions in a force field are not the state of thermodynamic equilibrium because there are unbalanced energy potentials so entropy can still increase. “Heat creep” up the thermal plane, restoring thermodynamic equilibrium (with its associated temperature gradient) is a reality throughout the Universe. Planets would be just as hot with or without internal energy generation, because the energy from their stars is sufficient and gets down to their cores by heat creep. It’s fascinating, believe me.

          1. Doug

            “The temperature gradient doesn’t necessarily happen from a heated surface upwards. It can just as easily happen from a heated layer at the top of a planet’s troposphere downwards.”

            So, you have accepted the standard understanding of how the GH Effect works!! You accept itSo why are you arguing to the contrary.

            1. GH Gas concentrations determine the effective average altitude at which IR radiation escapes to space.
            ACCEPTED BY YOU

            2. Radiative balance for the planet then determines the temperature at that altitude such that the temperature at that altitude is sufficient to generate a large enough intensity of radiation to bring about radiative balance.
            ACCEPTED BY YOU

            3. The vertical temperature profile of the atmosphere then determines the temperatures of layers below and above this layer.
            ACCEPTED BY YOU

            Ergo. You accept the standard model of the GH Effect.
            SO WHY ARE YOU ARGUING THAT YOU DON’T??

            What you dispute is the mechanism that generates the vertical temperature profile and allows heat to move up and down to establish that profile. You claim it is you heat creep mechanism. The conventional understanding is that it i driven by the expansion and compression of air as it moves up and down.

            Either way, disagreeing about the mechanism that generates the vertical temperature profile is NOT RELEVANT to understanding the GH Effect! The cause of that profile is immaterial. It only that it exists that matters.

            Why are you conflating two TOTALLY DIFFERENT, COMPLETELY UNRELATED discussions?

            [REPLY: Thank you Glenn. I feel much less crazy now. — Jeff]

          2. Doug: If the atmosphere on Uranus has been around long enough to reach a state of thermodynamic equilibrium, why hasn’t the interior of the earth. The earth has an internal energy source (radioactive decay) that delivers heat faster than it can escape to the surface without temperature gradient large enough to product convection. You can’t be sure whether any temperature gradient in the atmosphere of Uranus is the result of thermodynamic equilibrium or energy fluxes. We’ve never even sent a probe into the atmosphere of Uranus. Everything we do know is based on remote sensing and models of what we think might be happening.

            I can prove that the earth’s surface (at about 390 degK) can’t dissipate all of the SWR it receives by net radiation or thermal conduction through the atmosphere. The 1 m of rain that falls in the average year represent about 80 W/m2 of heat that is removed from the earth’s surface and released high in the atmosphere. This is about half of incoming SWR absorbed by the surface. Net radiation (OLR-DLR) removes less (about 56 W/m2). The thermal conductivity of air is so low (0.024 W/m-degK) that it takes a 40 degK/m temperature gradient to drive 1 W/m2 of power through air.

          3. Well Frank, it’s not me who is saying the interior of the Earth is not pretty close to a state of thermodynamic equilibrium: that’s your idea. If there were energy being generated faster than it can escape, then the interior of the Earth would of course be getting hotter and hotter. I’ve seen no claims regarding that happening. I have said in my book that if the radioactive decay (or fission or whatever) is insufficient to maintain a constant temperature in Earth’s core, then solar energy makes up the rest that is needed, that energy getting there by the “heat creep” mechanism I explain in my book. In the same way, heat creep supports the surface temperature because the direct solar radiation is nowhere near enough after half of it has been reflected or absorbed. An asphalt paved Earth receiving 163W/m^2 at its surface would be about -35C.

            A passing spacecraft (Voyager II from memory) has measured net radiative flux at TOA for Uranus and it is extremely small with error margins that could mean it is ± anyway. Given that the small solid core (55% the mass of Earth) is about 5,000K it could easily cause a net energy loss at TOA if it were still cooling or generating energy from mass. You also need to think about Venus where the surface cools by 5 degrees in just 4 months in the Venus night. Clearly it would keep cooling if the Sun’s radiation somehow stopped and thus did not warm it back up again the next Venus day. And if the surface of Venus did cool right down, the internal temperature gradient would increase significantly and so the rate of conduction of thermal energy out of the core would increase. The existing rate of conduction does not keep the surface from cooling at night. A faster rate might slow the cooling, but it is clear that energy generation could not keep up. The same goes for Uranus and Earth and probably planets and satellite moons throughout the Universe. The very fact that the Sun’s energy does replace the energy lost in the Venus night demonstrates that it is at least supporting the surface temperature, and that surface temperature clearly affects energy flow rates between the core and the surface.

            Measurements at TOA on Earth are not sufficiently accurate to determine whether the net incident radiation is ±0.5% of the total incident radiation, but it is usually in that range. So something must be wrong with your calculations that imply a significant amount of energy is being trapped. Just because there is a temperature gradient (of the order of 25 deg/Km) in Earth’s outer 10Km of its crust does not mean that there is necessarily any outward flow, because that gradient could represent the state of thermodynamic equilibrium. Clearly each sunny morning there is an inward flow. On the Moon there is considerable inward and outward flow over the course of its ~28 day rotation. The solar radiation when the Sun is overhead is about 1365W/m^2 and yet the lunar surface copes with absorbing at that kind of rate. Hence it can also cool at that rate at night. And if the Sun did not warm the Moon again the next lunar day, its core also would eventually cool right down, certainly within a few thousand years.

            On Earth the mean temperature gradient for the state of combined non-radiative and radiative thermodynamic equilibrium is about 7 deg/Km. It does not matter what the rate of convection is in the troposphere, because it is radiation that maintains energy balance with the Sun. If a region is in such a state of thermodynamic equilibrium, as it can be on a calm night in the early pre-dawn hours, then there is no net convection up or down. If then the lower regions in some column of air are heated convection is upwards: if the upper regions are heated (as at dawn on Venus because the Sun can only warm regions cooler than about 400K) then net convection is downwards – up the temperature gradient, restoring thermodynamic equilibrium. All this is explained in my book in two complete chapters with diagrams to help you understand what is a totally new paradigm quite distinct from your current way of thinking.

          4. PS:

            What do you think would happen to Earth if the solar radiation stopped? Its surface cools on average about 10 degrees at night and warms again due to the Sun the next day. So It could cool faster than Venus. Eventually the upper atmosphere would liquefy and rain liquid nitrogen and oxygen to a very cold surface. Do you seriously think it would keep warm running on its emergency “battery” radioactive decay in the core? What would stop the core also eventually cooling down quite a lot? Well in fact gravity would trap thermal energy down there, but if the surface cooled, say, 250 degrees, so too would the core cool by roughly the same amount because the temperature gradients (steep in the crust, shallow in the mantle) would remain roughly the same, subject to variations in specific heat due to temperature changes. We know this because Uranus provides us with an example of a planet 30 times further from the Sun and receiving only about 1/900th of the solar radiation. But on Uranus gravity has trapped thermal energy under the thermal plot with its gravitationally induced temperature gradient.

          5. In reply to Glenn, the IPCC explanation claims that there is “33 degrees of warming” supposedly caused by the sum of radiation from the colder atmosphere added to the solar radiation. They think back radiation from the colder troposphere helps the Sun to raise the surface temperature to a higher maximum each day. I make no such claim, and I refute theirs.

            The IPCC claim that the surface temperature for an Earth without any water vapour, carbon dioxide, methane etc would have a surface temperature of 254K to 255K. That is incorrect. The radiation reaching the surface through a cloudless transparent atmosphere would have a mean of (1,365/4)W/m^2 and the emissivity of a rocky planet without vegetation could not possibly be higher than 0.88. Putting these figures into a Stefan Boltzmann calculator we get close to the existing mean temperature of 287K to 288K, and so there is no warming by greenhouse gases to be explained.

            However, there are energy flows to be explained in all planetary tropospheres. The IPCC does not anywhere make any claim that there is non-radiative transfer of thermal energy from cooler to warmer regions in a planet’s troposphere. If you think I am wrong, then cite the reference in their documentation. There is no such energy flow in any K-T, NASA or IPCC energy budget diagram.

            The IPCC does not make any mention of water vapour cooling because of the fact that it lowers the magnitude of the temperature gradient. Empirical evidence that I present confirms this. The IPCC claims water vapour does most of that “33 degrees of warming” and that is an incorrect statement, because it cools. So what the IPCC claims and what I claim are opposite statements.

            The models based on the IPCC conjectures attempt to calculate our planet’s surface temperature from radiative fluxes. That is not valid reasoning because the ocean surface reacts very differently to solar radiation than it does to back radiation which does not penetrate more than about 10 microns. Radiative calculations cannot explain the temperature at the base of the nominal troposphere of Uranus. In contrast when I apply my hypothesis I calculate 329K compared with Wikipedia’s 320K, and that’s near enough for me.

          6. Doug, a few points.

            1, The IPCC don’t claim, they report! Like a journalist reporting what others have said. They are a reporting agency that reports on the current state of the science. So your statements should be rephrased as ‘the science says…..’

            2. The 33 Deg C figure is used as something illustrative. You are reading more into that figure, which I have used in the past as well, than is intended. It is simply an illustrative description of what the temperature of the earth would be, ALL ELSE BEING EQUAL, if the GH Effect were not present.

            Here is a thought experiment for you Doug. What would the temperature of the Earth be if it had all the current gases that are present in the atmosphere BUT those gases didn’t absorb infrared radiation. That they absorbed and emitted in the microwave region instead for example. So the world still has clouds, liquid water, ice etc. But the underlying vibrational frequencies of gases like CO2 were different. Its only a thought experiment but that is all the 33 Deg C figure is as well – an illustrative thought experiment!

            3. You still haven’t addressed my point that you have accepted the standard definition of the mechanism of the GH Effect.

            4. “The IPCC does not anywhere make any claim that there is non-radiative transfer of thermal energy from cooler to warmer regions in a planet’s troposphere. “. And they don’t make any claims that molecules in the atmosphere are made of atoms either! Because they don’t need to ‘claim’ basic, standard science that is known by all scientists working in all related fields. The role of convection in determining the temperature of the atmosphere goes all the way back, at least, to E O Hulbert in 1931.

            5. ” They think back radiation from the colder troposphere helps the Sun to raise the surface temperature to a higher maximum each day”. No Doug, you don’t understand how the GH Effect works. Back Radiation is utterly insignificant in causing the atmosphere below the TOA altitude to be warmer. than it is at that higher altitude. This is virtually all caused by convective/advective mixing creating a lapse rate. Where Back Radiation DOES play a significant role is in the final step of transferring energy from the atmosphere IMMEDIATELY above the surface – 10’s of meters – to the actual surface itself.

            Radiative transfer is significant in transferring energy from the solid and liquid surface of the earth INTO the atmosphere, transferring energy FROM the atmosphere back to the solid and liquid surface of the earth, and transferring energy from the upper atmosphere OUT TO space. Radiative transfer is a very minor mechanism for transporting energy WITHIN the atmosphere.

            6. ” There is no such energy flow in any K-T, NASA or IPCC energy budget diagram.”. Of course there isn’t Doug. Again these are only broadly illustrative diagrams. They are showing, indicatively, energy flows between the major components of the system. They do for example show non-radiative transfers between the surface and the atmosphere – convective Thermals and Evapotranspiration. Those diagrams show inter-component flows not intra-component flows. You are hugely over-reading things into this Doug.

            7.”The IPCC does not make any mention of water vapour cooling because of the fact that it lowers the magnitude of the temperature gradient. Empirical evidence that I present confirms this. The IPCC claims water vapour does most of that “33 degrees of warming” and that is an incorrect statement, because it cools. So what the IPCC claims and what I claim are opposite statements.”.

            Again, there is that ‘claims’ furphy. But they don’t need to ‘claim’ that water has a cooling effect. That is well known and understood! That is why the observed environmental Lapse Rate is lower than the dry Adiabatic Lapse Rate. Again you are expecting them to explain basic well established science. You haven’t discovered anything new Doug. You have ‘discovered’ Meteorology 101. So they don’t need to reiterate basic science.

            What they do report on, directly contradicting your point, is the negative Lapse Rate Feedback. It has been well known for many years now that one of the feedbacks in the climate system, a negative feedback, is the effect of increased evaporation of water on the lapse rate. That the lapse rate is expected drop as the world warms due to the ‘cooling’ effect of more evaporation. Section 7.2.4 in chapter 7 of the most recent AR5 WG1 report discusses among other things the lapse rate feedback.

            What you aren’t getting Doug is that water has BOTH a cooling AND a warming effect. Because there are two unrelated mechanisms involved. Water cools via phase change and warms through its contribution to the GH effect. Just as clouds cool by reflecting sunlight AND warm by contribution to the GH Effect. So does your ’empirical evidence’ actually measure the role of water in the GH Effect? Or have you only measured on aspect of what water does?

            8. “That is not valid reasoning because the ocean surface reacts very differently to solar radiation than it does to back radiation which does not penetrate more than about 10 microns”. True and well understood but what is your point? Mixing in the ocean then mixes heat absorbed, whether right at the surface or deeper where sunlight can penetrate. The top 50-100 meters of the ocean isn’t called the Well-Mixed Layer for nothing. And IR Radiation FROM the ocean as well as evaporation originates in that thin layer as well. So heat absorbed deeper from sunlight needs to be moved via mixing into the ‘skin’ layer as well to be transported away.

            9. ” Radiative calculations cannot explain the temperature at the base of the nominal troposphere of Uranus.”. Of course they can’t. But what is your point? As I have said the GH Effect is based on convective/advective processes WITHIN an atmosphere. Radiative calculations are not applied in doing something like this.

            Really Doug, you are simply repeating the assertion that you agree with the established science on this point.So what are you arguing about?

            I repeat. Radiative exchange is primarily what is involved BETWEEN the atmosphere and the surface, Convective exchange is primarily what is involved WITHIN the atmosphere.

            Doug. You seem to be putting a lot of energy into trying to show that a theory that DOES NOT EXIST is wrong! When that actually requires no effort at all. Whereas if you actually want to make a critique of the ACTUAL theory of how the GH Effect works then you might get a different response (and perhaps reach different conclusions)

            Doug, before you go much further you really need to get a few good, graduate or higher level textbooks on all of this and study them. In detail! I have had a number of people recommend to me Ray Pierre-Humberts ‘Principles of Planetary Climate’ here https://geosci.uchicago.edu/~rtp1/PrinciplesPlanetaryClimate/

            Go do some reading Doug rather than shadow boxing with what you ‘think’ the IPCC ‘claims’. Reporters don’t claim anything Doug, they report!

          7. Glenn, the IPCC is hardly an unbiased reporting agency. Certainly we can all agree on that!

          8. Yes, well what the IPCC “report” is what politicians read and get bluffed by. Contained within their website and other documentation is the “official” explanation of the radiative greenhouse effect quoted in my book and refuted therein.

            It is impossible for “ALL ELSE” to be equal. The more water you have, the more water vapour and clouds you find in the atmosphere and the greater the planet’s albedo becomes. This of course is one of the reasons why water vapour cools. If you remove the clouds the surface receives more radiative flux from the Sun. Elementary my dear Tamblyn. That simple and obvious fact is not mentioned and assumed not to be the case in IPCC documentation.

            As I said, there is no non-radiative heat transfer shown as entering the surface on any energy budget diagrams.

            Yes Glenn, the surface temperature of 288K is calculated with Stefan Boltzmann calculations by adding the back radiation flux to the solar flux. If you only use the solar flux of 163W/m^2 you get about -35C. Back radiation does not penetrate more than 10 microns into the ocean and its electro-magnetic energy is not converted to thermal energy in that 10 micron space.

            The official explanations of the GH effect are as in IPCC documentation and associated K-T, NASA and IPCC energy budget diagrams which show back radiation into the surface and use that flux to “confirm” the existing temperature of about 287.5K. That temperature is not achieved by back radiation. Solar radiation cannot achieve it on its own either, and nor can the combination.

            Conduction between the surface and the atmosphere passes both ways (more into the surface on cloudy days and more out at night) and the energy flow into the surface is incorrectly replaced with back radiation that does not cause thermal energy to be transferred from a colder atmosphere to a warmer surface. If you tried to use similar logic for Uranus your GH conjecture crumbles.

            SUMMARY: The radiative GH documentation clearly claims that water vapour and other GH gases warm the surface by 33 degrees in total. The IPCC quotes the temperatures and that is what influences politicians. It is also the basis for the sensitivity calculations used in predictions that are then used to alarm the world. But real world data can be used to prove the opposite, namely that water vapour cools.

          9. If the whole of the energy in back radiation were converted to thermal energy in a mere 10 microns of the surface, that layer would boil and thus not be warmed above 100C. A mere 100C in such a small percentage of the ocean would be rather like pouring in a jug of boiling water. As for the mixing, there is no reason to assume that all the thermal energy moves upwards by mixing and convection from the cooler thermocline regions. Physical mixing uses up energy and does not necessarily send all energy absorbed in 20 metres back to the surface. There must be downward convection into cooler regions for at least some of that solar energy in non-polar regions, escaping later in polar regions. In any event, there is not enough solar radiation and all of it striking an asphalt paved Earth would only support a temperature of -35C. So no mixing would be involved anyway in such icy conditions with frozen oceans reflecting much of the radiation as well.

          10. Jeff

            “Glenn, the IPCC is hardly an unbiased reporting agency. Certainly we can all agree on that!”

            Depends what you mean by biased Jeff.

            Is there a bis towards downplaying some aspects of the risks of AGW. Yes. That is clearly evident when one sees the process by which the full report produced by the scientists is then ‘negotiated over’ by the national representatives to produce the final SPM. Case in point with AR5 was the removal of some key references to a maximum Carbon Budget to remain below 2 Deg C of warming. Some governments at least want to downplay AGW because it is something they don’t want to have to deal with politically or economically.

            Is there natural human bias? Of course there is. Every individual is subject to that. But individuals biases tend to cancel out when large numbers of people from diverse backgrounds, nationalities, cultures, demographic all look at the same question.And scietists are the most skeptical people in the world. It’s not like herding cats. More like herding alley-cats. When large numbers of scientists agree on something, having thrown every dart they can at it to try and burst the balloon, then the world really should sit up and take notice. What is the point of having experts if we won’t listen to them?

            Some folks, perhaps including you, seem to think there is some sort of an agenda. That scientists are dancing to someone elses tune. Actually it is the opposite. Scientists have been slowly getting louder and louder in their warnings to government about the risks they see. Next year is the 50’th anniversary of the first, then low key, official warning from the scientists about the risk increased CO2 might pose. They have spent that half century, through 2-3 generations of scientists, trying to alert the world to the problem. Getting slowly and steadily louder as the world hasn’t listened. ‘houston, we might have a problem’…’houston, we really think there might be a problem’…’Houston, we really really think you need to look into this’… ‘Houston. Are You There?’…’HOUSTON WAKE UP’
            And slowly, screaming and kicking, most governments have been dragged to a reluctant acceptance of this.

            Consider something Jeff. Governments come and go. Left-Right-Left-Right…. Different countries. But there is not one National Academy of Science anywhere in the world with a position against it. Not one peak professional body against it. Not one Defense Force – the ultimate in hard-nosed thinkers. The Pentagon accepts it – hardly surprising since they were behind much of the research in the 50’s & 60’s that laid the foundations for understanding it.

            Yet the message from the scientists hasn’t changed for decades. If there is an ‘agenda’ from government for example that is driving this then wouldn’t the message from the scientists blow with the wind -at least the winds of the electoral cycle – and supposedly what is coming from their ‘paymasters’.

            Could the scientists have an agenda? Of course they do. The TRUTH – there, some words in nice big capital letters. What do I mean by that? Scientists are motivated by a deep desire to be right. To discover something and get it right. No matter how diligent a scientist’s career was, if they routinely backed incorrect ideas they don’t get the status, kudos, the simple ego-trip. Science doesn’t pay that well as a career for the amount of work you have to put in but there is the potential real pay-off. ‘I’m the guy who discovered X’. That s their motive, their agenda. Nothing noble about it, pure self-interest like the rest of us. They do science because they like love finding out stuff – and getting credit for that. If they wanted serious money they certainly wouldn’t pick science. Nor if they wanted fame among the general public they wouldn’t pick a field that most people don’t understand, value or are interested in.

            So why wouldn’t they just try to convince people they did discover something, even if it isn’t true? History is full of the victor’s version of events, not truth. Why wouldn’t the scientists take the same tack? Because in science history isn’t written by people; its written by the Laws of Physics. You might convince a trillion people you are right but the Laws of Physics have right-of-veto; they determine history and a scientist’s reputation, not people. Convince people of something that isn’t true and your fame will be short lived and hollow – Physics will out. And all your fellow scientists will be champing at the bit to prove you wrong if they can.

            Apart from a few wishful thinkers, scientists know that just convincing people doesn’t count for much. Being right is what counts. So ideas of vast conspiracies, group-think etc just don’t wash. Only an idiot would think you can con the Laws of Physics and get away with it. And scientists aren’t idiots.

            And who are the people for whom acceptance is hardest, who are most reluctant to accept the evidence, in and out of government? Those for whom the consequences of AGW and the sorts of actions needed to address it are hardest to swallow. Just ask yourself how many of these items tick boxes for you Jeff? How many of these things resonate with you as negatives?

            – Action to tackle this requires coordinated global action. It requires cooperation between nations with the strong helping the weak.
            – It requires regulation of a whole range of activities in life.
            – It requires that within our societies we act cooperatively, working together.
            – It requires that we have sufficient trust in our fellow humans, in government, scientists etc to accept that most of them are acting decently enough most of the time.
            – It requires changes that may require economic costs
            – It means that the way we live our lives has had negative impacts. That we can’t have a completely positive, ‘we are the greatest’ view of what we have done.
            – It requires that we accept that a sense of boundlessness that has been a central part of our cultures, particularly in the ‘frontier nations’ such as the USA or Australia is ending – from here on in human society has to have values centered on limitations and boundedness.
            – That as individuals we may need to curtail some of the exercise of our freedoms for the common good.
            – That our sense of man’s dominion over the earth (perhaps coming from a religious origin) is misplaced; we are simply one part of that earth and we prosper best when we work with the limits of our role rather than against it.
            – That we may have spent our entire working lives contributing to building a society and an economy that turns out to have some significant flaws and is in need of at least some degree of redesign because currently it is dysfunctional; our prosperity has been obtained through a flawed system and we need to accept that, unwittingly, not all our actions have been good.
            – That our view that we are so insignificant a force on the earth that what we do doesn’t matter is now wrong; the growth in our numbers and technologies means we have the power to cause vast changes on the earth and we have done – some of them negative ones. Humanity is now a geological force and we must act with extreme care now or we can cause great harm, to ourselves particularly.
            – That any God we may believe in (if you happen to be religious) will not intervene; that we are capable and allowed to wreak damage upon ourselves and said God won’t lift a finger.
            – That our ideas about how to build prosperity and well being, whether for ourselves or our communities, may have been flawed. That we might have to admit we, unwittingly, have made mistakes.

            For me Jeff, none of these points (and one can imagine more than this) are difficult or problematic to accept. None of those statements say anything terrible, disturbing or deeply offensive about us; admitting we might have made mistakes or made bad judgements isn’t problematic, That’s just us being human. If we need to live life with a pollyanna, ‘we are so good’ view of ourselves then that is just a character flaw we have. In reality each of us is a mixture of strengths and weaknesses. And accepting that is perfectly alright. Quite simply, we are human. We fuck up sometimes. And not being able to admit we have fucked up is also fucking up.

            I don’t necessarily think the changes and insights needed will be achievable, but there is nothing I find problematic in principle with any of them Nothing in that lists is something I feel any compulsion to reject..

            So what about yourself Jeff? How many of those points get your hackles up?

            Here is the point. The Laws of Physics don’t care what our feelings about this are, yours or mine. The Laws of Physics are life-and-death. All our values etc are just our incidental ruminations. If our deepest values, desires and beliefs are out of sync with the Laws of Physics, then the Laws of Physics will stomp on our most treasured ideas with utter disdain.

            So who is more likely to be biased Jeff. Someone who is able to accept the implications of what AGW means, as highlighted in my list? Or someone who finds it deeply disturbing to accept that list?

            Because all those points, and probably others have absolutely ZERO relevance. The science is what it is, and our values etc don’t count for a tinker’s damn in comparison.

            No doubt you will want to disagree with what I have said. Just consider what your disagreement is in the light of how I have framed my comment.

          11. Glenn,

            Houston, Houston, we no longer see anything, the signal disappeared.

            There are the laws of physics but also many many assumptions which must be validated by observation.

            You let believe that the quantitative theory of the greenhouse effect is strong. We are very far of that. See all discussions on the origin and formation of the temperature gradient. The laws of thermodynamics are routinely trampled in the early considerations, at best, for necessary simplifications. Yourself even claim that a temperature gradient is spontaneously established without heat flow.

          12. Glenn,

            Thanks for the reply, things like this make blogging fun.

            Of course I have many problems with what you have written but your argument is reasonably well considered although it contains a significant bit of ideological ‘belief’ in your list of points. One thing we do agree on 100% is that the science is the science and this is the world that god (or chance) has given us and nobody is going to come from the sky to tweak it back into shape if we screw it up. I also agree that humans have the power to make change to the world, no big denial surprise there.

            Where your scientific argument fails however is in step 1. Climate models have failed….. Completely 100% failed by basic statistical testing. No scientist can rationally state that they have passed even a 30 year test. Yet they just did in a global worldwide report. So we know from that simple validity check, that the IPCC is a biased source of climate information – in the alarmist direction — despite your implications that disagreement with some of the most extreme IPCC conclusions represent the actual bias against science.

            It is reasonable to state that the problem with the models is worse than simply exaggerating the warming trend by 2-3 times, they are not explanatory of the difference so mathematically and scientifically, they have little to no predictive scientific value. We don’t have a prediction that we can rely on. Scientifically, we cannot use the models to predict either when or even IF we will see significant warming. Therefore no action is required, necessary, or far more importantly, —- in our best interest.

            In addition, we don’t have any evidence that actually warming the planet would present any significant problem either. Hurricanes, droughts, rainfall, etc… none of the alleged dangers have actually been observed. Really that is all that should be required for our scientific discussion. We should all agree that while CO2 does add heat to the system, we don’t know what future temperatures will do, and we can’t observe any significant problems from the bit of warming we have experienced. End of story.

            However, that is not what the IPCC is promoting.

            IF we still irrationally assume that CO2 IS going to cause significant warming right now, and it IS irrational to assume such as we have no evidence for it, the way to combat CO2 warming is something that is far grayer an area than you have portrayed above. I would make the point that minimal global coordination is necessary and that we have only one technologically functional solution to generate such large quantities of energy. Minimal economic support is required, it may be a net economic gain, and very little regulation or change in lifestyle is needed. We simply agree to switch all new power plant construction to nuclear power and let the old plants run to their natural life cycle. New nucelar plants are safer, cleaner and more effective than the old technology and they continue to improve. We only need to do it in advanced countries that already have nuclear capability as the rest of the world doesn’t emit much in comparison. I will even predict that economically this WILL be the future of humanity whether the government intervenes or not. There are plenty of economic and physics based reasons that this will happen.

            So there is your perfect green solution!! Are you on board? I am guessing you are one of the all of the above guys who thinks that 20 bad solutions will add up to one good one. You may also be an extremist and believe that all nuclear power is bad. I don’t mean to be disparaging but a mix of these positions seems to be the all-too-common pseudo-intellectual “compromise” position these days.

            So from your unscientific observation that CO2 warming is going to be dangerous, you make several points on your list which are based on ideology. I find most of them wholly objectionable and very poorly considered. They are heavily socialist in their form and I believe that were we silly enough to adopt the plan as laid out by you, it would lead to global strife the likes of which we have not seen in human history. Of course, socialists in general don’t recognize the need to incentivize individual economic gain as a requirement for human prosperity. They believe simple wealth redistribution solves the problem, and they universally underestimate the harm to industry. It seems syou suffer from this affliction but I don’t know your whole story. What doesn’t make sense, is “Wealthy” functional capitalist governments supplying funds to maintain non-functioning socialist dictatorships. The result of redistribution, whether by country or individual is the same — rich productive people giving money to non-productive ones to “help” them stay with the status quo.

            I have always found the argument interesting that we simultaneously say that we must globally help the weak with climate action. Since weak nations with non-functioning socialist/dictatorship style governments produce little of anything, it seems quite a leap to say that we need to do anything for them. They certainly aren’t any geological Greenhouse gas belching force right? Shouldn’t our limited resources be used to tackle producing energy that doesn’t emit CO2. Far more importantly, do we really want to enable those sick non-functional governments to continue creating human misery? With no visible mitigation required as no scientifically observable damage has happened, what would those countries exactly do with the money?

            “It requires that we accept that a sense of boundlessness that has been a central part of our cultures, particularly in the ‘frontier nations’ such as the USA or Australia is ending – from here on in human society has to have values centered on limitations and boundedness.”

            .
            Glenn, this one is the worst point of all. Your assumption is that our natural existence won’t curtail our behavior sufficiently. You believe that capitalist free society cannot provide, again like global warming, against all observation to the contrary, and you are ready to stop it now rather than allow it to continue to succeed as it has and should. You have made a great leap of faith here toward a form of central government controlled existence which has already produced more than enough observational evidence of its non-functionality. You are wrong about this and it is the foundation of most of your conclusions. I am thoroughly frightened of this thought process, as it is so prevalent in society, and so observationally wrong, and it leads to such bad places.
            .
            As I have never convinced anyone who strongly believes that capitalism and freedom don’t functionally govern of anything to the contrary, I don’t expect you will agree with what I am writing. However, I will agree with you that it isn’t perfect, but it isn’t designed to be. What the system represents mathematically, is economic evolution. By allowing the strong to survive and the weak to fail, the human condition averages to a much higher level of functionality. When we don’t allow the weak to fail, out of compassion and fairness or whatever, we doom both they and humanity to a lower average level of function. Unfortunately, economic output is a much more sensitive system than the ocean moderated climate, and small changes in “fairness” result in large changes in average economic function. Now of course there is a balance and I’m not saying there shouldn’t be safety nets for the truly helpless but that is something very different than sending money to non-productive dysfunctional societies, and people, to help with some unmeasurable aspect of global warming.
            .
            It is an oxymoron to believe in evolution, yet not understand free markets.
            .
            So no, I don’t suffer from the delusion that we don’t make mistakes. I do suffer from the belief that we need to be forced to reach higher through the danger of poverty such that we continue to try and then make even more mistakes. We need a world which allows us to continue to grow economically, producing even more goods and services at an ever more effective rate. We most certainly do not need a world of limited economy and heavy central control of our behavior. Remember, it was free people with minimal regulation produced the cleanest economies on earth with the greatest wealth for even our failing poor. While these are political views, I hold them as observationally sound.
            .
            So moving on, reclassifying CO2 as a significant pollutant is unscientific at this point but if we assume that is the case, it is an entirely separate issue from whether we need massive regulation and limitation of our behavior is the “solution”. I can imagine many “solutions” which don’t fit your preferred and highly political list.
            .
            You made an interesting point that deserves a slightly different viewpoint and will segue into the now long-awaited conclusion.
            .

            “But there is not one National Academy of Science anywhere in the world with a position against it. “

            .
            You interpret this correlation as though it were evidence of scientific accuracy, yet again, observational evidence contradicts this position. There are almost zero people on these science foundations who can be described as capitalist, pro-growth, limited government or free-market supporters. It is rather apparent that these groups are self-sorting to a high degree by selection and support of like-minded people. This is no big conspiracy, just a generally greater reward for like minded conclusions. Again, I find that pro-government thinkers fail to grasp or perhaps to state, the powerful effect of group incentivization. e.g. Jones invites Mann to speak instead of Spencer.

            Your evidence of accuracy, is very obviously — the opposite. Hell, you and I would probably have disagreements about the color of the sky, yet somehow these people across the globe all are in agreement about a science where the scientifically predictive climate models HAVE STATISTICALLY FAILED??!! Your position that the correlation of message appears to represent scientific accuracy, simply doesn’t make rational sense.
            .
            So I would suggest that my argument on AGW is scientific, and is superior in accuracy to your own argument as presented here. My position on AGW is different than the politically motivated, generally exaggerated, and now unscientific global warming story given by National Science academies around the world. Those scientific arguments I make, are separate from solutions to energy production, on which my opinions are also scientific and all of that is separate from my political views, which I hold as unscientific observationally based opinion.
            .
            So you asked:
            .
            “So who is more likely to be biased Jeff. Someone who is able to accept the implications of what AGW means, as highlighted in my list? Or someone who finds it deeply disturbing to accept that list?”
            .
            I must answer that since the list you present is highly political, a person who represents it as “requirement” for a solution to the general human condition, while failing to recognize obvious alternative paths, is far more likely to be biased than someone who disagrees with the proposed political solution but represents the disagreement as politics.

            Add in the fact that the alleged “problem” has not been observed and well.. you get the point.

          13. It’s so much simpler to analyse temperature and precipitation data and realise that it confirms that water vapour cools, as I have in my published study.

            Then just come to grips with the fact that, in assuming the surface would have been 255K without GH gases, the likes of James Hansen “forgot” that there would be no clouds reflecting (as the NASA energy diagrams show) 30% of incident radiation. Furthermore they “forgot” that a planet’s effective (radiating) temperature is based on emissivity 1.0000 which is not the value for a dry, rocky planet without vegetation that loves its CO2. If they hadn’t forgotten these two things they would have found there is no warming (and in fact some cooling) when we add water and existing GH gases. Oh, you say, but 0.04% carbon dioxide raises the radiating altitude by a whole meter, Well it also lowers the temperature gradient for the same reason that water vapour does, because thermodynamic equilibrium not only includes mechanical equilibrium, but also radiative equilibrium. The rest is in my book, so don’t bother arguing before you read it, for I’ll quote chapter and paragraph (where I’ve already rebuked your objection) in response..

            [REPLY: Quoting nonsense does not make for a good argument. -JEFF]

        2. By the way Frank, you use the word “heat” in a different way from physicists. Why do you change terminology from “kinetic energy” to “heat” when you write “That heat WILL thermally diffuse through the column, just not as fast as in these simulations – because the heat has about 10^6 times further to go to reach the same equilibrium state as we see in these simulations.” This statement is not something you can deduce from the models – it’s just your assumption. Why would the kinetic energy which became greater (per molecule) at the bottom of the model, then become less. Why would entropy increase and then decrease in violation of the second law of thermodynamics? The Uranus troposphere has had plenty of time to become isothermal, but it hasn’t. You cannot pull wool over Cotton’s eyes. Check mate.

          1. Doug, the simulations show what they show because they are applying the laws of physics to a frequently-colliding group of molecules in a box. A gradient in density, pressure and temperature develops soon after gravity is turned on (as potential energy is converted to kinetic energy as molecules fall). The pressure and density gradients persist; the temperature gradient dissipates. As best I can tell, the temperature gradient dissipates when the gas is relatively dense because warmer molecules from below travel a relatively short vertical distance and collide with cooler molecules just above. As best in can tell, this dissipation is thermal diffusion. It happens even in dense gases when the mean free path is very short.

            Thermal diffusion in the atmosphere is extremely slow. The mean free path of a molecule in the troposphere is about 1 um and there are approximately 10^9 collisions per second. The size of the box I’ve used in these simulations is often 10-100 times bigger than the mean free path in the simulations. The temperature gradient dissipates in a few seconds in the simulations because the kinetic energy doesn’t have far to travel. For kinetic energy to diffusion 1 km in the atmosphere requires transport across 10^9 “mean free paths”.

            When talking about these simulations, I’m confused about how many molecules must be involved before it is appropriate to refer to changes in kinetic energy as heat flow and to refer to changes in mean kinetic energy as temperature. So I haven’t been consist using these terms. Gas pressure and temperature are concepts that emerge from the physics of molecules in a box when they collide frequently. A box containing 10 gas molecules shows conservation of kinetic energy when the box is expanded. When the box contains several hundred molecules, kinetic energy is not conserved because PdV work is done.

            Doug, I’m not interested in “pulling the wool” over anyone’s eyes. Science is a search for the truth, not a game of chess. I think the truth can be found in molecular dynamics simulations, preferably one that are in 3D and are more sophisticated. The truth can’t be found on Uranus, because know very little about what is happening there. What we know comes from remote sensing and MODELS fit to remote sensing data. That material makes good wool because there is no way to evaluate its reliability.

          2. Frank, whenever a molecule moves with a component of vertical motion its KE alters, keeping the sum (KE+PE)=constant. As you admit, you are considering a height of only a few mean free paths. Roderich Graeff needed heights of at least a metre in order to detect very small temperature differences in his sealed insulated cylinders, but he did detect such in nearly every one of over 800 such experiments this century. But the “killer” is the Ranque Hilsch vortex tube which develops very significant temperature differences in a centrifugal force field. If the gravito-thermal effect did not exist then an Earth paved with asphalt and receiving just 163W/m^2 of direct solar radiation would have a mean temperature around -35C. It is “heat creep” which supplies the rest of the required energy.

            You can’t do away with the Second Law. An isothermal state has unbalanced energy potentials simply because the molecules at the top have more gravitational PE and yet the same KE. If you have unbalanced energy potentials you do not have maximum entropy. If you do not have maximum entropy you do not have thermodynamic equilibrium. But the Second Law says thermodynamic equilibrium (with maximum entropy and no unbalanced energy potentials) will evolve. That’s what happens every calm night in the early pre-dawn hours when convection stops but the temperature gradient remains. If I were wrong then you have no explanation for the “heat creep” process explained over two chapters in my book. And if there is no heat creep then you have no way of explaining how the necessary energy gets from the upper troposphere of Venus down to the surface and actually raises its temperature. And we do have measured data from Russian probes dropped to the surface of Venus.

            Why would it be Frank, that every planetary troposphere exhibits the gravitationally induced temperature gradient reduced a little (as we would expect) by the inter-molecular radiation that occurs between some “greenhouse” molecules in its atmosphere? Is it just a coincidence that the core of Uranus is a just the right temperature (5,000K) such that the gases cool to just the right temperature (320K) at the base of its nominal troposphere (thousands of kilometres above the solid core) and continue cooling with just the right temperature gradient so as to get down to just the right radiating temperature (about 60K) at just the right altitude (about a further 350Km up) where there is a methane layer doing nearly all the absorbing and re-radiating of the weak solar radiation? Extend such questions to all planets with significant atmospheres (including gas planets and some satellite moons) in our Solar System and what is the probability that all exhibit a similar temperature gradient? What would happen if they all cooled off in another billion years, rather than being kept at current temperatures with energy from the Sun?

          3. Regarding other points you make, Frank:

            You need a few picograms (10^−12) of gas to determine temperature as we know from statistical mechanics.

            It does not matter how fast or slow the non-radiative heat transfers are, because there has been the life of the planet to get to thermodynamic equilibrium. But, that said, the thermal energy absorbed in the upper troposphere of Venus (where it’s <400K) and above does get to the surface and raise its temperature from 732K to 737K over a period of four months. Molecules move at about 1,800Km/hour between collisions, after all. If there were a straight line of molecules, KE could be relayed up 1Km in two seconds.

            It is just not possible for entropy to decrease. If as you say the temperature gradient develops that means entropy has increased to a maximum (at thermodynamic equilibrium) and so it can't then go into reverse gear and start decreasing, as would be happening if the temperature gradient were able to somehow level out in a gravitational field, having previously been in a state of thermodynamic equilibrium. Equilibrium is equilibrium. Systems don't move away from equilibrium states without any reason to do so, such as the addition of new energy.

            As far as I can understand what you are doing with the box is merely deducing that mean KE of all the molecules is conserved. Of course it is once thermodynamic equilibrium is attained. After all, you agreed the density gradient is preserved and so mean PE is constant, and thus mean KE is constant. All along I've said the mean sum (PE+KE)=constant. But that does not mean that in a column of air 1Km high the mean KE at the top won't represent a temperature that is, say, 7 degrees colder than the temperature represented by the mean KE at the bottom.

          4. Frank, Jeff and others:

            It’s not hard to understand the state of thermodynamic equilibrium. As explained in my book, start by imagining two horizontal planes of molecules with (PE + KE) the same for every molecule. If the planes are separated by the mean free path or less, then a molecule moving down from the top plane gains KE and loses the same amount of PE. The opposite happens for a molecule moving upwards between the planes. The important point is that, when either of these molecules collides with a molecule in the other plane it has gained or lost just the right amount of KE such that it has the same KE as the molecule in the plane it reaches. And because the KE matches in all collisions there is no propensity for further dispersion of KE (conduction, diffusion or convection) and thus we have the state of thermodynamic equilibrium,

            Only in a horizontal plane is there no change in KE during free path motion and so temperature does indeed level out in a horizontal plane, but not in a vertical plane. This was what Loschmidt visualised back in the 19th century and he was right. All attempts to prove him wrong (like Robert Brown’s) don’t demonstrate an understanding of thermodynamic equilibrium and the fact that, if you combine two systems you just get a new state of thermodynamic equilibrium with a new temperature gradient, but certainly no perpetual circulation of energy.

            You all need to come to grips with the process which the Second Law describes, and how it applies to all forms of energy and equilibrium. (Hence it is also the Second Law which tells us why there is a density gradient.) Then you need to understand entropy and the state of thermodynamic equilibrium (which includes mechanical equilibrium) and why that state must have no unbalanced energy potentials.

            Note that the Second Law applies to every independent process or to a sequence of dependent processes. An example of a “net” effect is seen in a siphon where water flows up the short side (the first dependent process) and down the longer side (the second dependent process) and so entropy does increase for the combination of processes or participating systems. But they must be dependent processes or as stated here be “participating systems.” If you cut the siphon at the top you no longer have two participating systems that are inter-dependent.

            Now, the electromagnetic energy in back radiation can only be used for a part of the surface’s own quota of outward electro-magnetic radiation. If its EM energy were converted to thermal (kinetic) energy in the surface then that KE could escape by conduction, evaporation etc and there is no subsequent dependent radiation process. So the first process transferring thermal energy from cold to hot can’t happen, even if those in Climatology Carbonland think it can. All one-way radiation only ever transfers thermal energy from hot to cold because there are no molecules involved and being affected by gravity. Hence the only way the temperature gradient builds up in a high troposphere (as for Uranus and Venus) where no significant direct solar radiation heats a surface (if any) is from the upper colder regions to the warmer regions by non-radiative heat transfer which can “creep” up the temperature plot if thermodynamic equilibrium has been disturbed with new energy at the top, and now needs to be restored by spreading that new energy in all accessible directions away from its source.

            [Doug, I am not involved in this discussion anymore. You are unqualified to discuss thermodynamics at a rational level. I had hoped for improvement but you have proven your OWN theory wrong here with your own words and quite frankly you appear unable/unwilling to grasp your own obvious contradictions. Go spend time with your kids or something, at least that would be productive. — JEFF]

          5. And, by the way, Frank you said “The pressure and density gradients persist; the temperature gradient dissipates.” Well, since pressure is proportional to the product of density and temperature, and you are saying pressure and density at any particular level do not change, but temperature does change, it would appear that you scrapped the Ideal Gas Law as well as the Second Law of Thermodynamics.

          6. What I have proved is that there is no warming for greenhouse gases to do. The Earth’s surface would be just as warm (287.5K) when it receives solar radiation of a quarter of 1,365W/m^2 and has emissivity 0.88. What is it that you can’t understand about that, Jeff?

  33. In summary, it’s borderline with a 100% CO2 atmosphere as to whether direct solar radiation to the surface, or “heat creep” (due to the gravito-thermal effect) will make the surface hotter. In comments above I got similar results (namely 295K and 292.5K) and I’m the first to admit there’s a fair margin of error in each estimate. Of course for a transparent atmosphere (100% nitrogen) it’s just as if the atmosphere weren’t there as far as the maximum temperature, so we just use the Stefan Boltzmann calculations with a realistic value for emissivity.

    Obviously a dry rocky planet without water and vegetation will have lower emissivity (and thus a higher temperature) than the existing Earth. The existing Earth also receives only 48% as much solar radiation as would the surface of the nitrogen planet if we assume no albedo (because of a lack of clouds) and no absorption by the nitrogen atmosphere. So there are two reasons why the surface of the nitrogen planet is raised to a higher temperature than the surface of the real Earth could be raised by direct solar radiation – lower emissivity and greater radiative flux getting to the surface.

    For the real Earth the solar radiation could only warm it to about 35 degrees below freezing point. So the real Earth’s temperature is determined by the gravito-thermal effect and its surface temperature is supported by the “heat creep” process explained in my book. Likewise for planets with far higher tropospheres like Venus and Uranus. But for Mercury with hardly any atmosphere the direct solar radiation determines the temperature.

    On the real Earth, the gravito-thermal effect would cause the surface to be about 300K if it weren’t for water vapour reducing the temperature gradient, thus causing the thermal plot to rotate about the pivoting altitude lowering the supported surface temperature to about 288K. Carbon dioxide has an infinitesimal effect also lowering the temperature gradient by inter-molecular radiation. It might be argued that is raises the pivoting altitude a bit, but I estimate that to be only by about 1 metre for the existing 0.04% concentration. It’s very debatable as to whether that warms the surface more than the lowering of the gradient cools it. Each has such an infinitesimal effect on the surface temperature (probably less than ±0/01 degree so it’s not worth worrying about. But the hoax that CO2 warms by 1 to 3 degrees if doubled is certainly worth worrying about and needs to be stopped in its tracks.

  34. In Roy Spencer’s October data just published there’s still no net warming, since 1998, and nor will there be until at least 2028. Everything in Earth’s climate pattern is still following the two main natural cycles which correlate beautifully with the 934 year and 60 year cycles in the inverted plot of the scalar sum of the angular momentum of the Sun and all the planets. I hope some of the lukes and warmists live to see the start of 500 years of cooling after the next 60 year maximum in 2058. Carbon dioxide has nothing to do with it.

  35. Footnote; Let me give credit to Teofilo Echeverria who (as was quite unknown to me at the time I published my book) had apparently been striving since the 1950’s to get his papers about this same “heat creep” process published. Finally, I guess towards the end of his life, he also has published it in a book which I linked in an earlier comment. Nothing else explains the temperatures throughout our solar system and the required energy flows to maintain such. Nothing else explains reality.

  36. So Jeff, your king can only get out of check if you can show my input data to be substantially wrong in these Stefan Boltzmann calculations …

    Input data

    Mean solar radiation to surface of nitrogen planet where there’s no backradiation: 1360 / 4 = 340 W/m^2

    Emissivity: 0.85 (based on several measured values for various rock types quoted in a comment above – probably high, but you need to show it to be higher still.

    Output; Temperature = 289.81391462999426K

    You’ll never get it down to 255K and get that “33 degrees of warming” so which move gets you out of checkmate, Jeff?

  37. You guys can argue all you like about Earths without GHG or atmospheres, but empirical data (30 years of real world temperature data from three continents) has been used to prove with statistical significance that moist regions have lower daily maximum and minimum temperatures than drier regions at similar latitudes and longitudes. Water vapour cools because its radiation properties reduce the effect of the gravitationally-induced temperature gradient, thus lowering the thermal profile at the surface end.

    It is ludicrous to think that people can be so gullible as to believe that water vapour jacks up the temperature at the surface end by about 30 degrees whilst at the same time we know it reduces the temperature gradient by nearly a third in magnitude. What on Earth would happen to radiative balance if both these really did occur simultaneously? Are you one of the gullible people, or do you think?

  38.  

    Jeff and others:

    All alarmist comments are based on IPCC documentation, and that documentation says the Earth’s surface would be 33 degrees colder without greenhouse gases. Water vapour is thus meant to be doing most of that warming from about 254.5K to about 287.5K to the nearest half degree.

    But in calculating the 254.5K temperature they fail to alter the albedo which, according to their energy diagrams includes 30% of solar radiation reflected back to space by those clouds which would only exist if the greenhouse pollutant, water vapour actually existed. But they have assumed it doesn’t in this scenario. So they incorrectly use only 70% of a quarter of the solar flux (1365W/m^2) and then they also assume incorrectly that emissivity is 1.0000, and so then then incorrectly get that temperature of 254.5K in Stefan-Boltzmann calculations.

    The ramifications of this enormous oversight are huge, because if they had not reduced the radiation by 30% due to the clouds that don’t exist. and if they had used a more realistic emissivity for a dry, rocky planet – say 0.88, then they would have got a temperature of 287.58K which is close enough to what is the existing mean temperature with GH gases that are thus doing no warming at all. I would add that the emissivity is more likely to be lower than 0.88, this giving a higher temperature above 290K, and so GH gases are cooling Earth.

  39. Even if you don’t understand the resonance process described in my paper “Radiated Energy and the Second Law of Thermodynamics” and even if you think that the electro-magnetic energy in a one-way independent passage of radiation from a cooler troposphere to a warmer surface can be converted back to thermal energy and somehow not violate the Second Law with this heat transfer from cooler to warmer, then consider what would happen. The radiation from the cooler source is well known not to penetrate the ocean surface by more than 10 microns, whilst that from the Sun penetrates more than 10 metres. There would thus be a supply of thermal energy (you claim, not me) comparable with that from the Sun but concentrated into less than one-millionth of the depth of water, and obviously boiling it. Suppose the ocean temperature is 290K and this boiling 10 micron surface layer is 373K. Mix it all up and we get (0.000001 x 373) + (0.999999 x 290) = 0.000373 + 289.999710 = 290.000083. So that’s a warming of 0.000083 degree. Something to spend billions worrying about?

    [Doug, not only do I recognize it, I actually steered you toward it — ya know, back in the day. You had no other option without admitting your error, but we seem to have forgotten our roots eh? —Jeff]

    [BTW, I steered you that way because it was refutable. It just took you years to get where I was when we were talking. Literally years to catch up and it pisses me off that I had to wait! I am not being facetious.

    I’m sorry our relationship has gone this way Doug. I’ve been a little mean over the years. Please move on to some other blog so I don’t continue to exercise my Id. Have maximum fun with your theory rather than bring it here. I think I’m the meanest blogger you’ve worked with, yet at the same time, I gave you the biggest chance.

    Spend 10 minutes looking back at the very beginning of our conversations. Open your mind a tiny bit and recognize how your discussions have changed. Look at the old chess game with an objective mind and you will see that what I say is true.

    At this point, you have actually admitted global warming. How far you have come yet you don’t realize it. It is my fault Doug. You try so hard to be scientific, right until the point where you realize your conclusion doesn’t fit and then mush-brain ensues.

    I’m tired of being tough on you. Free yourself from your nemesis and write on every other blog how stupid and ignorant I am. You have my full permission! ]

    1. No. It was Claes Johnson who explained the resonance process before me.

      How about reading the rest of the comment? Are you or are you not in agreement that there could be no more than 0.000083 degree of warming if the resonance process did not happen, which it does and I have never changed my stand on what I wrote in March 2012 in Radiated Energy and the Second Law of Thermodynamics?

      I don’t change my understanding of these matters, Jeff, but over the course of many thousands of hours of research, I have extended that understanding such that I can now explain all planetary temperature data, backed up with empirical data, which is something you have a long way to get to,

    2. i admitted that the globe is warming and that is documented in the Appendix of my March 2012 paper from which I quote below (snip where you like) …

      APPENDIX

      Frequently Asked Questions

      It is suggested that the reader studies the answers to all these seven questions in order,
      because they build upon each other. The arguments herein can lead to no other conclusion
      than that any surface warming by trace gases in the atmosphere is a physical impossibility.

      Q.1 How do you explain the fact that the Earth has been warming?

      Technically the Earth is currently in an interglacial period and the last few glacial periods
      have occurred at roughly 100,000 year intervals. This indicates the possibility of there being
      natural cycles, short and long, which appear to be related to astronomical orbital events. For
      example, the planet Jupiter has an effect on the eccentricity of the Earth’s orbit in such a way
      that the difference in the distances between the Sun and the Earth at the aphelion and
      perihelion can vary (over many thousands of years) from just over 0% when its orbit is nearly
      a true circle, up to about 5% when it is elliptical. Such variations affect the mean distance and
      that will then affect the mean radiative flux over the course of a year.

      Many scientists also believe there is clear evidence of a 60-year cycle which may be related to
      the alignment of the planets Jupiter and Saturn every 59.6 years. This cycle appears to have
      been the main cause of the observed temperature increases which raised alarm in the 30 years
      or so leading up to the maximum in 1998. However, there is also a longer cycle which
      appears to be very approximately 1,000 years. The underlying trend in the rate of increase
      can be detected when a trend line is added to the plot below (from this site) which shows 30
      year trend gradients.

      (You can look up that plot in the paper and it shows 0.05 degree/decade of long-term warming which will continue until the year 2058, followed by 500 years of cooling.)

    3. What I agree with is clearly stated in my book and my 2012 paper cited therein, because there is still nothing in the paper with which I disagree.

      As I said at the outset, Jeff, you could have saved us both a lot of time if you had spent an hour reading the book and discussing what is actually written therein, rather than what your wild imaginings think I write, say, understand or believe. Carbon dioxide and water vapour do not cause Earth’s surface to be warmer than otherwise for numerous reasons explained in my book, papers and articles. You have “taught” me nothing, Jeff. Sadly you are too narcissistic to deign to try to learn anything from me, which you could regarding the “heat creep” process that transfers thermal energy downwards in planetary tropospheres by non-radiative processes.

      [Doug, are you kidding me? What a strange universe your personality resides in. JEFF]

      1.  

        Yes, Jeff, I’m deadly serious about the validity of heat creep, being downward non-radiative heat transfer up the thermal profile restoring thermodynamic equilibrium with its associated (environmental) temperature gradient. That’s the first thing you’d need to prove wrong with valid physics for the $5,000 reward, and the second thing is to produce a study with similar methodology to that in the book, but which, instead of proving water vapour cools, proves the opposite, namely that it warms in the order of magnitude assumed by IPCC authors. Yes I’m confident my hypothesis is correct because it stands up to empirical testing and can be used to explain planetary tropospheric, surface and sub-surface temperatures throughout the Solar System.

        Now I’m out of action for the weekend, caring for my young(ish) wife who had an operation today, and then taking our 8 year old son to compete all day tomorrow in the Zone junior (under 18) lawn bowls for which he was selected in the team from all bowling clubs in the north-west of Sydney. Enjoy this: https://www.youtube.com/watch?v=viB48QW-AvY

        That’s all folks.

        [REPLY: You have been proven wrong already Doug. In your own words. — JEFF Glad to hear the fam is doing well.]

         

      2. And you, Jeff, have posted no counter arguments to this comment above. So raising the radiating altitude is more than offset by reducing the mean temperature of whatever is doing the radiating and reducing the solar flux reaching the surface, and so you have absolutely no valid evidence that greenhouse gases could warm Earth’s surface. At least three other commenters have agreed with me about this comment.

        So Checkmate.
        [Whatever Doug….]

  40.  

    Fortunately water vapour keeps the world somewhat cooler than it would otherwise be, as I have proved from 30 years of temperature and precipitation records from three continents. It does so because its radiating properties reduce the magnitude of the temperature gradient, as do such properties of carbon dioxide to a much lesser extent, and methane on Uranus.

    Bye

     

  41. PS: Some comments from Roy Spencer’s thread on the October data …

    Jerry L Krause says:

    November 7, 2014 at 9:29 AM

    Hi Doug:

    Very glad to see that someone else recognizes that some of which you write is right on. While some details might not be precisely accurate, everything you wrote is generally true. I will be composing a longer reply to your comment but hopefully others, silent or not, will recognize that the simple facts you referred to are correct and write their support for what you wrote.

    Have a good day,

    Jerry

    Reply

    JohnKl says:

    November 7, 2014 at 2:52 PM

    Hi Jerry L Krause,

    Doug makes an excellent point. In the past I’ve pointed out that the albedo levels chosen by the CAGW alarmists to arrive at their bogus 33 degree colder without GHG’s conclusion seems completely arbitrary and a small adjustment in the ASSUMED albedo makes the entire paranoid CAGW phantasm completely unworkable. Doug did a great job of illustrating this and other facts and pointing it out.

    Have a great day!

  42. There are several serious errors in the “physics” of the radiative greenhouse conjecture. As discussed in a comment above, there is not necessarily any significant difference between what the surface temperature would be without all greenhouse gases and what it is today. The issue comes down to whether the emissivity of a rocky dry planet would be greater or less than 0.88 and, given that many rocks, as well as soil, have a lower emissivity than that, then we can deduce that it would have been warmer without GH gases. That really should not surprise you, because you know that water vapour is what forms clouds (even though the IPCC authors thought the oceans and clouds would still be there without water vapour on the planet) and we know that there is 30% reflection by clouds (well at least that’s what NASA et al tell us) and we are also told that the atmosphere absorbs about 20% of incident solar radiation.

    What maintains the surface temperature is thus not radiation from GH gases, because it would have been hotter without them, rather like the Moon that reaches temperatures around 130C. But Earth would not cool down as quickly as the Moon does if it had the same atmosphere but no GH gases, because nitrogen and oxygen would then hold 100% of the energy in the atmosphere – even more than the 98% they currently hold.

    What does maintain the surface temperatures of Earth and Venus (and that at the base of the nominal troposphere of Uranus) is the gravitationally-induced temperature gradient which a correct understanding of the Second Law of Thermodynamics let’s us deduce is in fact the very state of thermodynamic equilibrium which that law says will evolve autonomously.

    And that’s why it doesn’t matter how much back radiation slows that small portion of surface cooling which is by radiation. All the cooling by radiation and non-radiative processes (unaffected by back radiation) slows down and possibly stops in calm conditions in the early pre-dawn hours because the gravito-thermal effect is maintaining the supporting temperature. Then, especially when there is excessive cloud cover over the oceans, the Sun’s energy absorbed above the clouds can actually make its way down to the ocean surface (and below) warming the oceans by non-radiative processes, not by direct solar radiation which mostly passes through the thin surface layer and could barely raise the mean temperature of an asphalt paved Earth above -35C.

  43. Well Jeff, you have passed up the opportunity of a lifetime to present the 21st century new paradigm in climate science on your blog. I didn’t come here to learn from you nor to seek your agreement that the physics in my book is correct. I know that to be the case, and I know that the hypothesis is confirmed by data throughout the Solar System. That can never and will never be able to be claimed for the radiative forcing conjecture. In that you don’t deign to read the book, even though I offered to post you a free one, your comments about what you think is in the book are of course way off because you don’t have the insight and correct understanding of thermodynamic equilibrium that I have. You probably don’t even know why it is so critical for a correct understanding of atmospheric physics. I leave you with this retired physics educators’ comments …

    “Doug Cotton shows how simple thermodynamic physics implies that the gravitational field of a planet will establish a thermal gradient in its atmosphere. The thermal gradient, a basic property of a planet, can be used to determine the temperatures of its atmosphere, surface and sub-surface regions. The interesting concept of “heat creep” applied to diagrams of the thermal gradient is used to explain the effect of solar radiation on the temperature of a planet. The thermal gradient shows that the observed temperatures of the Earth are determined by natural processes and not by back radiation warming from greenhouse gases. Evidence is presented to show that greenhouse gases cool the Earth and do not warm it.”

    John Turner B.Sc.;Dip.Ed.;M.Ed.(Hons);Grad.Dip.Ed.Studies (retired physics educator)

  44. In my book “Why It’s Not Carbon Dioxide After All” I put forward an hypothesis which correctly explains temperature data throughout the Solar System and no doubt beyond. There is no scope for any additional warming by so-called greenhouse gases, and in fact I show why these gases cool rather than warm. Empirical evidence in a study I published therein also confirms that the main greenhouse gas water vapour certainly cools. This completely debunks the greenhouse conjecture. All global warming is natural and will end around the year 2058 after which nearly 500 years of cooling will commence.

    In applying my hypothesis to Venus I allowed for the variation in specific heat of carbon dioxide, so we get a curved temperature profile. It still works, just as it does on Uranus. There’s little point in discussing a planet like Mars with an insignificant atmosphere, but the hypothesis still works.

    But I don’t use barometric equations because pressure is irrelevant. Pressure is proportional to the product of temperature and density. High pressure does not maintain high temperatures. Temperature and density cause pressure, not the other way around.

    The state of thermodynamic equilibrium (which the Second Law of Thermodynamics says will be approached) has a density gradient and a temperature gradient. The pressure gradient is merely a corollary. Until you all understand this, you understand nothing about atmospheric physics.

    And that is why calculations based on barometric equations give only approximate results. Those who think temperature is caused by pressure are mistaken. Where does the required thermal energy come from? That’s the whole point of my book wherein I’m one of only two authors to explain (independently) the energy flows from valid physics.

    None of you understands how the energy gets from the cooler regions (<400K on Venus) where it is absorbed from Solar radiation to the hotter Venus surface and raises its temperature from 732K to 737K over the 4 months of sunshine. To raise a temperature you need net input of thermal energy, and that does not come from radiation into the Venus surface. Nor does pressure magically supply energy.

Leave a comment